Trans Finito s

112
NUMEROS TRANSFINITOS Por: DIEGO ALEJANDRO MEJ ´ IA GUZM ´ AN

description

NUMEROS TRANSFINITOSPor:DIEGO ALEJANDRO MEJ´I A GUZM´A N

Transcript of Trans Finito s

NUMEROS TRANSFINITOS

Por:

DIEGO ALEJANDRO MEJIA GUZMAN

2

INDICE GENERAL

1.. NUMEROS ORDINALES . . . . . . . . . . . . . . . . . . . . . . . . . . . . . . . . . . . 5

1.1. AXIOMAS DE ZF . . . . . . . . . . . . . . . . . . . . . . . . . . . . . . . . . 5

1.2. CONJUNTOS BIEN ORDENADOS . . . . . . . . . . . . . . . . . . . . . . . . . 10

1.3. ESTRUCTURA DE LOS NUMEROS ORDINALES . . . . . . . . . . . . . . . . . . 15

1.4. SOBRE CLASES . . . . . . . . . . . . . . . . . . . . . . . . . . . . . . . . . . 22

1.5. INDUCCION Y RECURSION TRANSFINITA . . . . . . . . . . . . . . . . . . . . 25

1.6. ARITMETICA DE ORDINALES . . . . . . . . . . . . . . . . . . . . . . . . . . . 34

2.. NUMEROS CARDINALES . . . . . . . . . . . . . . . . . . . . . . . . . . . . . . . . 49

2.1. CARDINALES Y ALEPHS . . . . . . . . . . . . . . . . . . . . . . . . . . . . . 50

2.2. ARITMETICA CARDINAL . . . . . . . . . . . . . . . . . . . . . . . . . . . . . 58

2.3. NUMERO DE HARTOGS . . . . . . . . . . . . . . . . . . . . . . . . . . . . . . 62

2.4. COFINALIDAD . . . . . . . . . . . . . . . . . . . . . . . . . . . . . . . . . . 66

2.5. EXPONENCIACION ARITMETICA Y CARDINAL INFINITA . . . . . . . . . . . . 75

3.. BUENA FUNDACION . . . . . . . . . . . . . . . . . . . . . . . . . . . . . . . . . . 85

3.1. JERARQUIA ACUMULATIVA . . . . . . . . . . . . . . . . . . . . . . . . . . . 86

3.2. RELACIONES BIEN FUNDADAS . . . . . . . . . . . . . . . . . . . . . . . . . 93

3.3. TEORIA DE CONJUNTOS CON EL AXIOMA DE REGULARIDAD . . . . . . . . . . 102

4

Capıtulo 1

Numeros Ordinales

La estructura de los numeros ordinales es definitiva para la redaccion en el lenguaje matematico

en el sentido de que, como estos numeros permiten medir la “longitud” de un buen orden, se puede

enumerar cualquier tipo de sucesion, lo cual permite mas versatilidad y precision en el lenguaje. Ba-

jo este criterio, los ordinales se presentan como una extension transfinita de los numeros naturales,

mas alla de lo finito, y poseen propiedades como induccion, recursion y una aritmetica. Todos estos

aspectos los desarrollamos en este capıtulo.

Para fines de aplicacion de los resultados de este capıtulo en el uso de modelos de la Teorıa

de Conjuntos, seremos explıcitos de cuando un resultado requiere el axioma de Partes, el axioma

del Infinito, el Axioma de Eleccion y el axioma de Regularidad. En principio, todos los resultados

de este capıtulo a partir de 1.3 se prueban en ZF−−P−Inf (la Teorıa de Conjuntos sin los cuatro

axiomas mencionados anteriormente), a menos que se indique lo contrario.

1.1. Axiomas de ZF

La siguiente definicion contiene la notacion sobre los axiomas que utilizaremos para desarrollar

La Teorıa de Conjuntos en este documento.

1.1 Definicion. La Teorıa de Conjuntos ZF (Zermelo-Fraenkel-Skolem) es una teorıa de primer

orden con igualdad cuyo lenguaje es {=,∈} y cuyos axiomas son

ZF0 Axioma de Existencia. ∃A(A = A).

ZF1 Axioma de Extensionalidad. ∀A,B[∀x(x ∈ A ⇐⇒ x ∈ B)⇒ A = B

].

ZF2 Axioma de Regularidad. ∀X[(∃x(x ∈ X))⇒ ∃y∈X¬∃z∈X(z ∈ y)

].

ZF3 Esquema Axiomatico de Comprension. Si ϕ(x) es una formula cuyas variables libres estan

entre x,A,w1, . . . , wn, entonces

∀A,w1,...,wn∃Y ∀x(x ∈ Y ⇔ x ∈ A ∧ ϕ(x)).

ZF4 Axioma de Pares. ∀a,b∃Y (a ∈ Y ∧ b ∈ Y ).

ZF5 Axioma de Union. ∀A∃Y ∀x,z((z ∈ x ∧ x ∈ A)⇒ z ∈ Y ).

ZF6 Esquema Axiomatico de Reemplazamiento. Si ϕ(x, y) es una formula cuyas variables libres

estan entre x, y,A,w1, . . . , wn, entonces

∀A,w1,...,wn [(∀x∈A∃!yϕ(x, y))⇒ ∃Y ∀x∈A∃y∈Y ϕ(x, y)].

ZF7 Axioma del Infinito (Inf ). Bajo los axiomas ZF1, ZF3, ZF4 y ZF5 se pueden definir las

nociones ∅, {a, b},⋃A y, en consecuencia, S(x) ≡ x ∪ {x}. Bajo estos axiomas, el Axioma

del Infinito se enuncia como ∃X(∅ ∈ X ∧ ∀x∈X(S(x) ∈ X)).

ZF8 Axioma de Partes (P). ∀A∃Y ∀z(z ⊆ A⇒ z ∈ Y ).

Bajo los Axiomas ZF1, ZF3 y ZF4 se define par ordenado como (x, y) = {{x}{x, y}}. Esto da pie

a definir las nociones de relacion, funcion, orden parcial (C.P.O. Conjunto parcialmente ordenado),

orden total (C.T.O. Conjunto totalmente ordenado) y buen orden (C.B.O. Conjunto bien ordenado).

Bajo estas nociones, denotamos

AC Axioma de Eleccion. ∀A∃R(〈A,R〉 es C.B.O.)

Definimos por ZFC a la teorıa ZF + AC. Adoptamos tambien la notacion

ZF− es ZF sin el axioma ZF2.

ZF−Inf es ZF sin el axioma ZF7.

ZF−P es ZF sin el axioma ZF8.

Esta notacion la combinamos para generar teorıas como ZF−−P (ZF sin los axiomas ZF2 ni ZF8),

ZF−−P−Inf (ZF sin los axiomas ZF2, ZF7 ni ZF8) o ZFC−−P (ZF sin los axiomas ZF2 ni

ZF8 y con AC).

6

La axiomatica propuesta en la definicion anterior contiene la forma debil de los axiomas ZF4,

ZF5, ZF6 y ZF8. En la practica se utilizan, en vez de los axiomas mencionados, las formas fuertes

expuestas en el siguiente resultado. La importancia de conservar la forma debil es util para compro-

bar que ciertas clases transitivas son modelos internos de ZF.

1.2 Lema. (a) El axioma ZF0 es redundante (se sigue del Calculo de Predicados).

(b) De ZF3 se sigue el Axioma del Vacıo. ∃Y ∀x(x /∈ Y ).

(c) De ZF3 se sigue que ZF4 equivale a ∀a,b∃Y ∀x(x ∈ Y ⇔ (x = a ∨ x = b)).

(d) De ZF3 se sigue que ZF5 equivale a ∀A∃Y ∀z(z ∈ Y ⇔ ∃x∈A(z ∈ x)).

(e) De ZF3 se sigue que ZF8 equivale a ∀A∃Y ∀z(z ∈ Y ⇔ z ⊆ A).

(f) Sea ϕ(x, y) una formula cuyas variables libres estan entre x, y,A,w1, . . . , wn. De ZF3 se sigue

que ZF6 para ϕ(x, y) equivale a

∀A,w1,...,wn [(∀x∈A∃!yϕ(x, y))⇒ ∃Y ∀y(y ∈ Y ⇔ ∃x∈Aϕ(x, y))].

(g) De ZF1 y ZF3 se sigue la unicidad ∃!Y en los incisos (b)-(f).

Demostracion. Para los incisos (c)-(f) solo probamos una implicacion, pues la recıproca es directa.

(a) La afirmacion A = A⇒ ∃A(A = A) es logicamente valida. Por lo tanto, se sigue ZF0.

(b) De ZF0 existe un A (tal que A = A). Luego, aplique ZF3 a A y a la formula ϕ(x) ≡ x 6= x

para generar un Y tal que ∀x(x ∈ Y ⇔ x ∈ A∧x 6= x). Este Y satisface la afirmacion buscada.

(c) Dados a y b, por ZF4, existe Y ′ tal que a, b ∈ Y ′. Por ZF3 aplicado a ϕ(x) ≡ x = a ∨ x = b

y A ≡ Y ′ obtenemos que existe Y tal que ∀x(x ∈ Y ⇔ (x ∈ Y ′ ∧ (x = a ∨ x = b))). Este Y

satisface la afirmacion buscada.

(d) Dado A existe, por ZF4, un Y ′ tal que ∀x,z(z ∈ x ∧ x ∈ A⇒ z ∈ Y ′). Como en la prueba

anterior, se obtiene el Y esperado al aplicar ZF3 a A ≡ Y ′ y a ϕ(z) ≡ ∃x∈A(z ∈ x).

(e) Prueba analoga a las dos anteriores.

(f) DadosA,w1, . . . .wn suponga que ∀x∈A∃!yϕ(x, y). Por ZF6 existe Y ′ tal que ∀x∈A∃y∈Y ′ϕ(x, y).

Por ZF3 existe Y tal que ∀y(y ∈ Y ⇔ y ∈ Y ′ ∧ ∃x∈Aϕ(x, y)). Para concluir que Y es quien

buscamos, basta probar que ∃x∈Aϕ(x, y)⇒ y ∈ Y ′. En efecto, si existe un x ∈ A tal que

ϕ(x, y) entonces, como x ∈ A, existe un y′ ∈ Y ′ tal que ϕ(x, y′). Luego, y = y′ ∈ Y ′.

7

(g) Inmediato.

Del lema anterior se siguen que las definiciones usuales de∅, {a, b}, {a},⋃A, {x ∈ A / ϕ(x)},

P(A) (partes de A), entre otras, en La Teorıa de Conjuntos. Prestamos especial atencion al axioma

ZF6 (Reemplazamiento) ya que permite definir conjuntos a partir de operaciones definidas sobre

un conjunto. La premisa ∀x∈A∃!yϕ(x, y) le da un contenido funcional a la formula ϕ(x, y) sobre

el conjunto A: “Para todo x ∈ A existe una unica imagen y relacionada mediante ϕ(x, y)”. Ası,

El Axioma de Reemplazamiento ((f) del Lema anterior) nos dice que el conjunto de imagenes de A

sobre ϕ, es decir, {y / ∃x∈Aϕ(x, y)}, esta bien definido.

En vista de lo anterior, cuando tenemos terminos definidos de la forma t(x), por ejemplo,

t(x) = A ∪ x, t(x) =⋃x o t(x) = P(x), es posible determinar directamente del Axioma

de Reemplazamiento que {t(x) / x ∈ A} es un conjunto bien definido. Los resultados que men-

cionamos y probamos de aquı en adelante en este capıtulo corresponden a ZF−−P−Inf , a menos

que se indique lo contrario.

1.3 Lema. (a) Sea ϕ(x) una formula donde Y ′ no es libre. Entonces {x / ϕ(x)} es conjunto si y

solo si ∃Y ′∀x(ϕ(x)⇒ x ∈ Y ′) (esto ultimo denota que ∃Y ′({x / ϕ(x)} ⊆ Y ′)).

(b) Sea t(x) un termino definido. Dado un conjunto A, {t(x) / x ∈ A} ≡ {y / ∃x∈A(y = t(x))}es un conjunto bien definido.

(c) Sea A un conjunto, t(x) un termino definido y A = {t(x) / x ∈ A}. Si ϕ(y) es una formula

donde x no es libre, entonces

(i) ∀y∈Aϕ(y)⇔ ∀x∈Aϕ(t(x)). (ii) ∃y∈Aϕ(y)⇔ ∃x∈Aϕ(t(x)).

(d) En el inciso anterior, defina⋃x∈A t(x) ≡

⋃{t(x) / x ∈ A} y

⋂x∈A t(x) ≡

⋂{t(x) / x ∈ A}

(este ultimo cuando A 6= ∅). Entonces

(i)⋃x∈A t(x) = {z / ∃x∈A(z ∈ t(x))}. (ii)

⋂x∈A t(x) = {z / ∀x∈A(z ∈ t(x))}.

Demostracion. (a) Supongamos que existe Y ′ tal que ∀x(ϕ(x)⇒ x ∈ Y ′). De ZF3 tenemos que

{x ∈ Y ′ / ϕ(x)} es un conjunto y, claramente, {x / ϕ(x)} = {x ∈ Y ′ / ϕ(x)}.

(b) Es claro que ∀x∈A∃!y(y = t(x)). Luego, por ZF6 (1.2(f)), {y / ∃x∈A(y = t(x))} es conjunto.

8

(c) Probemos (i).

∀y∈Aϕ(y)⇔ ∀y(y ∈ A⇒ ϕ(y))

⇔ ∀y((∃x∈A(y = t(x)))⇒ ϕ(y)) pues A = {t(x) / x ∈ A}

⇔ ∀y((∃x(x ∈ A ∧ y = t(x)))⇒ ϕ(y))

⇔ ∀y∀x((x ∈ A ∧ y = t(x))⇒ ϕ(y)) Calculo de Predicados

⇔ ∀x∀y(x ∈ A⇒ (y = t(x)⇒ ϕ(y)))

⇔ ∀x(x ∈ A⇒ ∀y(y = t(x)⇒ ϕ(y))) Calculo de Predicados

⇔ ∀x(x ∈ A⇒ ϕ(t(x))) Calculo de Predicados

⇔ ∀x∈Aϕ(t(x))

(d) Con A ≡ {t(x) / x ∈ A} obtenemos del inciso anterior que⋃A = {z / ∃y∈A(z ∈ y)} =

{z / ∃x∈A(z ∈ t(x))}. De la misma forma para⋂A.

1.4 Ejemplo. Dados A y y conjuntos, A × {y} es un conjunto. Esto se sigue directamente de 1.3

ya que, con t(x) ≡ (x, y), se sigue que {(x, y) / x ∈ A} es un conjunto.

1.5 Ejemplo. Dados A y B conjuntos, A × B = {(x, y) / x ∈ A ∧ y ∈ B} es un conjunto. En

efecto, con t(y) ≡ A × {y} (el cual es conjunto por el ejemplo anterior) obtenemos de 1.3 que

A = {A× {y} / y ∈ B} es un conjunto. Luego, A × B =⋃A =

⋃y∈B A × {y}, el cual es

conjunto.1

1.6 Ejemplo (ZF−−Inf ). De 1.3 se sigue directamente que {P(x) / x ∈ A} es un conjunto.

1.1 Ejercicio. (a) Enuncie el axioma ZF7 independiente de los axiomas ZF1, ZF3, ZF4 y ZF5.

(b) Pruebe que la afirmacion dada en 1.2(f) implica ZF3.

(c) Sea ϕ(x, y) una formula cuyas variables libres estan entre x, y,A,w1, . . . , wn. Bajo los axiomas ZF1,ZF3, ZF4 y ZF5, pruebe que ZF6 para ϕ(x, y) equivale a la siguiente afirmacion

∀A,w1...,wn [(∀x∈A∃!yϕ(x, y))⇒ ∃f (f funcion ∧ domf = A ∧ ∀x∈Aϕ(x, f(x)))]

1.2 Ejercicio. Pruebe 1.3(c)(ii).

1Notese que en esta prueba precindimos del Axioma de Partes ZF8 para probar que el producto cartesiano de dosconjuntos es conjunto. Esto no puede hacerse, por ejemplo, para probar que AB ≡ {f / f : A→ B} es un conjunto,pues aquı es indispensable el Axioma de Partes.

9

1.2. Conjuntos Bien Ordenados

2.1 Definicion. Sea 〈A,≤〉 un C.P.O.

(1) Dado x ∈ A llamamos Sx(A,≤) ≡ {y ∈ A / y < x} el segmento inicial de x respecto a

〈A,≤〉. Cuando no hay lugar a confusion, escribimos Sx en vez de Sx(A,≤).

(2) B ⊆ A es una seccion de A si y solo si ∀x∈A∀y∈B(x < y⇒ x ∈ B).

(3) Sean 〈A,≤A〉 y 〈B,≤B〉 dos C.P.O. y f : A→ B.

(i) f es creciente sii ∀x,y∈A(x ≤A y⇒ f(x) ≤B f(y)).

(ii) f es estrictamente creciente sii ∀x,y∈A(x <A y⇒ f(x) <B f(y)).

(iii) f es un isomorfismo sii f es biyectiva y ∀x,y∈A(x ≤A y⇔ f(x) ≤B f(y)).

2.2 Lema. Sea 〈A,≤〉 un C.B.O. y f : A→ A una funcion estrictamente creciente. Entonces

(a) ∀x∈A(x ≤ f(x)).

(b) Si ademas f es biyectiva (es decir, un isomorfismo), entonces f = IA (el isomorfismo identi-

dad).

Demostracion. (a) Si razonamos por el absurdo, el conjunto B = {x ∈ A / x > f(x)} es no

vacıo, lo cual nos permite definir m = mınB. Claramente, f(m) < m, lo cual implica que

f(f(m)) < f(m). Ası, f(m) ∈ B y es menor que mınB, lo cual es una contradiccion.

(b) Al aplicar (a) a f−1 obtenemos que x ≤ f−1(x) para todo x ∈ A, es decir, f(x) ≤ x. Por lo

tanto, f(x) = x para todo x ∈ A.

2.3 Lema. Ningun C.B.O. es isomorfo a un segmento inicial de sı mismo.

Demostracion. Supongamos por el contrario que existe un x ∈ A y un isomorfismo f : A → Sx.

Como Sx ⊆ A entonces f : A → A es estrictamente creciente, lo cual implica, por 2.2, que

x ≤ f(x). Ası, f(x) /∈ Sx, lo cual es absurdo.

2.4 Lema. Si 〈A,≤A〉 y 〈B,≤B〉 son C.B.O. isomorfos, entonces el isomorfismo entre ellos es

unico.

10

Demostracion. Sean f : A → B y g : A → B isomorfismos. Luego, g−1 ◦ f : A → A es un

isomorfismo y, de 2.2, se sigue que g−1 ◦ f = IA. Por lo tanto, f = g.

1.3 Ejercicio. Sean 〈A,≤A〉 y 〈B,≤B〉 dos C.P.O. y sea f : A→ B un isomorfismo. Pruebe que

(a) Si x ∈ A y y ∈ Sx(A,≤A), entonces Sy(Sx(A,≤A),≤A) = Sy(A,≤A).

(b) Si x ∈ A, entonces f�Sx : Sx → Sf(x) es un isomorfismo.

(c) Si x ∈ A, entonces existe un y ∈ B tal que Sx(A,≤A) ∼= Sy(B,≤B).

(d) Si C ⊆ A es seccion de A, entonces f [C] es seccion de B.

1.4 Ejercicio. Sea 〈A,≤A〉 un C.B.O. y C ⊆ A. Pruebe que C es seccion de A si y solo si C = A o C es unsegmento inicial de A.

El siguiente Teorema permite dar una nocion de “orden” (bajo isomorfismo) entre conjuntos

bien ordenados. Dados dos C.B.O. es posible determinar cuando uno de ellos es “mas extenso”que

el otro (el mas pequeno es isomorfo a un segmento inicial del otro) o cuando son “igual de extensos”

(es decir, isomorfos). Este hecho toma una forma mas precisa en la siguiente seccio gracias al

concepto de ordinal.

2.5 Teorema. Sean 〈A,≤A〉 y 〈B,≤B〉 dos C.B.O. Entonces se cumple una y solo una de las

siguientes afirmaciones

(i) A ∼= B. (ii) ∃x∈A(Sx(A,≤A) ∼= B). (iii) ∃y∈B(A ∼= Sy(B,≤B)).

Demostracion. Consideremos la relacion g = {(x, y) ∈ A×B / Sx ∼= Sy}. Sea C = domg y

D = rang.

(a) g es funcion. Sean (x, y1), (x, y2) ∈ g, es decir, Sx ∼= Sy1 y Sx ∼= Sy2 . Luego, Sy1 ∼= Sy2 . Si

y1 <B y2 entonces Sy1 es un segmento inicial de Sy2 (por el Ejercicio 1.3) isomorfo a Sy2 , lo

cual contradice el Lema 2.3. Del mismo modo, no sucede que y2 <B y1. Por lo tanto, y1 = y2.

(b) g : C → D es biyectiva. Como en el argumento anterior, se obtiene que g−1 es funcion.

(c) g es estrictamente creciente. Supongamos x, z ∈ C tal que x <A z. Sea y = g(x) yw = g(z),

es decir, Sx ∼= Sy y Sz ∼= Sw. Supongamos ademas quew ≤B y. Como g es biyectiva, entonces

w 6= y, por lo cual w <B y. El siguiente argumento esta justificado por el Ejercicio 1.3: como

Sx es segmento inicial de Sz y Sz ∼= Sw, entonces existe y′ ∈ Sw tal que Sx ∼= Sy′ . Ası,

y′ < w < y, lo cual indica que Sx es isomorfo a un segmento inicial de Sy y, como Sx ∼= Sy,

concluımos que Sy es isomorfo a un segmento inicial de sı mismo, lo cual contradice 2.3. Por

lo tanto, y <B w.

11

(d) C es seccion de A. Sean x ∈ A y z ∈ C tal que x <A z. Como z ∈ C entonces Sz ∼= Sg(z).

Luego, del Ejercicio 1.3, como Sx es segmento inicial de Sz , entonces existe y ∈ Sg(z) tal que

Sx ∼= Sy. Ası, x ∈ C.

(e) D es seccion de B. Prueba analoga a la anterior.

Todo lo anterior indica que C ∼= D y, del Ejercicio 1.4, C es A o un segmento inicial de A, y D es

B o un segmento inicial de B. Por lo tanto, se generan las siguientes cuatro posibilidades:

(i’) A = C y B = D.

(ii’) ∃x∈A(C = Sx) y D = B.

(iii’) ∃y∈B(D = Sy) y C = A.

(iv’) ∃x∈A(C = Sx) y ∃y∈B(D = Sy).

La opcion (iv’) no se cumple. En efecto, si x ∈ A y y ∈ B son tales que C = Sx y D = Sy,

entonces Sx ∼= Sy. lo cual implica x ∈ C = Sx, una contradiccion.

De las opciones (i’), (ii’) o (iii’) se sigue directamente que se cumple una entre (i), (ii) o (iii) en

el enunciado del Teorema. Veamos ahora que solo una opcion es posible. Del Lema 2.3 se sigue

directamente que (i) y (ii) no se dan simultaneamente, lo mismo que (i) y (iii). Veamos ahora que

(ii) y (iii) no se cumplen a la vez. Si existen x ∈ A y y ∈ B tal que Sx ∼= B y A ∼= Sy, como x ∈ Adel Ejercicio 1.3 se sigue que existe y′ ∈ Sy tal que Sx ∼= Sy′ . Como B ∼= Sx obtenemos que B es

isomorfo a Sy′ , un segmento inicial de sı mismo, lo cual contradice 2.3.

1.5 Ejercicio. Sean 〈A,≤A〉 y 〈B,≤B〉 dos C.P.O. tal que A ∩ B = ∅. Definamos la relacion ≤A∪B enA ∪B como

x ≤A∪B y⇔ (x, y ∈ A ∧ x ≤A y) ∨ (x, y ∈ B ∧ x ≤B y) ∨ (x ∈ A ∧ y ∈ B) x, y ∈ A ∪B.

Pruebe que

(a) 〈A ∪B,≤A∪B〉 es un C.P.O. El menor estricto en este C.P.O. esta dado por

x <A∪B y⇔ (x, y ∈ A ∧ x <A y) ∨ (x, y ∈ B ∧ x <B y) ∨ (x ∈ A ∧ y ∈ B) x, y ∈ A ∪B.

(b) El C.P.O. 〈A ∪B,≤A∪B〉 extiende a 〈A,≤A〉 y a 〈B,≤B〉, es decir, ≤A⊆≤A∪B y ≤B⊆≤A∪B .

(c) Si 〈A,≤A〉 y 〈B,≤B〉 son C.T.O. entonces 〈A∪B,≤A∪B〉 es un C.T.O. ¿Que se puede decir del recıpro-co?

(d) Si 〈A,≤A〉 y 〈B,≤B〉 son C.B.O. entonces 〈A ∪ B,≤A∪B〉 es un C.B.O. ¿Que se puede decir delrecıproco?

(e) A es seccion de A ∪B.

(f) Si 〈A,≤A〉 ∼= 〈A′,≤A′〉, 〈B,≤B〉 ∼= 〈B′,≤B′〉 y A′ ∩ B′ = ∅, entonces 〈A ∪ B,≤A∪B〉 ∼= 〈A′ ∪B′,≤A′∪B′〉.

12

1.6 Ejercicio. Sean 〈A,≤A〉 y 〈B,≤B〉 dos C.P.O. Definamos la relacion ≤A×B en A×B como

(x, y) ≤A×B (z, w)⇔ x <A z ∨ (x = z ∧ y ≤B w) (x, y), (z, w) ∈ A×B.

Esta relacion recibe el nombre de Orden Lexicografico de A×B. Pruebe que

(a) 〈A×B,≤A×B〉 es un C.P.O. El menor estricto en este C.P.O. esta dado por

(x, y) <A×B (z, w)⇔ x <A z ∨ (x = z ∧ y <B w) (x, y), (z, w) ∈ A×B.

(b) Si 〈A,≤A〉 y 〈B,≤B〉 son C.T.O. entonces 〈A × B,≤A×B〉 es un C.T.O. ¿Que se puede decir delrecıproco?

(c) Si 〈A,≤A〉 y 〈B,≤B〉 son C.B.O. entonces 〈A × B,≤A×B〉 es un C.B.O. ¿Que se puede decir delrecıproco?

(d) Si 〈A,≤A〉 ∼= 〈A′,≤A′〉 y 〈B,≤B〉 ∼= 〈B′,≤B′〉 entonces 〈A×B,≤A×B〉 ∼= 〈A′ ×B′,≤A′×B′〉.

1.7 Ejercicio. Sean 〈A,≤A〉 y 〈B,≤B〉 dos C.B.O.

(a) (ZF−−P). Si A ∩B = ∅, ¿Es cierto que 〈A ∪B,≤A∪B〉 ∼= 〈B ∪A,≤B∪A〉.?

(b) (ZF−−P). ¿Es cierto que 〈A×B,≤A×B〉 ∼= 〈B ×A,≤B×A〉.?

(c) Sea p /∈ A. Es claro que 〈{p}. ≤{p}〉 es un C.B.O. con ≤{p}= {(p, p)}. Pruebe que 〈A ∪ {p},≤A∪{p}〉es un C.B.O. cuyo maximo es p. ¿Que se puede decir de p respecto a 〈{p} ∪A,≤{p}∪A〉?

(d) Dado z pruebe que A ∼= A× {z} ∼= {z} ×A.

(e) Pruebe que 2×A ∼= (A× {0}) ∪ (A× {1}) (donde 0 = ∅, 1 = {0} y 2 = {0, 1}).

1.8 Ejercicio. Sea 〈A,≤A〉 un C.P.O. cuyos elementos son disjuntos dos a dos y ∀A∈A(〈A,≤A〉 C.P.O.)Sea ≤⋃

A una relacion en⋃A definida por

x ≤⋃A y⇔ ∃A∈A(x, y ∈ A ∧ x ≤A y) ∨ ∃A,B∈A(A <A B ∧ x ∈ A ∧ y ∈ B) x, y ∈

⋃A.

Pruebe que

(a) 〈⋃A,≤⋃

A〉 es un C.P.O. El menor estricto en este C.P.O. esta dado por

x <⋃A y⇔ ∃A∈A(x, y ∈ A ∧ x <A y) ∨ ∃A,B∈A(A <A B ∧ x ∈ A ∧ y ∈ B) x, y ∈

⋃A.

(b) El C.P.O. 〈⋃A,≤⋃

A〉 extiende a cada C.P.O. 〈A,≤A〉 con A ∈ A.

(c) Si 〈A,≤A〉 es un C.T.O. y ∀A∈A(〈A,≤A〉 C.T.O.) entonces 〈⋃A,≤⋃

A〉 es un C.T.O.

(d) Si 〈A,≤A〉 es un C.B.O. y ∀A∈A(〈A,≤A〉 C.B.O.) entonces 〈⋃A,≤⋃

A〉 es un C.B.O. Sugerencia: Alconsiderar C ⊆

⋃A no vacıo, tomeM = mınA {A ∈ A / C ∩A 6= ∅} ym = mınM (C∩M). Pruebe

que m = mın⋃A(C).

(e) Pruebe que los recıprocos de (c) y (d) son ciertos mientras ∅ /∈ A.

1.9 Ejercicio. Sean 〈A,≤A〉 y 〈B,≤B〉 dos C.P.O. con B 6= ∅ y sea A = {{x} ×B / x ∈ A}. Defina larelacion ≤A en A como {x} ×B ≤A {z} ×B ⇔ x ≤B z para x, z ∈ A. Pruebe que 〈A,≤A〉 es un C.P.O.y, segun el ejercicio anterior, pruebe que 〈

⋃A,≤⋃

A〉 = 〈A×B,≤A×B〉.

1.10 Ejercicio. Sea 〈A,≤〉 un C.B.O. Si C ⊆ A entonces A ∼= C o ∃x∈A(C ∼= Sx), pero no se dan ambas ala vez (cabe advertir aquı que C hereda el orden de A).

13

2.6 Ejemplo ((ZF−−P)). Denotemos por ω al conjunto de los numeros naturales. Se conoce que

ω esta bien ordenado por ∈ y que ω /∈ ω. Luego, el buen orden definido para S(ω) = ω ∪ {ω}esta dado, segun el Ejercicio 1.5, por

x <ω∪{ω} y⇔ (x, y ∈ ω ∧ x ∈ y) ∨ (x = y = ω ∧ x <{ω} y) ∨ (x ∈ ω ∧ y = ω)⇔ x ∈ y

para x, y ∈ S(ω). Por lo tanto, 〈S(ω),∈〉 es un C.B.O. que extiende a ω agregando al conjunto ω

como maximo. Tambien se tiene que Sω(S(ω),∈) = ω, lo cual implica que ω es (isomorfo a) un

segmento inicial de S(ω). Por lo tanto, 〈S(ω),∈〉 � 〈ω,∈〉 por el Teorema 2.5. Es usual denotar

ω + 1 = S(ω) (lo cual justificaremos en la seccion 1.5).

2.7 Ejemplo ((ZF−−P)). Sea B = {ω} ∪ ω cuyo buen orden esta definido, segun el Ejercicio 1.5,

por x <B y⇔ (x, y ∈ ω ∧ x ∈ y) ∨ (x = ω ∧ y ∈ ω) para x, y ∈ B. Esto le da a B un orden de la

forma

ω <B 0 <B 1 <B 2 <B . . . <B n <B n+ 1 <B . . .

Definamos la funcion f : B → ω dada por

f(x) ={

0, x = ωx+ 1, x ∈ ω

f es biyectiva y estrictamente creciente. En efecto, si x, y ∈ B y x <B y entonces x, y ∈ ω∧x ∈ y,

o x = ω ∧ y ∈ ω. En el primer caso, f(x) = x + 1 ∈ y + 1 = f(y) y, en el segundo, f(x) = 0 ∈y + 1 = f(y). Por lo tanto, f es un isomorfismo. Luego, 〈{ω} ∪ ω,≤{ω}∪ω〉 ∼= 〈ω,∈〉.

2.8 Ejemplo ((ZF−−P)). Consideremos el buen orden 2×ω definido por el Ejercicio 1.7 y veamos

que ω ∼= S(1,0)(2×ω,≤2×ω). Notese que (x, y) <2×ω (z, w)⇔ (x = 0∧z = 1)∨(x = z∧y ∈ w)

para (x, y), (z, w) ∈ 2 × ω. Esto indica que S(1,0),≤2×ω = {0} × ω. En efecto, si (x, y) ∈ 2 × ωentonces

(x, y) < (1, 0)⇔ (x = 0 ∧ 1 = 1) ∨ (x = 1 ∧ y ∈ 0)⇔ x = 0⇔ x = 0 ∧ y ∈ ω.

Ahora, sea f : ω → S(1,0)(2 × ω,≤2×ω) definida por f(0, x) = x para cada x ∈ ω. f es un

isomorfismo, pues es biyectiva y (0, x) <2×ω (0, y) ⇐⇒ x ∈ y para todo x, y ∈ ω. Por lo tanto,

ω es isomorfo a un segmento inicial de 2× ω, lo cual implica que no son isomorfos.

2.9 Ejemplo ((ZF−−P)). Consideremos el buen orden 〈ω×2,≤ω×2〉 definido como en el Ejercicio

1.6 y consideremos f : ω × 2 → ω definido por f(n, d) = 2n + d para cada (n, d) ∈ ω × 2. f es

un isomorfismo, pues es biyectiva y estrictamente creciente. En efecto, si (n, d), (m, e) ∈ ω × 2 y

14

(n, d) <ω×2 (m, e), entonces n ∈ m o n = m ∧ d = 0 ∧ e = 1. Del primer caso obtenemos que

f(n, d) ≤ 2n + 1 < 2m ≤ f(m, e) y, del segundo, f(n, d) = 2n < 2m + 1 = f(m, e). Por lo

tanto, 〈ω × 2,≤ω×2〉 ∼= 〈ω,∈〉.

1.11 Ejercicio ((ZF−−P)). Pruebe que ω + 1 es isomorfo a un segmento inicial de 2× ω.

Segun lo anterior, ω, ω+ 1 y 2×ω configuran diferentes tipos de buenos ordenes (no isomorfos

entre sı). Vimos que los ordenes de {ω}∪ω y de ω×2 son del mismo tipo de ω, y estan los ordenes

de tipo finito, que varıan segun el cardinal del conjunto. En la proxima seccion veremos una forma

precisa de clasificar los tipos de buenos ordenes, segun sus propiedades de extension, mediante los

numeros ordinales. Estos objetos estan bien ordenados y, en consecuencias, llevan a clasificar los

tipos de orden en una lista bien ordenada.

1.3. Estructura de los Numeros Ordinales

La siguiente definicion pretende extender el concepto de “numero natural”, como conjunto bien

ordenado por ∈, de una forma transfinita que permita clasificar cualquier conjunto bien ordenado.

3.1 Definicion (Ordinal). Sea x un conjunto.

(i) x es transitivo sii ∀y∈x(y ⊆ x).

(ii) x es un ordinal sii es transitivo y esta bien ordenado por ∈ (como menor estricto).

(iii) 0 = ∅, 1 = S(0), 2 = S(1), etc. Estos son los numeros naturales, lo cuales son ordinales. 2

Bajo el Axioma del Infinito se tiene que ω y S(ω) son ordinales. En terminos estrictos, si x

es un ordinal, su buen orden esta dado por la relacion ∈x= {(a, b) ∈ x× x / a ∈ b}, pero como

esta denota el pertenece restringido a los elementos de x, simplemente decimos que x esta bien

ordenado por ∈, escribiendo 〈x,∈〉 en vez de 〈x,∈x〉.

3.2 Lema. Fijemos x, y y z ordinales. Entonces:

(a) Si u ∈ x entonces u es un ordinal. Ademas, u = Su(x,∈).

(b) x /∈ x.

2En la subseccion 1.5.2 veremos que no es necesario el Axioma del Infinito para definir los numeros naturales.

15

(c) Si x ∈ y y y ∈ z entonces x ∈ z.

(d) Si 〈x,∈〉 ∼= 〈y,∈〉 entonces x = y.

(e) Se cumple solo una de las siguientes condiciones: x ∈ y o x = y o y ∈ x.

(f) Todo conjunto no vacıo de ordinales tiene mınimo respecto a la relacion ∈.

(g) Todo conjunto de ordinales esta bien ordinado por ∈.

Demostracion. (a) Veamos que u es transitivo. Si a ∈ b y b ∈ u, como u ∈ x entonces b ∈ x y,

luego, a ∈ x por transitividad de x. Ahora, como a, b, u ∈ x, x esta bien ordenado por ∈, a ∈ by b ∈ u, se sigue que a ∈ u. Por otra parte, como x es transitivo, entonces u ⊆ x y, como x

esta bien ordenado por ∈, u hereda a ∈ como buen orden.

(b) Supongamos x ∈ x y llamemos y = x. Como y ∈ x y x esta bien ordenado por ∈ como menor

estricto, entonces y /∈ y, es decir, x /∈ x, lo cual es una contradiccion.

(c) Inmediato de que z es transitivo.

(d) Si x ∼= y existe un isomorfismo f : x→ y, es decir, f es biyectiva y a ∈ b⇔ f(a) ∈ f(b) para

todo a, b ∈ x. Para probar que x = y basta ver que f es la funcion identidad. Supongamos que

el conjunto A = {a ∈ x / a 6= f(a)} es no vacıo, por lo cual podemos definir m = mın(A)

(respecto al orden de x). Luego, si a ∈ m entonces, como m ∈ x, a ∈ x y luego a /∈ A ya

que es menor que su mınimo. Por lo tanto, a = f(a) ∈ f(m) para todo a ∈ m, lo cual indica

que m ⊆ f(m). Por otra parte, si d ∈ f(m), como f(m) ∈ y entonces d ∈ y. Luego, como

f es isomorfismo, d, f(m) ∈ y y d ∈ f(m), entonces f−1(d) ∈ f−1(f(m)) = m, por lo cual

d = f(f−1(d)) = f−1(d). Ası, d ∈ m para todo d ∈ f(m). Lo anterior permite concluir que

f(m) = m, lo cual contradice que m ∈ A.

(e) De 2.5 se cumple solo una de las siguientes condiciones: x ∼= y o ∃a∈x(Sa ∼= y) o ∃b∈y(Sb ∼= y).

La primera condicion equivale a x = y, la segunda equivale, por (a), a que ∃a∈x(a ∼= y), lo cual

equivale a ∃a∈x(a = y) por el inciso anterior, es decir, y ∈ x y, de la misma forma, la tercera

condicion equivale a x ∈ y.

(f) Sea C 6= ∅ un conjunto de ordinales. Los incisos anteriores indican que C esta linealmente

ordenado por ∈, por lo cual la existencia de un mınimo de C se reduce a ∃m∈C∀a∈C(a /∈ m),

es decir, que ∃m∈C(m∩C = ∅). Como C 6= ∅ entonces existe a ∈ C. Si a∩C = ∅ esta listo

16

el resultado. En el caso contrario, a ∩ C ⊆ a es no vacıo y, como a es ordinal, entonces existe

m = mına(a ∩ C) (mınimo respecto a a). Luego, ∀v∈a∩C(v /∈ m), es decir, a ∩ C ∩m = ∅.

Como m ∈ a entonces m ⊆ a, lo cual da lugar a que C ∩m = ∅.

(g) Se sigue directamente del resultado anterior.

El Lema anterior indica que los ordinales son una estructura bien ordenada por in pero, para

efectos de un C.B.O., se debe tener que {x / x ordinal} es un conjunto. Esto no se cumple segun

el siguiente resultado (Paradoja de Burali-Forti).

3.3 Teorema. ¬∃Y ∀x(x ordinal⇒ x ∈ Y ).

Demostracion. Supongamos que existe dicho Y , lo cual implica que ON = {x / x ordinal} es un

conjunto y, por el Lema 3.2, un C.B.O. por la relacion ∈. ON tambien es transitivo por 3.2(a) y por

lo tanto es un ordinal, es decir, ON ∈ ON, lo cual contradice a 3.2(b).

Por notacion utilizamos de aquı en adelante las letras griegas α, β, γ, ξ, ζ,etc. para denotar ordi-

nales. Con esto muchas veces no diremos, por ejemplo, “sea α un ordinal” sino que se dejara dentro

del contexto. Tambien, como los ordinales estan bien ordenados por ∈ usaremos la notacion

α < β ⇔ α ∈ β α ≤ β ⇔ (α ∈ β ∨ α = β)

para cualquier par de ordinales α y β. Tambien podemos notar del Lema 3.2 que todo ordinal

esta formado por los ordinales menores que el, es decir, α = {ξ ordinal / ξ < α}.

1.12 Ejercicio. Sean x y y ordinales.

(a) Utilize unicamente la definicion de ordinal para probar que x ( y⇒ x ∈ y.

(b) Pruebe que x ∩ y es un ordinal.

(c) Pruebe 3.2(e) sin utilizar el Teorema 2.5 (Esto da una prueba del Lema 3.2 sin utilizar el Teorema 2.5).

1.13 Ejercicio. Dados α y β ordinales, pruebe que

(a) α ⊆ β ⇔ α ≤ β. (b) α ( β ⇔ α < β.

1.14 Ejercicio. Si C es un conjunto no vacıo de ordinales, entonces mın(C) =⋂C. Sugerencia: Utilize el

ejercicio anterior.

3.4 Lema. (a) Todo conjunto transitivo de ordinales es un ordinal.

17

(b) Si C 6= ∅ es un conjunto de ordinales, entonces⋂C es un ordinal y

⋂C = mın(C).

(c) SiX es un conjunto de ordinales entonces⋃X es un ordinal. Ademas,

⋃X = sup(X) (supre-

mo respecto a los ordinales). Esto indica que todo conjunto de ordinales tiene supremo.

(d) Si α es un ordinal entonces S(α) tambien es ordinal.

Demostracion. (a) De 3.2 se sigue que un conjunto de ordinales esta bien ordenado por ∈ y, si

ademas es transitivo, entonces es un ordinal.

(b) Ver el Ejercicio 1.14.

(c) Si y ∈⋃X entonces existe α ∈ X tal que y ∈ α. Como α es un ordinal entonces y tambien.

Esto muestra que⋃X es un conjunto de ordinales, ası que basta ver que es transitivo para

garantizar que es un ordinal. En efecto, si β ∈ γ y γ ∈⋃X entonces existe un α ∈ X tal que

γ ∈ α. Luego, β ∈ α, por lo cual β ∈⋃X .

Por otra parte, si α ∈ X entonces α ⊆⋃X y, por el Ejercicio 1.13, α ≤

⋃X . Esto indica que⋃

X es cota superior de X , ası que falta probar que es la mınima cota superior. En efecto, si

δ es cota superior de α, es decir, ∀α∈X(α ≤ δ), como ≤ equivale a ⊆ segun el Ejercicio 1.13,

entonces⋃X ⊆ δ, es decir,

⋃X ≤ δ.

(d) De 3.2(a) tenemos que S(α) = α∪{α} es un conjunto de ordinales. Basta probar entonces que

es transitivo. Supongamos ξ ∈ β y β ∈ S(α), es decir, ξ < β y, β ∈ α o β = α. Luego, ξ < β

y β ≤ α, lo cual da lugar a que ξ ≤ α, es decir, ξ ∈ S(α).

La estructura de ordinales sirve para caracterizar los C.B.O. de la siguiente forma.

3.5 Teorema. Si 〈A,R〉 es un C.B.O. entonces existe un unico ordinal α tal que 〈A,R〉 ∼= 〈α,∈〉.

Demostracion. Denotemos ≤A= R. La unicidad se sigue directamente de 3.2(d), ası que veamos

la existencia. Si llamamos A = {x ∈ A / ∃β(Sx ∼= β)} tenemos que ∀x∈A∃!β(Sx ∼= β) lo cual nos

permite definir el conjunto α ={β ord. / ∃

x∈A(Sx ∼= β)}

(por Axioma de Reemplazamiento) y la

funcion g : A→ α tal que, para x ∈ A, g(x) es el unico ordinal isomorfo con Sx (notese que hasta

este punto no hemos probado que α es un ordinal). Es claro que α = {β ord. / ∃x∈A(Sx ∼= β)}Veamos los siguientes hechos.

(i) A es seccion de A. Supongamos x, y ∈ A tal que x <A y y y ∈ A. Luego, existe un ordinal

β tal que Sy ∼= β. Del Ejercicio 1.3 existe un ξ ∈ β tal que Sx ∼= Sξ(β,∈) = ξ. Por lo tanto,

x ∈ A.

18

(ii) α es un ordinal. Supongamos ξ ∈ β y β ∈ α. Luego, existe x ∈ A tal que β ∼= Sx y, por el

Ejercicio 1.3, existe z ∈ Sx tal que ξ = Sξ ∼= Sz . Ası, ξ ∈ α. Por lo tanto, α es un conjunto

transitivo de ordinales, es decir, un ordinal.

(iii) g es un isomorfismo. Como g es sobreyectiva, basta probar que es estrictamente creciente.

Sean x, y ∈ A tal que x <A y y sea γ = g(y), es decir, Sy ∼= γ. Por el Ejercicio 1.3, existe

β ∈ γ tal que Sx ∼= Sβ(γ,∈) = β. Por lo tanto, β = g(x) y g(x) < g(y), pues β ∈ γ.

Lo anterior verifica que A ∼= α. La prueba termina si probamos que A = A. Como A es seccion de

A, por el Ejercicio 1.4 se tiene que A = A o que existe b ∈ A tal que A = Sb, pero este ultimo caso

no es posible porque implica que α ∈ α.

3.6 Definicion (Tipo de un C.B.O.). Dado 〈A,R〉 un C.B.O. llamamos type〈A,R〉 (tipo de 〈A,R〉)al unico ordinal isomorfo con 〈A,R〉. Cuando la relacion R es evidente del contexto, simplemente

escribimos type(A).

El Teorema anterior da una mejor perspectiva de lo que representa el Teorema 2.5. Los C.B.O.

isomorfos entre sı tendran un mismo tipo de buen orden, es decir, una longitud igual que se mide

por el ordinal con el que son isomorfos. De este modo, un C.B.O. va a tener menor longitud que otro

sii los ordinales que los miden conservan la misma relacion de desigualdad. El siguiente Ejercicio

verifica esta situacion.

1.15 Ejercicio. Sean 〈A,≤A〉 y 〈B,≤B〉 dos C.B.O. Pruebe que

(a) A ∼= B ⇔ type(A) = type(B).

(b) Si b ∈ B entonces type(Sb) < type(B).

(c) ∃b∈B(A ∼= Sb)⇔ type(A) < type(B).

Esto significa que el Teorema 2.5 es consecuencia directa del Teorema 3.5 y de la tricotomıa en los ordinales.

Hasta el momento hemos discutido los ordinales como una estructura bien ordenada que permite

clasificar (y medir) el tipo de un C.B.O. Ahora vamos a discutir esta estructura desde sus propiedades

internas, partiendo del hecho de que los primeros ordinales son los numeros naturales lo cual se

justifica a continuacion.

3.7 Lema. (a) 0 = ∅ es el ordinal mas pequeno.

(b) ∀α,β(β < S(α)⇔ β ≤ α).

19

(c) ∀α¬∃β(α < β < S(α)), es decir, entre un ordinal y su sucesor no hay ordinales.

(d) α < S(α) y ∀β>α(β ≥ S(α)), es decir, S(α) es el menor ordinal mayor que α.

Demostracion. (a) Es imposible que α ∈ 0 para cualquier ordinal α. Por lo tanto, 0 ≤ α.

(b) Dados α y β ordinales, β ∈ S(α) si y solo si β ∈ α ∨ β = α y, esto es, β ≤ α.

(c) Si β < S(α) entonces β ≤ α por (b), por lo cual no se cumple α < β.

(d) Es claro que α ∈ S(α). Ahora, si β > α del inciso anterior se sigue que S(α) ≤ β.

El resultado anterior garantiza que la primera cadena de ordinales es 0 < S(0) < S(S(0)) < ...

la cual corresponde a los numeros naturales. Con el Axioma del Infinito tenemos por el Lema 3.4

que ω =⋃ω = sup(ω), el supremo de los numeros naturales. Ası, la cadena inicial de ordinales se

extiende a

0 < 1 < 2 < . . . < n < n+ 1 < . . . < ω < S(ω) < S(S(ω)) < . . .

Mas adelante denotaremos por ω+n, con n ∈ ω, al resultado de aplicar n-veces la operacion sucesor

a ω. Luego, podemos definir ω + ω = supn∈ω ω + n y extender la cadena inicial de ordinales a

0 < 1 < 2 < . . . < n < n+ 1 < . . . < ω < ω + 1 < ω + 2 < . . . < ω + n < . . . < ω + ω

En general, estamos construyendo la cadena de ordinales del siguiente modo: partiendo de un or-

dinal α aplicamos sucesor constantemente para generar una cadena de ordinales α < S(α) <

S(S(α)) < . . . y luego se toma el supremo de esta cadena para acotarla, y continua el proceso

tomando sucesores con este supremo. Al ordenar los ordinales de este modo se pueden distinguir

tres tipos de ordinales: el cero, los generados al aplicar sucesor y los generados al tomar el supremo

de una cadena definida. Esta idea de tomar supremo se parece a la idea de tomar el “lımite” de una

sucesiones, por lo cual a este tipo de ordinales se les llama ordinales lımite.

En busca de formalizar la anterior discusion, introduzcamos la siguiente

3.8 Definicion (Ordinal sucesor, ordinal lımite). α se llama ordinal sucesor sii ∃β(α = S(β)).

Cuando α 6= 0 y no es ordinal sucesor, le llamamos ordinal lımite.

3.9 Lema. Las siguientes afirmaciones son equivalentes.

20

(i) γ es un ordinal lımite.

(ii) γ 6= 0 y ∀α<γ(S(α) < γ).

(iii) γ 6= 0 y ∀α<γ∃β(α < β < γ).

(iv) γ 6= 0 y γ =⋃γ = sup(γ).

(v) Existe un conjunto de ordinales X 6= ∅ tal que γ = sup(X) y γ /∈ X .

Demostracion. (i)⇒ (ii) Claramente γ 6= 0. Ahora, si α < γ entonces S(α) ≤ γ por 3.7. La

igualdad no se da ya que γ no es ordinal sucesor.

(ii)⇒ (iii) Inmediato de (ii) con β = S(α).

(iii)⇒ (iv) Como γ es transitivo, entonces⋃γ ⊆ γ. Por otra parte, si α ∈ γ entonces existe un β

tal que α ∈ β ∈ γ. Por lo tanto, α ∈⋃γ.

(iv)⇒ (v) Tome X = γ.

(v)⇒ (i) Sea X un conjunto no vacıo de ordinales tal que γ = sup(X) =⋃X y γ /∈ X . Como

X es no vacıo, existe α0 ∈ X y, como γ /∈ X entonces α0 < γ. Esto indica que γ 6= 0.

Por otra parte, si γ = S(β) para algun ordinal β entonces, como γ = sup(X) /∈ X entonces

∀α∈X(α < γ), es decir, ∀α∈X(α ≤ β) (pues γ = S(β)). Luego, β es una cota superior de X

menor que sup(X), lo cual es un absurdo. Por lo tanto, γ 6= 0 no es ordinal sucesor, es decir,

es un ordinal lımite.

Es claro que el 0 no es un ordinal sucesor ni lımite. Ası, los ordinales estan clasificados en

1. Ordinal cero. Solo el 0 que es el primer ordinal.

2. Ordinales sucesor. Aquellos que son el sucesor de un ordinal.

3. Ordinales lımite. Aquellos que son el supremo (lımite) de una sucesion (o conjunto) de ordinales

donde el no esta incluıdo. En particular, son el supremo (lımite) de los ordinales menores que el.

De aquı podemos inferir que en la cadena

0 < 1 < 2 < . . . < n < n+ 1 < . . . < ω < ω + 1 < ω + 2 < . . . < ω + n < . . . < ω + ω

21

ω y ω + ω son ordinales lımite, mientras que los demas, excepto el cero, son ordinales sucesor.

Notese que ω es entonces el primer ordinal lımite. Con base en este hecho discutiremos en la

subseccion 1.5.2 la relacion entre el Axioma del Infinito y la existencia de ordinales lımite.

1.16 Ejercicio. Dado un ordinal sucesor α, pruebe que S(⋃α) = α. Esto indica que

⋃α es el predecesor

del ordinal sucesor α. Claramente, 0 y los ordinales lımite no tienen predecesor, lo cual se ilustra del hechoque, si γ = 0 o γ es un ordinal lımite, entonces

⋃γ = γ.

1.4. Sobre Clases

Dada cualquier formula ϕ(x) se sabe que en ZF no siempre se da que {x / ϕ(x)} es un conjun-

to, es decir, que ∃Y ∀x(x ∈ Y ⇔ ϕ(x))3. Ası, cuando {x / ϕ(x)} no es conjunto, se piensa que no

vale la pena mencionarlo en Teorıa de Conjuntos. Sin embargo, muchas veces es util enunciar esta

clase de colecciones, independiente de si son conjuntos o no, para precisar la notacion en una de-

mostracion y, por lo tanto, hacerla mas intuitiva. De este modo, a una coleccion del tipo {x / ϕ(x)}le llamamos clase, independiente de que sea o no un conjunto. Cuando una clase no es conjunto, le

llamamos clase propia y, cuando si lo es, obviamente le llamaremos conjunto.

Es usual denotar las clases por letras e interpretar afirmaciones sobre estas a partir de las no-

ciones conocidas sobre conjuntos. Por ejemplo, si denotamos por A = {x / ϕ(x)} y por B =

{x / ψ(x)} donde ψ(x) es una formula, interpretamos que

x ∈ A⇔ ϕ(x)

A ⊆ B⇔ ∀x(x ∈ A⇒ x ∈ B)

⇔ ∀x(ϕ(x)⇒ ψ(x))

A = B⇔ ∀x(x ∈ A⇔ x ∈ B)

⇔ ∀x(ϕ(x)⇔ ψ(x))

A 6= ∅⇔ ∃x(x ∈ A)

⇔ ∃xϕ(x)

A transitivo⇔ ∀x∀y(x ∈ y ∧ y ∈ A⇒ x ∈ A).

Ası, muchas definiciones usuales sobre conjuntos se pueden extender para clases. Para denotar

3Donde Y no es libre en ϕ(x).

22

clases, a diferencia de los conjuntos, utilizaremos letras en negrilla (como en los ejemplos ante-

riores).

4.1 Definicion. (1) V = {x / x = x} es la clase propia que representa el universo.

(2) ON = {x / x ordinal}.

(3) Una clase A es inductiva sii 0 ∈ A ∧ ∀x∈A(S(x) ∈ A).

Es claro de la definicion que V y ON son clases propias transitivas e inductivas.

Ası como las clases propias, es usual definir “relaciones” que no son conjuntos, por ejemplo

V ×V = {(x, y) / x, y ∈ V}

E = {(x, y) ∈ ON×ON / x ∈ y} .

Este tipo de clases tienen la forma {(x, y) / ϕ(x, y)} en donde ϕ(x, y) es una formula. Ası, lla-

mamos clase relacion a una clase del tipo R = {(x, y) / ϕ(x, y)}. Si es claro del contexto que

es clase, simplemente le llamaremos relacion. Cuando se sabe que es clase propia se le puede lla-

mar relacion propia. Ademas, podemos extender las nociones domR = {x / ∃y((x, y) ∈ R)} y

ranR = {y / ∃x((x, y) ∈ R)}. Esta nocion se puede generalizar a clase relacion de n variables.

El concepto de funcion se extiende del mismo modo para clases. Si ψ(x, y) es una formula,

F = {(x, y) / ψ(x, y)} y ademas se cumple que ∀x∈domF∃!yψ(x, y), entonces a F se le llama

clase funcion (o simplemente funcion si el contexto lo permite). Tambien extendemos la nocion

F : A → B a que F es clase funcion, domF = A y ranF ⊆ B. Si x ∈ domF, llamamos F(x) al

unico y tal que (x, y) ∈ F.

Ası como en conjuntos, la “igualdad”4 entre clases relacion y clases funcion se pueden obtener

del mismo modo.

4.2 Lema. (a) Sean R y S dos relaciones. Entonces

(i) R ⊆ S⇔ ∀x,y((x, y) ∈ R⇒ (x, y) ∈ S).

(ii) R = S⇔ ∀x,y((x, y) ∈ R⇔ (x, y) ∈ S).

4Mas que igualdad, es una equivalencia

23

(b) Sean F y G dos clases funcion. Entonces

(i) F ⊆ G⇔(domF ⊆ domG ∧ ∀x∈domF(F(x) = G(x))

).

(ii) F = G⇔(domF = domG ∧ ∀x∈domF(F(x) = G(x))

).

Del Axioma de Reemplazamiento surge la siguiente consecuencia.

4.3 Lema. Sea F una clase funcion. Si domF es conjunto, entonces ranF y F son conjuntos.

La notacion de clases permite aligerar la escritura de enunciados y demostraciones en Teorıa de

Conjuntos. Como primer ejemplo, tenemos que no solo los conjuntos no vacios de ordinales tienen

mınimo, sino tambien las clases compuestas por ordinales.

4.4 Teorema. Sea C ⊆ ON. Si C 6= ∅ entonces existe mın C.

Demostracion. Como C 6= ∅ existe un γ ∈ C. Es claro que γ ∩C es un conjunto de ordinales. Si

este es vacıo, entonces γ = mın C, si no lo es, entonces mın C = mın(γ ∩C).

Por ejemplo, mın ON = 0, mın {α ∈ ON / α sucesor} = 1 y mın {α ∈ ON / α limite} =

ω (esto ultimo en ZF−−P).

Las nociones de orden parcial y buen orden se puede extender para clases. Si A es una clase y

R ⊆ A×A la nocion de que 〈A,R〉 es un orden parcial o orden total se extiende de forma evidente.

Para que sea buen orden, se le adjunta la propiedad de que todo subconjunto no vacio tiene mınimo,

no necesariamente toda subclase no vacıa.

1.17 Ejercicio. Pruebe que si A 6= ∅ entonces⋂

A es un conjunto. Use este hecho para probar en el Teorema4.4 que mın C =

⋂C.

1.18 Ejercicio. Pruebe que C ⊆ ON es clase propia si y solo si C es no acotado.

1.19 Ejercicio. Pruebe que si C ⊆ ON es transitiva, entonces C ∈ ON o C = ON.

1.20 Ejercicio. Sea 〈A,R〉 un buen orden tal que, para todo a ∈ A, la clase Sa = {x ∈ A / x < a} es unconjunto (<= R). Pruebe con esto que toda subclase no vacıa de A tiene mınimo.

1.21 Ejercicio (Induccion transfinita para buenos ordenes). Sea 〈A,R〉 como en el ejercicio anterior. Pruebeque si T ⊆ A cumple que ∀a∈A(Sa ⊆ T⇒ a ∈ T), entonces A = T.

24

1.5. Induccion y Recursion Transfinita

Los principios de Induccion y Recursion conocidos para los numeros naturales se extienden para

los numeros ordinales. La version de Induccion para ordinales se conoce como Induccion Transfini-

ta, cuya forma es similar a la induccion generalizada que se conoce para los numeros naturales. Del

mismo modo, la Recursion Transfinita es una generalizacion del principo de Recursion conocido

para los numeros naturales. En las subsecciones a continuacion exponemos varias formas en las que

se puede mencionar ambos principios, ademas de ofrecer una discusion sobre el Axioma del Infini-

to. Para los fines de esta seccion fijamos C ⊆ ON una clase transitiva, es decir, C es un ordinal

o C = ON segun el Ejercicio 1.19.

1.5.1. Induccion Transfinita

5.1 Teorema (Induccion Transfinita). Si T ⊆ C cumple

∀α∈C(α ⊆ T⇒ α ∈ T)

entonces T = C.

Demostracion. Supongamos que C \T 6= ∅. Por 4.4 esta clase tiene mınimo, el cual llamaremos

α. Como C es transitivo entonces α ⊆ C. Ahora, todos los ordinales menores que α no estan en

C \T pero, como estan en C (pues α ⊆ C) entonces pertenecen a T. Por lo tanto, α ⊆ T y luego,

de la hipotesis, concluımos α ∈ T, lo cual es una contradiccion.

5.2 Corolario. Si T ⊆ C cumple que

(i) 0 ∈ C⇒ 0 ∈ T.

(ii) (Paso Inductivo) S(α) ∈ C ∧ α ∈ T⇒ S(α) ∈ T

(iii) (Paso Lımite) Si γ ∈ C es un ordinal lımite y ∀α<γ(α ∈ T), entonces γ ∈ T.

entonces T = C.

Demostracion. Segun el Teorema anterior, basta probar que, si β ∈ C y β ⊆ T, entonces β ∈ T.

Para β hay tres posibilidades:

β = 0. De (i) se sigue que 0 ∈ T.

β es sucesor. Existe α tal que β = S(α). Como α ∈ β ⊆ T entonces α ∈ T y, por (ii), S(α) ∈ T.

25

β es lımite. Se sigue directamente de (iii) que β ∈ T.

Los siguientes enunciados son las formas usuales para la induccion transfinita. Ambas son con-

secuencias directas de los resultados previos al tomar T = {α ∈ C / ϕ(α)}.

5.3 Corolario. Sea ϕ(x) una formula. Si se cumple

∀α∈C[(∀ξ<αϕ(ξ))⇒ ϕ(α)]

entonces ∀α∈Cϕ(α).

5.4 Corolario. Sea ϕ(x) una formula. Si se cumple

(i) 0 ∈ C⇒ ϕ(0).

(ii) (Paso Inductivo) S(α) ∈ C ∧ ϕ(α)⇒ ϕ(S(α))

(iii) (Paso Lımite) Si γ ∈ C es un ordinal lımite y ∀α<γϕ(α), entonces ϕ(γ).

entonces ∀α∈Cϕ(α).

1.5.2. Sobre los numeros naturales y El Axioma del Infinito

En la clase de numeros ordinales hay una forma de identificar los numeros naturales sin necesi-

dad de recurrir al Axioma del Infinito, lo cual ilustramos en la siguiente definicion.

5.5 Definicion (Numero Natural). α es un numero natural sii α es un ordinal tal que ∀β≤α(β =

0 ∨ β sucesor). Definimos ω = {α / α num. natural}.

Con esta definicion tenemos que los axiomas de Peano se siguen cumpliendo para ω, de modo

que el Axioma del Infinito en Teorıa de Conjuntos no es necesario para demostrar las propiedades

de los naturales, sino solo para expresar que la clase de los numeros naturales es un conjunto.

5.6 Lema. (a) ω es una clase inductiva.

(b) Todo ordinal lımite es inductivo.

(c) ω es una clase transitiva.

(d) (Induccion en ω) Si ϕ(x) es una formula tal que

26

(i) ϕ(0)

(ii) ∀n∈ω(ϕ(n)⇒ ϕ(S(n)))

entonces ∀n∈ωϕ(n).

(e) Si A es una clase inductiva, entonces ω ⊆ A

Lo anterior indica que ω es la menor clase inductiva, lo cual garantiza que satisface las mismas

propiedades que el ω definido por el Axioma del Infinito (bajo dicho Axioma, ambos ω y ω son el

mismo conjunto).

Demostracion. (a) Como ∀β≤0(β = 0) entonces 0 es un numero natural, es decir, 0 ∈ ω. Ahora

supongamos α ∈ ω. Si β ≤ S(α) entonces β < α o β = S(α). De ambos casos es inmediato

que β es cero o es un ordinal sucesor (pues α es un numero natural). Por lo tanto, S(α) ∈ ω.

(b) Si γ es un ordinal lımite entonces 0 ∈ γ ya que γ ≥ 0 y γ 6= 0 (al ser ordinal lımite). Ademas,

∀α<γ(S(α) < γ), lo cual permite concluir que γ es inductivo.

(c) Si β ∈ α y α ∈ ω, entonces todo ordinal menor o igual a β es menor que α y, por lo tanto, son

0 o sucesor.

(d) Del inciso anterior podemos aplicar 5.4 a C = ω. Este describe la induccion bajo tres condi-

ciones, pero la tercera condicion es irrelevante, puesto que ω no contiene ordinales lımite.

(e) El resultado se sigue de inmediato por induccion sobre la formula ϕ(x) : x ∈ A. Claramente,

0 ∈ A. Ahora, si n ∈ ω y n ∈ A, entonces S(n) ∈ A ya que A es inductivo.

Del resultado anterior, obtenemos las siguientes equivalencias para el Axioma del Infinito.

5.7 Teorema. Las siguientes afirmaciones son equivalentes.

(i) Axioma del Infinito. (ii) ω es un conjunto. (iii) Existe un ordinal lımite.

Demostracion. (i)⇒ (ii) Si existe un conjunto inductivo A entonces ω ⊆ A por 5.6. Ası, ω es un

conjunto.

(ii)⇒ (iii) Basta probar que ω = ω es un ordinal lımite. En efecto, ω es un ordinal, ya que es un

conjunto transitivo de ordinales. Ademas, ω 6= 0 y ∀α<ω(S(α) < ω), lo cual implica que ω

es lımite por 3.9.

27

(iii)⇒ (i) Inmediato de 5.6 ya que todo ordinal lımite es un conjunto inductivo.

Tambien podemos introducir las siguiente definicion sin recurrir al Axioma del Infinito.

5.8 Definicion (Conjunto finito). Un conjuntoA es finito sii ∃n∈ω(A ≈ n). De lo contrario, decimos

que A es infinito.

1.22 Ejercicio. Pruebe que ω es clase propia si y solo si ω = ON.

1.23 Ejercicio. Pruebe que todo numero natural es menor que cualquier ordinal lımite.

1.24 Ejercicio. Pruebe que el Axioma de Eleccion implica que el Axioma del Infinito equivale que existe unconjunto infinito (segun la definicion anterior). Sugerencia: Utilize type(A).

1.5.3. Recursion Transfinita

El principio de recursion de los numeros naturales lo extendemos a la Recursion Transfinita

para numeros ordinales. Este principio afirma que se puede construir una sucesion, indizada por

ordinales, donde el α-esimo termino de la sucesion se define a partir de los terminos que lo preceden.

La demostracion de este principio la expodremos de una forma mas general en el Capıtulo 3.

5.9 Teorema (Recursion Transfinita). Sea F : C×V→ V. Entonces existe un unico G : C→ V

tal que G(α) = F(α,G�α) para todo α ∈ C.

Notese que G�α es conjunto por 4.3. De esta forma, F esta bien aplicada para definir G. En

este contexto, G�α representa la sucesion 〈G(ξ)〉ξ<α, es decir, los elementos de la sucesion G que

preceden a α. Ası, el α-esimo termino de la sucesion (es decir, G(α)) se define a partir de F, de

α y de los elementos de la sucesion que lo preceden. Cabe anotar que el primer termino, G(0), se

define como F(0,G�0) = F(0,∅) = F(0, 0). Ası, la recursion esta bien fundamentada, a partir de

un termino inicial G(0) = F(0, 0), y definiendo los siguientes a partir de los que se han definido

previamente.

Una forma simple de este principio se presenta de modo que, al definir una sucesion, se comien-

za con un 0-esimo termino fijo, se define el S(α)-esimo termino a partir del anterior (α-esimo termi-

no) y se define el γ-esimo termino, con γ ordinal lımite, a partir de los terminos que lo preceden.

Esto lo enunciamos en el siguiente resultado.

5.10 Corolario. Supongamos que 0 ∈ C, sea a0 un conjunto y F : C ×V → V. Entonces existe

un unico G : C→ V tal que

28

(i) G(0) = a0.

(ii) Si S(α) ∈ C entonces G(S(α)) = F(α,G(α)).

(iii) Si γ ∈ C es un ordinal lımite, entonces G(γ) = F(γ,G�γ)

Demostracion. Definamos F : C×V→ V de modo que5

F(α, v) =

a0 si (α, v) = (0, 0)F(⋃α, v(

⋃α)) si α es ordinal sucesor, v es funcion y domv = α

F(α, v) si α es ordinal lımite, v es funcion y domv = α0 en otro caso.

De 5.9 existe un unico G : C→ V tal que G(α) = F(α,G�α) para todo α ∈ C. Luego,

(i) cuando α = 0 obtenemos G(0) = F(0, 0) = a0;

(ii) cuando S(α) ∈ C se tiene que

G(S(α)) = F(S(α),G�S(α)) = F(⋃S(α),G(

⋃S(α))) = F(α,G(α));

(iii) cuando γ ∈ C es lımite, se tiene que G(γ)) = F(γ,G�γ) = F(γ,G�γ).

Si G′ : C→ V tambien satisface (i), (ii) y (iii) (con G′ en vez de G) entonces, de la definicion de

F, se sigue que G′(α) = F(α,G′�α) para todo α ∈ C. En efecto,

(1) si α = 0 entonces G′(0) = a0 = F(0, 0) = F(α,G′�α);

(2) si α = S(β) para algun ordinal β, entonces

G′(α) = F(β,G′(β)) = F(α,G′�α);

(3) si α es ordinal lımite, G′(α) = F(α,G′�α) = F(α,G′�α) por definicion de F.

Por lo tanto, de la unicidad de G respecto a F segun 5.9, G = G′.

De ambos resultados obtenemos las formas de Recursion sin parametros, como enunciamos a

continuacion.

5.11 Corolario. Sea F : V → V. Entonces existe un unico G : C→ V tal que G(α) = F(G�α)

para todo α ∈ C.5Recordar que, si α es un ordinal sucesor, entonces

⋃α es el predecesor de α segun el Ejercicio 1.16.

29

Demostracion. Defina F : C×V → V como F(α, v) = F(v). Por 5.9 existe un unico G : C→V tal que G(α) = F(α,G�α) = F(G�α) para todo α ∈ C. Si G′ : C → V es otra funcion que

satisface G′(α) = F(G′�α) para todo α ∈ C, entonces G′(α) = F(α,G′�α), lo cual indica que

G′ satisface la recursion respecto a F y, como G tambien satisface esta recursion, concluımos que

G = G′.

5.12 Corolario. Supongamos que 0 ∈ C, sea a0 un conjunto y F : V → V. Entonces existe un

unico G : C→ V tal que

(i) G(0) = a0.

(ii) Si S(α) ∈ C entonces G(S(α)) = F(G(α)).

(iii) Si γ ∈ C es un ordinal lımite, entonces G(γ) = F(G�γ)

Demostracion. Defina F : C × V → V como F(α, v) = F(v). Al aplicar el Corolario 5.10 a

F obtenemos una unica G : C → V que satisface (i), (ii) y (iii) de este resultado. La unicidad es

directa, como en el resultado anterior.

1.25 Ejercicio. Pruebe la unicidad de G del Teorema 5.9.

1.26 Ejercicio. Sea A una clase y F : C×V→ V.

(a) Suponga que F(α, v) ∈ A para todo α ∈ C y v : α → A. Pruebe que, al aplicar 5.9, se obtieneG : ON→ A.

(b) Suponga que a0 ∈ A y que F(α, v) ∈ A si α ∈ C y v ∈ A, o si α ∈ C es lımite y v : α → A. Pruebeque, al aplicar 5.10, se obtiene G : ON→ A.

1.27 Ejercicio. Enuncie y demuestre resultados similares al ejercicio anterior, pero respecto a los Corolarios5.11 y 5.12.

1.28 Ejercicio. No existe una clase funcion inyectiva de una clase propia en un conjunto.

Como ejemplos de la Recursion Transfinita, veamos las siguientes equivalencias del Axioma de

Eleccion.

5.13 Teorema ((ZF−−Inf )). El Axioma de eleccion (todo conjunto tiene un buen orden) equivale

a que todo conjunto tiene una funcion de eleccion.

30

Demostracion. Es claro que AC implica que todo conjunto A posee una funcion de eleccion, pues

basta definirF : P(A) \ {∅} −→ A

B 7−→ F (B) = mınB

partiendo de que A posee algun buen orden. Ahora, supongamos que F : P(A) \ {∅} → A es

una funcion de eleccion, y veamos que es posible definir un buen orden para A. Dado que A es un

conjunto, A 6= V, por lo cual existe un d /∈ A. Extendamos F a F : P(A) → A ∪ {d} de modo

que F (∅) = d. Definamos F : V→ A ∪ {d} tal que F(x) = F (A \ ranx). Luego, del Teorema de

la Recursion, existe un unico G : ON→ A ∪ {d} tal que, para todo ordinal α,

G(α) = F(G�α) = F (A \ {G(ξ) / ξ < α}).

Probemos los siguientes resultados sobre G.

(1) Si α < β y G(α) = d, entonces G(β) = d. La unica forma de que G(α) = d es que se cumpla

que A ⊆ {G(ξ) / ξ < α}. Ası, como α < β, entonces A ⊆ {G(ξ) / ξ < β}, lo cual implica

que G(β) = d.

(2) Si α < β y G(β) 6= d, entonces G(α) 6= d y G(α) 6= G(β). G(α) 6= d se sigue inmediata-

mente de (1). Como G(β) 6= d entonces A \ {G(ξ) / ξ < β} 6= 0 y G(β) es elemento

de ese conjunto, es decir, G(β) /∈ {G(ξ) / ξ < β}. Por lo tanto, como α < β, entonces

G(α) 6= G(β).

(3) Existe un α tal que G(α) = d. Supongamos por el contrario que G(α) 6= d para todo ordinal

α. De (2) se deduce que G : ON→ A es inyectiva, lo cual contradice el Ejercicio 1.28.

De (3) podemos tomar a κ = mın {α ∈ ON /G(α) = d}. Luego, sea g = G�κ : κ → A (el

codominio esA por (2)). g es biyectiva, pues de (2) se obtiene que g es inyectiva y, como G(κ) = d,

entonces A ⊆ {G(ξ) / ξ < κ} = rang. Mediante esta funcion podemos definir la siguiente relacion

de orden en A:

a ≤A b⇔ g−1(a) ≤ g−1(b) a, b ∈ A.

Se sigue directamente que A esta bien ordenado por dicha relacion.

5.14 Teorema. El Axioma de Eleccion equivale al Lema de Zorn6.

6Todo orden parcial, cuyas cadenas esten acotadas superiormente, tiene un elemento maximal.

31

Demostracion. Sea 〈A,≤A〉 un orden parcial cuyas cadenas estan acotadas superiormente. Como

en la prueba de 5.13 podemos definir F : P(A) \ {∅} → A, independiente de que P(A) sea

conjunto, tal que F(B) ∈ B para todo B ⊆ A no vacıo. Veamos que A tiene un elemento maximal.

Supongamos que A no tiene elementos maximales, es decir ∀x∈A∃y∈A(x <A y). Esto indica que,

para todo x ∈ A, el conjunto Ax = {y ∈ A / x <A y} es no vacıo. Como ∅ es cadena de A

entonces tiene una cota superior a0 ∈ A. Definamos F : ON×V→ A de modo que

F(α, x) =

F({a ∈ A / a cota sup. de ranx}) si x es funcion, domx = α es ordinal lımite

y ranx es una cadena de A,F(Ax(

⋃α)) si α es sucesor y x : α→ A

a0 en otro caso.

Por el Teorema de la Recursion, existe una funcion G : ON→ A tal que G(α) = F(α,G�α) para

todo ordinal α. Esto implica que

G(0) = a0

G(S(α)) = F(AG(α)) α ∈ ONG(γ) = F(γ,G�γ) si γ es ordinal lımite.

Veamos que G es estrictamente creciente, es decir, α < β ⇒G(α) <A G(β). Procedamos por

induccion sobre β.

β = 0. Se cumple trivialmente.

Paso inductivo. Supongamos que α < β ⇒G(α) <A G(β). Si α < S(β), es decir, α ≤ β,

entonces se dan dos casos. Si α = β entonces G(S(β)) = F(AG(β)) ∈ AG(β), por lo cual

G(β) <A G(S(β)). Si α < β entonces G(α) <A G(β) <A G(S(β)).

Paso lımite. Sea γ un ordinal lımite tal que, para todo β < γ, α < β ⇒G(α) <A G(β). Esta

condicion indica que ranG�γ = {G(α) / α < γ} es una cadena en A, por lo cual G(γ) es

una cota superior de este conjunto. Por lo tanto, si α < γ, entonces G(α) <A G(S(α)) ≤AG(γ).

Lo anterior prueba que G : ON → A es una funcion inyectiva, lo cual contradice el Ejercicio

1.28.

1.5.4. Recursion Transfinita Multiple

Mas adelante, para definir la suma y el producto de ordinales por Recursion Transfinita, se debe

aplicar una Recursion por cada ordinal α, es decir, tantas recursiones como numeros ordinales. De

32

este modo, lograr hacer todas estas Recursiones en un solo paso es lo que se denomina Recursion

Multiple, lo cual se ilustra en el siguiente Teorema. En adelante, fijemos una clase P que hara las

veces de “clase indizante” para la cantidad de Recursiones que se quieran ejecutar de forma multiple.

5.15 Teorema. Sea F : P × C ×V → V. Entonces existe una unica G : P × C → V tal que

G(z, α) = F(z, α,G�{z} × α) para todo z ∈ P y α ∈ C.

Demostracion. A modo de notacion, si z ∈ P y v es una funcion, sea fv,z : {z} × domv → V tal

que fv,z(z, x) = v(x) para todo x ∈ domv. Dado z ∈ P sea Fz : C×V→ V tal que, para α ∈ C

y x ∈ V,

Fz(α, x) ={

F(z, α, fz,x) si x es funcion,0 en otro caso.

Veamos primero la unicidad para el enunciado del Teorema. Si G y G′ son dos funciones que

satisfacen el enunciado, para z ∈ P definamos Gz : C → V y G′z : C → V tal que Gz(α) =

G(z, α) y G′z(α) = G′(z, α). Luego, dado α ∈ C se sigue que fGz�α,z : {z} × α → V es tal que

fGz�α,z(z, ξ) = [Gz�α](ξ) = Gz(ξ) = G(z, ξ) para todo ξ ∈ α. Luego, fGz�α,z = G�{z} × α y,

por lo tanto,

Gz(α) = G(z, α) = F(z, α,G�{z} × α) = F(z, α, fGz�α,z) = Fz(α,Gz�α).

Del mismo modo se demuestra que G′z(α) = Fz(α,G′z �α). Ası, Gz y G′z satisfacen la misma

recursion con Fz , lo cual da lugar a que Gz = G′z . Por lo tanto, G(z, α) = Gz(α) = G′z(α) =

G′(z, α) para todo z ∈ P y α ∈ C, lo cual implica que G = G′.

Para la existencia, consideremos primero el caso en que C = C es un ordinal. Luego, existe una

unica funcion Gz : C → V (la cual es conjunto) tal que Gz(α) = Fz(α,Gz�α) para todo α ∈ C.

Luego, sea G : P × C → V definida por G(z, α) = Gz(α) y veamos que esta satisface el

enunciado. En efecto, dado z ∈ P y α ∈ C se tiene que

G(z, α) = Fz(α,Gz�α) = F(z, α, fGz�α,z) = F(z, α,G�{z} × α).

Ahora, veamos el caso en que C = ON. Para todo z ∈ P y γ ∈ ON existe una unica funcion

Gγ,z : γ → V tal que Gγ,z(α) = Fz(α,Gγ,z�α). Si γ1 < γ2 entonces Gγ1,z ⊆ Gγ2,z puesto que,

como ambas satisfacen la misma recursion con Fz , por la unicidad del Teorema de la Recursion se

tiene que Gγ1,z = Gγ2,z �γ1. Por lo tanto, Gz =⋃γ∈ONGγ,z es una funcion bien definida, mas

aun, Gz : ON→ V es tal que Gz(α) = GS(α),z(α). Luego, definimos G : P×ON→ V tal que

G(z, α) = Gz(α). Esta funcion satisface el enunciado del Teorema, pues G(z, α) = GS(α),z(α) =

Fz(α,GS(α),z�α) = Fz(α,Gz�α) = F(z, α, fGz�α,z) = F(z, α,G�{z} × α).

33

Ası como en la subseccion 1.5.3, del Teorema anterior se tienen otras formas de expresar la

Recursion Transfinita Multiple.

5.16 Corolario. Sea F : P × V → V. Entonces existe una unica G : P × C → V tal que

G(z, α) = F(z,G�{z} × α) para todo z ∈ P y α ∈ C.

5.17 Corolario. Sea C 6= ∅, F : P × C × V → V y c : P → V. Entonces existe una unica

funcion G : P×C→ V tal que, para z ∈ P,

(i) G(z, 0) = c(z),

(ii) G(z, S(α)) = F(z, α,G(z, α)) cuando S(α) ∈ C,

(iii) Si γ ∈ C es un ordinal lımite, entonces G(z, γ) = F(z, γ,G�{z} × γ)

Demostracion. Definamos F : P×C×V→ V tal que

F(z, α, x) =

c(z) si (α, x) = (0, 0),F(z,

⋃α, x(z,

⋃α)) si x : {z} × α→ V y α es sucesor,

F(z, α, x�{z} × α) si x : {z} × γ → V y α es lımite.

De 5.15 existe una funcion G : P×C→ V tal que G(z, α) = F(z, α,G�{z} × α). Esta funcion

satisface las condiciones del enunciado.

5.18 Corolario. Supongamos C 6= ∅, F : P ×V → V y c : P → V. Entonces existe una unica

funcion G : P×C→ V tal que, para z ∈ P,

(i) G(z, 0) = c(z),

(ii) G(z, S(α)) = F(z,G(z, α)) cuando S(α) ∈ C,

(iii) Si γ ∈ C es un ordinal lımite, entonces G(z, γ) = F(z,G�{z} × γ)

1.29 Ejercicio. Para cada uno de los cuatro resultados anteriores, enuncie y demuestre un resultado analogoa los ejercicios 1.26 y 1.27.

1.6. Aritmetica de ordinales

En esta seccion definimos las operaciones de suma, producto y exponenciacion entre ordinales,

las cuales extienden las operaciones definidas para numeros naturales. Damos dos versiones distin-

tas de estas operaciones, la primera respecto a tipos de buen orden y la segunda mediante Recursion

Transfinita. Se probara tambien que ambas nociones coinciden.

34

1.6.1. Aritmetica de ordinales desde los tipos de buen orden.

6.1 Definicion. Dados α, β ∈ ON definimos

(i) α + β = type((α × {0}) ∪ (β × {1})). Esto representa el tipo de buen orden que sucede al

adjuntar a α el buen orden β por encima.

(ii) α ·β = type(β×α). Esto representa el tipo de orden que ordenar lexicograficamente a β×α.

1.30 Ejercicio. Sean A, B y C conjuntos bien ordenados.

(a) Si A y B son disjuntos, entonces type(A ∪B) = type(A) + type(B).

(b) type(A×B) = type(B) · type(A).

(c) Si A, B y C son disjuntos dos a dos, pruebe que A ∪ (B ∪ C) es isomorfo a (A ∪B) ∪ C como buenosordenes (de hecho, son iguales, pues sus ordenes seran iguales).

(d) Utilice el inciso anterior para probar que la suma de ordinales es asociativa.

(e) Pruebe que A× (B × C) y (A×B)× C son isomorfos como buenos ordenes.

(f) Utilize el inciso anterior para probar que el producto de ordinales es asociativo.

6.2 Lema. Sean α, β y γ ordinales. Entonces:

(a) Si A, B y C son conjunto bien ordenados y B ∩ C = ∅, entonces (B ∪ C)× A es isomorfo a

(B ×A) ∪ (C ×A).

(b) α · (β + γ) = α · β + α · γ.

(c) Si β ≤ γ entonces α+ β ≤ α+ γ.

(d) Si β ≤ γ entonces α · β ≤ α · γ.

Demostracion. (a) Es claro que (B ∪C)×A = (B×A)∪ (C×A), ası que solo debemos probar

que ≤(B∪C)×A=≤(B×A)∪(C×A). Fijemos (a, b), (c, d) ∈ (B ∪ C)× A, es decir, a, c ∈ B ∪ Cy b, d ∈ A. Luego,

(a, b) ≤(B∪C)×A (c, d)⇔ a <B∪C c ∨ (a = c ∧ b ≤A d)⇔ (a, c ∈ B ∧ a <B c) ∨ (a, c ∈ C ∧ a <C c) ∨ (a ∈ B ∧ c ∈ C) ∨ (a = c ∧ b ≤A d)⇔ (a, c ∈ B ∧ a <B c) ∨ (a, c ∈ C ∧ a <C c) ∨ (a ∈ B ∧ c ∈ C)∨

(a, c ∈ B ∧ a = c ∧ b ≤A d) ∨ (a, c ∈ C ∧ a = c ∧ b ≤A d)⇔ (a, c ∈ B ∧ (a <B c ∨ (a = c ∧ b ≤A d)))∨

(a, c ∈ C ∧ (a <C c ∨ (a = c ∧ b ≤A d))) ∨ (a ∈ B ∧ c ∈ C)⇔ (a, c ∈ B ∧ (a, b) ≤B×A (c, d)) ∨ (a, c ∈ C ∧ (a, b) ≤C×A (c, d)) ∨ (a ∈ B ∧ c ∈ C)⇔ ((a, b), (c, d) ∈ B ×A ∧ (a, c) ≤B×A (c, d))∨

((a, b), (c, d) ∈ C ×A ∧ (a, c) ≤C×A (c, d)) ∨ ((a, b) ∈ B ×A ∧ (c, d) ∈ C ×A)⇔ (a, b) ≤(B×A)∪(C×A) (c, d)

35

La cuarta formula surge de que, como a, c ∈ B ∪ C, entonces a = c⇒ (a, c ∈ B ∨ a, c ∈ C)

y, en la quinta formula, se aplica propiedad distributiva con a, c ∈ B y con a, c ∈ C. Los pares

ordenados de la septima formula surgen porque b, d ∈ A.

(b) Del Ejercicio 1.7, {x} ×A ∼= A× {x} ∼= A para cualquier C.B.O. A y cualquier x. Luego, del

Ejercicio 1.30,

[(β × {0}) ∪ (γ × {1})]× α ∼= [({0} × β) ∪ ({1} × γ)]× α∼= (({0} × β)× α) ∪ (({1} × γ)× α)∼= ({0} × (β × α)) ∪ ({1} × (γ × α))∼= ((β × α)× {0}) ∪ ((γ × α)× {1})

El tipo del primer C.B.O. es α ·(β+γ) y el tipo del ultimo es α ·β+α ·γ. Como son isomorfos,

entonces ambos tipos son iguales.

(c) Como β ≤ γ entonces β ⊆ γ y luego (α×{0})∪ (β×{1}) ⊆ (α×{0})∪ (γ×{1}). Ademas,

el orden del primero esta contenido en el segundo conjunto, por lo cual el primer conjunto es

isomorfo al segundo o a un segmento inicial (ver Ejercicio 1.10). Luego, del Ejercicio 1.15, el

tipo del primer conjunto es menor igual que el del segundo, es decir α+ β ≤ α+ γ.

(d) Como β ≤ γ entonces β ⊆ γ. Luego, β × α ⊆ γ × α y el orden del primero esta contenido en

el segundo . En efecto, si (a, b), (c, d) ∈ β × α entonces

(a, b) ≤γ×α (c, d) ⇔ (a <γ c) ∨ (a = c ∧ b ≤α d)⇔ (a ∈ c) ∨ (a = c ∧ b ≤α d)⇔ (a <β c) ∨ (a = c ∧ b ≤α d)⇔ (a, b) ≤β×α (c, d)

Por el Ejercicio 1.10, β × α es isomorfo a γ × α o a un segmento inicial. Luego, del Ejercicio

1.15, type(β × α) ≤ type(β × α), es decir, α · β ≤ α · γ.

6.3 Teorema. Dados α, β y γ ordinales,

(a) α+ 0 = 0 + α = α.

(b) α+ 1 = S(α).

(c) α+ S(β) = S(α+ β).

(d) Si γ es lımite, α+ γ = supβ<γ(α+ β).

(e) α · 0 = 0 · α = 0.

(f) α · 1 = 1 · α = α.

(g) α · S(β) = α · β + β.

(h) Si γ es lımite, α · γ = supβ<γ(α · β).

36

Demostracion. (a) Inmediato de que (α×{0})∪ (0×{1}) = α×{0} y (0×{0})∪ (α×{1}) =

α× {1}, donde ambos son isomorfos a α.

(b) (α × {0}) ∪ (1 × {1}) = (α × {0}) ∪ ({0} × {1}) ∼= α ∪ {α} = S(α). Al tomar tipos,

α+ 1 = S(α).

(c) De la asociatividad de la suma en el Ejercicio 1.30 y de (b), α + S(β) = α + (β + 1) =

(α+ β) + 1 = S(α+ β).

(d) Del Lema 6.2 se sigue que, dado β < γ, α + β ≤ α + γ. Por lo tanto, α + γ es cota superior

del conjunto S = {α+ β / β < γ}. Veamos ahora que, si τ es cota superior de S, entonces

α + γ ≤ τ . En efecto, α < α + 1 ≤ τ (del Ejercicio 1.23, 1 < γ), por lo cual α ( τ . Luego,

como τ \ α ⊆ τ , este esta bien ordenado, por lo cual podemos definir δ = type(τ \ α). Luego

τ = α ∪ (τ \ α) ∼= (α × {0}) × (δ × {1}), lo cual indica que τ = α + δ. Si δ < γ entonces

δ + 1 < γ y luego α + δ + 1 ∈ S, dando lugar a que τ = α + δ < α + δ + 1 ≤ τ , es decir,

τ < τ , lo cual es imposible. Por lo tanto, γ ≤ δ y, del Lema 6.2, α+ γ ≤ α+ δ = τ .

(e) Claramente α× 0 = 0× α = 0.

(f) Inmediato de que α× {0} ∼= {0} × α ∼= α.

(g) Del Lema 6.2 se tiene que α · S(β) = α · (β + 1) = α · β + α · 1 = α · β + α.

(h) Del Lema 6.2 se sigue que, dado β < γ, α · β ≤ α · γ. Por lo tanto, α · γ es cota superior

del conjunto P = {α · β / β < γ}. Veamos ahora que, si ε es cota superior de P , entonces

α · γ ≤ ε. Supongamos por el contrario que ε < α · γ = type(γ × α). Esto indica que ε es

isomorfo a un segmento inicial de γ × α, es decir, existe (β, ξ) ∈ γ × α tal que ε ∼= S(β,ξ). Es

claro que (β, ξ) <γ×α (β + 1, 0) y

S(β+1,0) = {(x, y) ∈ γ × α / (x ∈ β + 1) ∨ (x = β + 1 ∧ y < 0)}

= {(x, y) ∈ γ × α / x ∈ β + 1} = (β + 1)× α.

Por lo tanto, ε es isomorfo a un segmento inicial de (β + 1) × α, es decir, ε < α · (β + 1)7.

Como β + 1 < γ, lo anterior contradice el hecho que ε es cota superior de P .

7Esto lo podemos afirmar porque, en el Lema 6.2, probamos que el orden de γ ×α contiene al orden de (β + 1)×α.

37

Las propiedades anteriores sobre suma y producto en ordinales indican que estas operaciones

extienden a la suma y producto definidos para los numeros naturales. Recordemos que la suma de

naturales se define por recursion, ası

n+ 0 = n n ∈ ωn+ S(m) = S(n+m) n,m ∈ ω

lo cual corresponde a las propiedades mencionadas en el Teorema anterior para la suma ordinal.

Para el producto sucede lo mismo, pues

n · 0 = 0 n ∈ ωn · S(m) = n ·m+ n n,m ∈ ω

Vimos que la suma y producto ordinales son asociativos y que el producto distribuye respecto a

la suma por la izquierda. Preguntas como si la suma y el producto son conmutativos, o si el pro-

ducto distribuye respecto a la suma por la derecha, no se pueden responder en ZF−−P− Inf . Si

suponemos la negacion del Axioma del Infinito, la respuesta de la pregunta anterior es afirmativa.

Por otra parte, asumiendo el Axioma del Infinito, se puede encontrar un contraejemplo para cada

caso, como veremos en el siguiente ejemplo.

6.4 Ejemplo (ZF−−P). Como ω + 1 = S(ω), es claro que ω < ω + 1. Por otra parte, 1 + ω =

supn<ω(1+n) = ω 6= ω+1, lo cual muestra que la suma de ordinales no es conmutativa. Tenemos

tambien que ω · 2 = ω · (1 + 1) = ω + ω = supn<ω(ω + n). Luego, ω < ω + 1 < ω + 2 ≤ω+ ω = ω · 2. Por otra parte, 2 · ω = supn<ω(2 · n) = ω 6= ω · 2, lo cual indica que el producto no

es conmutativo. Tambien tenemos que el producto no distribuye respecto a la suma por la derecha,

pues (1 + 1) · ω = 2 · ω = ω, y 1 · ω + 1 · ω = ω + ω = ω · 2 6= ω.

1.31 Ejercicio. (a) ((ZF−−P)). Pruebe que n+ ω = ω para todo n ∈ ω.

(b) Pruebe que, dado un ordinal lımite γ, se tiene que n+ γ = γ para cualquier natural n. ¿Se puede afirmarque γ + n = n+ γ? ¿Por que?

(c) ¿Por que la negacion del Axioma del Infinito implican que la suma y producto de ordinales son conmu-tativos, y que el producto distribuye respecto a la suma por derecha?

1.6.2. Funciones normales

En esta subseccion exponemos una serie de resultados sobre funciones de ordinales, las cuales

nos permitiran demostrar propiedades aritmeticas de ordinales en la proxima subseccion. Fijemos

C ⊆ ON transitivo.

38

6.5 Definicion (Funcion continua, Funcion normal). Sea F : C→ ON. Decimos que F es continua

sii, para todo ordinal lımite γ ∈ C, F(γ) = supα<γ(F(α)). Decimos que F es normal cuando es

estrictamente creciente y continua.

6.6 Lema. Si F : C→ ON es estrictamente creciente, entonces ∀α∈C(F(α) ≥ α).

Demostracion. Supongamos que A = {α ∈ C / F(α) < α} es no vacıo, por lo cual existe δ =

mın A. Como F(δ) < δ entonces F(F(δ)) < F(δ), lo cual indica que F(δ) ∈ A y es menor que

mın A, lo cual es una contradiccion.

6.7 Lema. Una funcion F : C→ ON es normal si y solo si cumple

(i) ∀α∈C(F(α) < F(S(α))) y

(ii) F es continua.

Demostracion. Supongamos que F cumple (i) y (ii), ası que resta probar que es estrictamente

creciente. Veamos por induccion sobre β que α < β ⇒ F(α) < F(β).

β = 0. No hay nada que probar.

Paso inductivo. Supongamos α < β ⇒ F(α) < F(β) y α < S(β), es decir, α ≤ β. Si α = β

entonces F(β) < F(S(β)) por (i). Si α < β entonces F(α) < F(β) < F(S(β)).

Paso lımite. Sea γ ∈ C ordinal lımite tal que, para β < γ, α < β ⇒ F(α) < F(β). Si α < γ

entonces existe un ordinal β tal que α < β < γ. Luego, F(α) < F(β) ≤ supβ<γ F(β) =

F(γ).

6.8 Lema. Sea F : C→ ON una funcion normal. Entonces

(a) ∀α,β∈C(α < β ⇔ F(α) < F(β)).

(b) Si γ ∈ C es un ordinal lımite, entonces F(γ) es lımite.

(c) Si A ⊆ C es conjunto no vacıo, entonces F(supA) = supα∈AF(α).

(d) Si A ⊆ C es no vacıa, entonces F(mın A) = mınα∈A F(α).

Demostracion. (a) Se sigue de que F es estrictamente creciente y de la tricotomıa de ordinales.

39

(b) Sea γ ∈ C un ordinal lımite. Claramente, {F(α) / α < γ} es un conjunto no vacıo y F(γ) es

su supremo y no pertenece a el. Por lo tanto, de 3.9, F(γ) es un ordinal lımite.

6.9 Teorema. Sean F : C → D y G : D → ON funciones normales donde D ⊆ ON es

transitivo. Entonces G ◦ F es una funcion normal.

Demostracion. Sea γ ∈ C un ordinal lımite, y veamos que G(F(γ)) = supα<γ G(F(α)). Es claro

que G ◦F es estrictamente creciente, de donde se sigue ≥. Por otra parte, como F(γ) es un ordinal

lımite, entonces G(F(γ)) = supβ<F(γ) G(β). Si β < F(γ) = supα<γ F(α) entonces existe α < γ

tal que β < F(α) y, luego, G(β) < G(F(α)) ≤ supα<γ G(F(α)). Como lo anterior se cumple

para todo β < F(γ), concluımos que G(F(γ)) = supβ<F(γ) G(β) ≤ supα<γ G(F(α)).

6.10 Teorema ((ZF−−P) Punto Fijo). Sea F : ON → ON una funcion normal. Entonces

∀α∃β>α(F(β) = β)).

Demostracion. Por Recursion en ω, definamos la siguiente sucesion de ordinales

β0 = S(α)βS(n) = F(βn) para n ∈ ω.

Por 6.6, βn ≤ F(βn) = βS(n), lo cual implica que la sucesion 〈βn〉n∈ω es creciente. Luego, sea

β = supn∈ω βn. Claramente, β ≥ β0 > α y F(β) = supn∈ω F(βn) = supn∈ω βS(n) = β por el

Lema 6.8.

1.32 Ejercicio. Pruebe los incisos (c) y (d) del Lema 6.8.

1.33 Ejercicio. Pruebe que el Teorema 6.9 sigue siendo valido al quitar la hipotesis de que ranF ⊆ D.Sugerencia: Pruebe que dom(G ◦ F) es transitivo.

1.6.3. Aritmetica de ordinales definida por Recursion.

Para la suma y producto de ordinales tenemos otra definicion dada por Recursion Transfinita, la

cual es equivalente a la vista en la subseccion 1.6.1.

6.11 Teorema (Suma de ordinales). Existe una unica operacion + : ON×ON→ ON tal que

α+ 0 = αα+ S(β) = S(α+ β)α+ γ = supβ<γ{α+ β} si γ es un ordinal lımite

40

Demostracion. Definamos F : ON×V→ ON tal que

F(α, x) =

S(x) si x es ordinal,⋃ξ<γ x(α, ξ) si x : {α} × γ → ON y γ es ordinal lımite,

0 en otro caso.

Aplicamos el Corolario 5.18 a P = C = ON, F y a c : ON → ON tal que c(α) = α, para

obtener una unica funcion s : ON×ON→ ON tal que

s(α, 0) = αs(α, S(β)) = F(α, s(α, β)) = S(s(α, β))s(α, γ) = supβ<γ{s(α, β)} si γ es un ordinal lımite.

Al denotar α+ β = s(α, β), se sigue el resultado.

6.12 Teorema (Producto de ordinales). Existe una unica operacion · : ON×ON→ ON tal que

α · 0 = 0α · S(β) = α · βα · γ = supβ<γ{α · β} si γ es un ordinal lımite

Demostracion. Definamos F : ON×V→ ON tal que

F(α, x) =

x+ α si x es ordinal,⋃ξ<γ x(α, ξ) si x : {α} × γ → ON y γ es ordinal lımite,

0 en otro caso.

Aplicamos el Corolario 5.18 a P = C = ON, F y a c : ON → ON tal que c(α) = 0, para

obtener una unica funcion p : ON×ON→ ON tal que

p(α, 0) = 0p(α, S(β)) = F(α,p(α, β)) = p(α, β) + αp(α, γ) = supβ<γ{p(α, β)} si γ es un ordinal lımite.

Al denotar α · β = p(α, β), se sigue el resultado.

En la subseccion 1.6.1 demostramos en el Teorema 6.3 que la suma y producto definida por

tipos (Definicion 6.1) satisface la misma recursion que la suma y producto definidos en los dos Teo-

remas anteriores. Puesto que solo existe una funcion que satisfaga una recursion, podemos concluir

entonces que ambos conceptos definen la misma suma y el mismo producto de ordinales.

En lo que sigue vamos a probar las principales propiedades aritmeticas de los ordinales. Aunque

algunas propiedades las probamos para la Definicion 6.1, volveremos a demostrarlas de forma in-

dependiente a los resultados establecidos en la subseccion 1.6.1.

6.13 Lema. Sea α un ordinal.

41

(a) 0 + α = 0.

(b) α+ 1 = S(α).

(c) α · 1 = 1 · α = α.

(d) 0 · α = 0.

Demostracion. (a) Procedamos por induccion sobre α. Claramente, 0 + 0 = 0. Para el paso in-

ductivo, si 0 + α = α, entonces 0 + S(α) = S(0 + α) = S(α). Para el paso lımite, si γ es

ordinal lımite y ∀α<γ(0 + α = α), entonces 0 + γ = supα<γ(0 + α) = supα<γ(α) = γ.

(b) α+ 1 = α+ S(0) = S(α+ 0) = S(α).

(c) α · 1 = α ·S(0) = α · 0 +α = 0 +α = α. Veamos ahora por induccion sobre α que 1 ·α = α.

Es claro que 1 · 0 = 0. Para el paso inductivo, si 1 · α = α, entonces 1 · S(α) = 1 · α + 1 =

α + 1 = S(α). Para el paso lımite, si γ es un ordinal lımite y ∀α<γ(1 · α = α), entonces

1 · γ = supα<γ(1 · α) = supα<γ(α) = γ.

(d) Se sigue por induccion sobre α.

6.14 Lema. Sean α, β y γ ordinales.

(a) La funcion +α : ON→ ON, tal que +α(β) = α+ β, es normal.

(b) β ≤ α+ β.

(c) α < β ⇔ γ + α < γ + β.

(d) γ + α = γ + β ⇔ α = β.

(e) α ≤ β ⇔ γ + α ≤ γ + β.

(f) Si γ es lımite, α+ γ es lımite.

Demostracion. Al probar (a), los incisos siguientes son consecuencias inmediatas de que +α es

una funcion normal en virtud de 6.6 y 6.8. Por 6.7 veamos que +α es continua y que +α(β) <

+α(S(β)). De la definicion de suma, es claro que +α es continua. Por otra parte, +α(S(β)) =

α+ S(β) = S(α+ β) > β.

6.15 Lema. Sean α 6= 0, β y γ ordinales.

(a) La funcion ·α : ON→ ON, tal que ·α(β) = α · β, es normal.

(b) β ≤ α · β.

(c) α < β ⇔ γ · α < γ · β.

(d) γ · α = γ · β ⇔ α = β.

(e) α ≤ β ⇔ γ · α ≤ γ · β.

(f) Si γ es lımite, α · γ es lımite.

42

Demostracion. Al probar (a), los incisos siguientes son consecuencias inmediatas de que ·α es una

funcion normal en virtud de 6.6 y 6.8. Por 6.7 veamos que ·α es continua y que ·α(β) < ·α(S(β)).

De la definicion de producto, es claro que ·α es continua. Por otra parte, ·α(S(β)) = α · S(β) =

α · β + α > α · β por 6.14(b).

6.16 Lema. Sean α, β y γ ordinales. Entonces

(a) α+ (β + γ) = (α+ β) + γ.

(b) α · (β + γ) = α · β + α · γ.

(c) α · (β · γ) = (α · β) · γ.

(d) α · β = 0⇔ α = 0 ∨ β = 0.

Demostracion. Los tres primeros incisos se prueban por induccion sobre γ.

(a) Para γ = 0 es claro queα+(β+0) = α+β = (α+β)+0. Para el paso inductivo, siα+(β+γ) =

(α+β) +γ entonces α+ (β+S(γ)) = S(α+ (β+γ)) = S((α+β) +γ) = (α+β) +S(γ).

Para el paso lımite, si γ es lımite tal que ∀ξ<γ(α+ (β + ξ) = (α+ β) + ξ), entonces, debido a

que la composicion de funciones normales es normal (por 6.9), α + (β + γ) = +α(+β(γ)) =

supξ<γ +α(+β(ξ)) = supξ<γ(α+ (β + ξ)) = supξ<γ((α+ β) + ξ) = (α+ β) + γ.

(b) Para γ = 0 es claro que α · (β + 0) = α · β = α · β + α · 0. Para el paso inductivo, si

α · (β + γ) = α · β + α · γ entonces α · (β + S(γ)) = α · S(β + γ) = (α · (β + γ)) + α =

(α · β+α · γ) +α = α · β+ (α · γ+α) = α · β+α ·S(γ). Para el paso lımite, sea γ lımite tal

que ∀ξ<γ(α · (β + ξ) = α · β + α · ξ). Si α = 0 el resultado se sigue. Si α 6= 0 entonces, como

+β y ·α son normales, α · (β+γ) = ·α(+β(γ)) = supξ<γ ·α(+β(γ)) = supξ<γ(α · (β+ ξ)) =

supξ<γ(α · β + α · ξ) = supξ<γ +α·β(·α(ξ)) = +α·β(·α(γ)) = α · β + α · γ.

(c) Para γ = 0 es claro que α·(β ·0) = 0 = (α·β)·0. Para el paso inductivo, si α·(β ·γ) = (α·β)·γentonces α · (β ·S(γ)) = α · (β ·γ+β) = α · (β ·γ)+α ·β = (α ·β) ·γ+α ·β = (α ·β) ·S(γ).

Para el paso lımite, sea γ lımite tal que ∀ξ<γ(α · (β · ξ) = (α · β) · ξ). Si α = 0 o β = 0 el

resultado se sigue. Si ambos son diferentes de cero, entonces, por 6.9, α · (β · γ) = ·α(·β(γ)) =

supξ<γ ·α(·β(ξ)) = supξ<γ(α · (β · ξ)) = supξ<γ((α · β) · ξ) = (α · β) · γ.

(d) Si α · β y α 6= 0, entonces α · β = α · 0 y esto implica que β = 0 por 6.15(d).

Ası como en la aritmetica en los numeros naturales, en ordinales podemos hablar de resta y de

algoritmo de la division, segun el siguiente resultado.

43

6.17 Teorema. Sean α y β ordinales.

(a) α ≤ β ⇒ ∃!δ(α+ δ = β).

(b) Si β 6= 0 existen unicos δ, ρ tal que α = β · δ + ρ y ρ < δ.

Demostracion. Ambas pruebas pueden efectuarse directamente o por induccion.

(a) La unicidad es inmediata, pues si α ≤ β, α + δ1 = β = α + δ2, entonces δ1 = δ2. Tambien la

existencia es inmediata cuando α = β (aquı δ = 0). Veamos entonces que α < β ⇒ ∃δ(α+δ =

β) por induccion sobre β. Si β = 0 se sigue trivialmente. Para el paso inductivo, si α < S(β)

entonces α ≤ β. Por la hipotesis inductiva, existe δ tal que α + δ = β. Luego, α + S(δ) = β.

Para el paso lımite, sea γ lımite tal que ∀β < γ(α < β ⇒ ∃δ(α + δ = β)). Sea α < γ y

definamos, para cada α < β < γ, a δβ como el unico ordinal (por la unicidad del δ en este

Teorema) tal que α + δβ = β. Sea δ = supα<β<γ δβ . Luego, por 6.8, α + δ = +α(δ) =

supα<β<γ(α+ δβ) = supα<β<γ(β) = γ.

(b) ConsideremosQ = {ξ ∈ ON / β · ξ ≤ α}. Este es un conjunto, pues si ξ ∈ Q entonces β ·ξ ≤α ≤ β · α, lo cual impica que ξ ≤ α y, por lo tanto, Q ⊆ S(α). Es claro que Q 6= ∅, pues

0 ∈ Q, y Q es transitivo, pues si ξ1 < ξ2 y ξ2 ∈ Q entonces β · ξ1 < β · ξ2 ≤ α. Ahora,

sea δ = sup(Q) =⋃

(Q) ⊆ Q. Si δ /∈ Q entonces δ es un ordinal lımite por 3.9 y, ademas,

α < β ·δ = supξ<δ β ·ξ, por lo cual existe ξ < δ tal que α < β ·ξ, lo cual contradice que ξ ∈ Q(esto ultimo se da porque δ ⊆ Q). Por lo tanto, δ ∈ Q, lo cual da lugar a que δ = max(Q). Por

(a), existe un ρ tal que α = β · δ + ρ. Como δ + 1 > δ = max(Q) entonces δ /∈ Q, lo cual

indica que β · δ + ρ = α < β · (δ + 1) = β · δ + β, lo cual implica que ρ < β.

Para la unicidad, supongamos que existen δ′ y ρ′ < β tal que α = β · δ′ + ρ′. Luego, α <

β · δ′ + β = β · S(δ′), lo cual implica que S(δ′) /∈ Q. Ademas, β · δ′ ≤ β · δ′ + ρ′ = α,

lo cual da lugar a que δ′ ∈ Q. Luego, como Q es transitivo y S(δ) /∈ Q, se concluye que

δ′ = max(Q) = δ. Luego, ρ = ρ′ por la unicidad en (a)

Para una prueba directa de (a) se toma δ = sup {ξ ∈ ON / α+ ξ ≤ β}. Tambien podemos probar

(b) por induccion sobre α.

1.34 Ejercicio. Pruebe que

(a) α ≤ β ⇒ α+ γ ≤ β + γ.

(b) α+ γ < β + γ ⇒ α < β.

44

Indique contraejemplos de los recıprocos de estas afirmaciones, y un contraejemplo de la propiedad cancela-tiva de la suma a derecha.

1.35 Ejercicio. Pruebe que

(a) α ≤ β ⇒ α · γ ≤ β · γ.

(b) α · γ < β · γ ⇒ α < β.

Indique contraejemplos en ZF−−P de los recıprocos de estas afirmaciones, y un contraejemplo de la propiedadcancelativa del producto a derecha.

1.36 Ejercicio (Exponenciacion de ordinales.). Pruebe que existe una unica funcion e : ON×ON→ ONtal que, si denotamos αβ = e(α, β), se tiene

α0 = 1αS(β) = αβ · ααγ = supβ<γ αβ si γ es lımite.

1.37 Ejercicio. Dado α ordinal, pruebe que

(a) α1 = α.

(b) 1α = 1.

(c) 00 = 1.

(d) α 6= 0⇒ 0α = 0.

1.38 Ejercicio. Dado α > 1, β y γ ordinales, pruebe que

(a) αβ > 0

(b) La funcion eα : ON→ ON, tal que eα(β) = αβ , es normal.

(c) β ≤ αβ .

(d) β < γ ⇔ αβ < αγ .

(e) αβ = αγ ⇔ β = γ.

(f) β ≤ γ ⇔ αβ ≤ αγ .

(g) Si γ es lımite, entonces αγ es lımite.

1.39 Ejercicio. Dados α, β y γ ordinales, pruebe que

(a) αβ+γ = αβ · αγ .

(b) (αβ)γ = αβ·γ .

Ademas, de un contraejemplo en ZF−−P para (α · β)γ = αγ · βγ .

1.40 Ejercicio. Sean α y β ordinales. Pruebe que

(a) α+ β es lımite si y solo si, α es lımite y β = 0, o β es lımite.

(b) α · β es lımite si y solo si α y β no son cero, y alguno de los dos es lımite.

1.41 Ejercicio. Sean α y β ordinales. Pruebe que

(a) Existe γ > β tal que α+ γ = γ.

(b) Si α 6= 0, existe γ > β tal que α · γ = γ.

45

(c) Si α > 1 entonces existe γ > β tal que αγ = γ.

1.42 Ejercicio. Sean α 6= 0 y β > 1 ordinales. Entonces existen unicos η, 0 < δ < β y ρ < βη tal queα = βη · δ + ρ. Sugerencia: Tome η = sup

{ξ ∈ ON / βξ ≤ α

}y luego aplique algoritmo de la division

con βη .

1.43 Ejercicio. Dado α y β > 1 ordinales, pruebe que existen unicos n ∈ ω, f : n→ β \{0} y e : n→ ONestrictamente decreciente, tal que

α =∑k<n

βe(n−1−k) · f(n− 1− k) = βe(n−1) · f(n− 1) + . . .+ βe(2) · f(2) + βe(1) · f(1) + βe(0) · f(0)

(cuando n = 0 esta suma es 0). Sugerencia: Razone por induccion generalizada sobre α. Suponga que todoξ < α satisface el enunciado. Si α = 0 es inmediato. Si α 6= 0 utilize el ejercicio anterior para hallar η,0 < δ < β y ρ < βη tal que α = βη · δ + ρ. Luego, aplique la hipotesis inductiva sobre ρ.

1.44 Ejercicio. Sea A un conjunto. Pruebe que:

(a) nA = {f : n→ A / f func.} es un conjunto para todo n ∈ ω. Sugerencia: Pruebe por induccion sobren que ∃Y ∀f (f ∈ Y ⇔ f : n → A). En el paso inductivo es importante notar que hay una biyeccionentre n+1A y nA×A.

(b) (ZF−−P). A<ω =⋃n∈ω

nA es conjunto.

(c) Si B es finito, entonces BA es conjunto.

(d) (ZF−−P). P<ω(A) = {X ⊆ A / X finito} es un conjunto. Sugerencia: Considere una aplicacionsobreyectiva de A<ω sobre P<ω(A).

1.45 Ejercicio. Sean α y β ordinales. Defina el conjunto

e(β, α) = {f : β → α / f(ξ) = 0 excepto en un subconjunto finito de puntos de β}

Defina la siguiente relacion en e(β, α)

f <e g⇔ ∃ξ∈β(f(ξ) < g(ξ) ∧ ∀η<β(ξ < η⇒ f(η) = g(η))),

es decir, f <e g sii en el maximo ordinal ξ < β (si existe) donde f(ξ) 6= g(ξ), se cumple que f(ξ) < g(ξ).Considere tambien la funcion f0 ∈ e(β, α) de modo que f0(ξ) = 0 para todo ξ < β Pruebe que

(a) Si f, g ∈ e(β, α) y f 6= g entonces f(ξ) 6= g(ξ) solo en un subconjunto finito no vacıo de β.

(b) Si f, g ∈ e(β, α) y f 6= g, entonces existe un unico ordinal ξ < β tal que f(ξ) 6= g(ξ) y f(η) = g(η)para todo ξ < η < β. A este ordinal lo llamaremos ξf,g. Es claro que ξf,g = ξg,f . Si f 6= f0, denotemospor ξf = ξf,f0 , y pruebe que este es el unico ordinal ξ tal que f(ξ) 6= 0 y f(η) = 0 para todo ξ < η < β.Sugerencia: Tome ξ = max {ξ < β / f(ξ) 6= g(ξ)}, el cual existe por el inciso anterior.

(c) 〈e(β, α), <e〉 es un buen orden (con <e como menor estricto). Sugerencia: La propiedad del buen ordense demuestra por induccion generalizada sobre β. Sea C ⊆ e(β, α) no vacıo. Si f0 ∈ C es claroquien es el mınimo. Si f0 /∈ C considere µ = mın {ξf / f ∈ C}, M = {f ∈ C / ξf = µ} y ν =mın {f(µ) / f ∈M}. SeaB = {f ∈M / f(µ) = ν} yA = {f�µ / f ∈ B} ⊆ e(µ, α). Por la hipotesisinductiva,A tiene un mınimo h′. Luego, existe un unico h ∈ B tal que h′ = h�µ. Pruebe que h = mınC.

46

(d) type(e(β, α)) = αβ . Sugerencia: El resultado es evidente si α ≤ 1 o β = 0. Suponga entonces que α > 1y β 6= 0. Si τ < αβ , utilize el ejercicio 1.43 para probar que existen unicos nτ ∈ ω, fτ : nτ → α \ {0}y eτ : nτ → β estrictamente decreciente, tal que τ =

∑k<nτ

αeτ (nτ−1−k) · fτ (nτ − 1 − k) =αeτ (nτ−1) ·fτ (nτ −1) + . . .+αeτ (2) ·fτ (2) +αeτ (1) ·fτ (1) +αeτ (0) ·fτ (0). Definamos Fτ : β → α demodo que, si ξ = eτ (k) para algun (unico) k < nτ , entonces Fτ (ξ) = fτ (k), de lo contrario, Fτ (ξ) = 0.Esto da lugar a una funcion F : αβ → e(β, α) tal que F (τ) = Fτ . Esta funcion es un isomorfismo. Sepuede tomar como inversa a la funcion G : e(β, α)→ αβ tal que

G(f) =ξ<β∑

αξ · f(ξ) = . . .+ α2 · f(2) + α · f(1) + f(0)

(Suma recorriendo los ξ < β en orden decreciente hasta 0. Note que, debido a que f es 0 excepto en unnumero finito de puntos de β, esta suma es finita).

(e) e(β, α) es un conjunto. Sugerencia: La prueba es directa del inciso anterior. Sin embargo, hay otro tipode prueba, dada como sigue. Considere casos para β. Si β ∈ ω ya esta resuelto. Si β /∈ ω entoncesexiste un ordinal lımite ≤ β, dando lugar a que ω = ω es conjunto. Por lo tanto, se puede probar queA =

⋃X∈P<ω(β)

Xα es un conjunto y, luego, construya una funcion sobreyectiva de A sobre e(β, α).

47

48

Capıtulo 2

Numeros Cardinales

Ası como los numeros ordinales se contruyen para medir longitud, los numeros cardinales se

definen para medir tamano. Para definir los cardinales existen dos enfoques, el primero toma aque-

llos ordinales cuyo tamano es mayor que el de sus predecesores (1.3), el segundo fija una funcion

| · | : V → ON que cumple que |x| = |y| si y solo si x ≈ y, de modo que esta definicion contenga

a la primera (ver Capıtulo 3). La segunda definicion se utiliza en los cursos basicos de Introduccion

a la Teorıa de Conjuntos sin definirla precisamente. Puesto que ambas definiciones son equivalentes

bajo el Axioma de Eleccion y para definir la segunda se requiere del axioma de Regularidad, solo

nos concentraremos en utilizar la primera definicion para los fines de este capıtulo (como lo hacen

los libros que trabajan en topicos avanzados de la Teorıa de Conjuntos). Una distincion que podemos

dar entre ambas definiciones es que en la primera los cardinales estan bien ordenados, mientras que

en la segunda se necesita el Axioma de Eleccion para garantizar que estan linealmente ordenados

(ver 3.5), y que en la primera definicion no se puede definir, sin AC, el cardinal de un conjunto que

no tenga buen orden.

En este capıtulo nos centramos en definir los conceptos y propiedades basicas de los numeros

cardinales. Seremos muy precisos al indicar cuando un resultado requiere o no el Axioma de Elec-

cion. En la primera seccion definimos los cardinales (alephs) y su estructura bien ordenada con sus

propiedades basicas; en la segunda definimos la aritmetica cardinal que resulta independiente del

Axioma de Eleccion; en la tercera introducimos el Cardinal de Hartogs y su aplicacion para obtener

resultados sin el Axioma de Eleccion, e incluso equivalencias interesantes con este axioma; en la

cuarta introducimos la nocion de cofinalidad y en la ultima seccion demostramos propiedades de la

exponienciacion y de la aritmetica infinita de cardinales, las cuales dependen fuertemente de AC.

En principio, trabajamos las secciones 2.1, 2.2 y 2.4 en la teorıa ZF−−P−Inf , la seccion 2.3 en

ZF−−Inf y la seccion 2.5 en ZF−, a menos que se indique lo contrario.

Para el desarrollo de este capıtulo, asumimos que el lector conoce las propiedades basicas de

equipotencia y cardinales que se dictan en un curso introductorio de Teorıa de Conjuntos.

2.1. Cardinales y Alephs

Recordemos la notacion A ≈ B (A equipotente con B) que significa que existe una biyeccion

entre A y B, la cual denota que A y B tienen el mismo tamano. A � B (A esta dominado por B)

denota que existe una funcion inyectiva de A en B, lo que intuitivamente significa que el tamano

de A es menor o igual que el de B. Tambien tenemos A ≺ B (A domina estrictamente a B) si

A � B ∧A 6≈ B.

1.1 Lema. ≈ es una relacion de equivalencia,� es reflexiva y transitiva.A � B yB � A implican

A ≈ B (Cantor-Bernstein 1887). ≺ es irreflexiva y transitiva

Con el fin de asignar un numero que represente el tamano de un conjunto, introducimos la

siguiente

1.2 Definicion (Cardinal). Dado un conjunto A que tiene un buen orden, |A| = mın {α / A ≈ α}se llama el cardinal de A. Un ordinal α lo llamamos cardinal si |α| = α.

Como los cardinales son ordinales entonces estos heredan el buen orden de ON, es decir, si

llamamos CA = {α / α cardinal} entonces CA esta bien ordenado por ∈. Esto llevarıa a pensar

que ∀A,B(A � B ∨ B � A), pero esto ultimo es una afirmacion muy diferente al buen orden de

cardinales pues, segun la definicion anterior, solo es posible definir la cardinalidad de un conjunto

bien ordenado1. De este modo la afirmacion anterior es valida solo para buenos ordenes, y el hecho

de que se cumpla en general es equivalente a AC (ver 3.5). Bajo AC es claro que todo conjunto

tiene un cardinal.

1.3 Lema. Sean A un conjunto que tiene buen orden.

(a) |A| ≈ A. (b) |α| ≤ α.

(c) α es un cardinal si y solo si ∀β<α(α 6≈ β).

(d) |A| es un cardinal, por lo cual ||A|| = |A|.

(e) Si β ≤ α y β es un cardinal, entonces β ≤ |α|. Esto indica que |α| es el mayor cardinal ≤ α.1Cohen probo en 1963 que en ZF no se puede demostrar que P(ω) tiene buen orden.

50

(f) α ≤ β ⇒ |α| ≤ |β|. (g) |α| < |β| ⇒ α < β.

Demostracion. (a) Inmediato de la definicion de |A|.

(b) Inmediato de que α ≈ α, pues |α| es el menor ordinal equipotente con α.

(c) Si α es un cardinal, es decir, |α| = α, entonces α es el menor cardinal equipotente con α, por

lo cual todo ordinal menor que α no es equipotente con α. Recıprocamente, si |α| 6= α, de (b)

se tiene |α| < α y claramente |α| ≈ α, por lo cual ∃β<α(α ≈ β).

(d) Sea α = |A|. Si β < α entonces A 6≈ β, por lo cual α 6≈ β.

(e) Supongamos que γ = |α| < β ≤ α, es decir, γ ⊆ β ⊆ α. Como α ≈ γ entonces γ ≈ β con

γ < β y, por (c), β no es cardinal.

(f) |α| ≤ α ≤ β y, por (e), |α| ≤ |β|.

(g) Contrarrecıproco de (f).

El siguiente Teorema expresa la relacion entre ≈, � y ≺ con el buen orden de los cardinales.

1.4 Teorema. Sean A y B conjuntos que tienen buen orden.

(a) A ≈ B ⇔ |A| = |B|.

(b) Sean α y β cardinales. Si α � β entonces α ≤ β

(c) A � B ⇔ |A| ≤ |B|.

(d) A ≺ B ⇔ |A| < |B|.

Demostracion. Fijemos α = |A| y β = |B|. Claramente A ≈ α y B ≈ β.

(a) Si A ≈ B entonces α ≈ B y A ≈ β. Luego, por definicion de cardinalidad, α ≥ |B| = β y

β ≥ |A| = α. Por lo tanto α = β. El recıproco es directo de que α ≈ β.

(b) Supongamos por el contrario que β < α, lo cual implica que β � α. Luego, por el Lema 1.1

α ≈ β y, por (a), α = |α| = |β| = β lo cual es una contradiccion.

(c) Si A � B entonces α � β, lo cual implica que α ≤ β por (b). Recıprocamente, si α ≤ β es

claro que α � β y, por lo tanto, A � B.

51

(d) A ≺ B ⇔ A � B ∧A 6≈ B ⇔ |A| ≤ |B| ∧ |A| 6= |B|.

Introducimos a continuacion resultados que nos permiten distinguir cuando un cardinal u ordinal

es finito o infinito, tanto en tamano, como en longitud. Para denotar cardinales, usaremos las letras

del tipo κ, λ, µ, etc.

1.5 Lema. ∀n∈ω(n cardinal)

Demostracion. Si m < n es claro que m 6≈ n. Por lo tanto, n es un cardinal.

Recordemos que un conjuntoA es finito si ∃n∈ω(A ≈ n). Es claro en este caso queA tiene buen

orden, por lo cual |A| esta definido. Ademas, |A| = |n| = n, es decir, |A| es un numero natural. De

este modo, el tamano de un conjunto finito se mide mediante un numero natural.

1.6 Definicion. κ es un cardinal finito si κ ∈ ω. De lo contrario, es un cardinal infinito. Un ordinal

γ es infinito si γ /∈ ω.

1.7 Lema. Si A ≈ B entonces A es finito si y solo si B es finito. Ademas, si |A| se puede definir,

entonces A es finito si y solo si |A| es cardinal finito.

1.8 Lema (ZF−−P). ω es un cardinal infinito.

Demostracion. Es claro que ∀n<ω(ω 6≈ n). Ası, ω es un cardinal y es infinito.

1.9 Lema. (a) (ZF−−P). Si ω � A y x /∈ A, entonces A ∪ {x} ≈ A.

(b) Si α es un ordinal infinito, entonces ω es conjunto y ω ≤ α. Ademas, α ≈ α+ 1.

(c) (ZF−−P). α es cardinal (ordinal) infinito si y solo si α ≥ ω. De esto se sigue que ω es el menor

cardinal (ordinal) infinito.

(d) Si A es infinito y tiene buen orden, entonces ω es conjunto y ω � A2.

Demostracion. (a) Si ω � A y x /∈ A, considere B ⊆ A tal que B ≈ ω ≈ ω \ {0}. Por lo tanto,

B∪{x} ≈ (ω\{0})∪{0} = ω ≈ B y luegoA = (A\B)∪B ≈ (A\B)∪(B∪{x}) = A∪{x}.

(b) Si α es un ordinal infinito entonces α /∈ ω, lo cual implica que ω es conjunto y α ≥ ω. Luego,

ω � α, lo cual implica que α+ 1 = α ∪ {α} ≈ α.2Sin AC no se puede demostrar que todo conjunto infinito domina a ω, mas aun, no se puede demostrar que si A es

infinito y x /∈ A, entonces A ∪ {x} ≈ A.

52

(c) De (b) tenemos una implicacion. Ahora, si α ≥ ω es cardinal (ordinal) es claro que α no es un

numero natural, es decir, es un cardinal (ordinal) infinito.

(d) Sea A infinito y R un buen orden para A. Sea α = type(A,R). Como A es infinito, es claro que

α no es un numero natural, es decir, α es un ordinal infinito. Por (b), ω es conjunto y α ≥ ω, de

donde ω � α y α ≈ A y, por lo tanto, ω � A.

1.10 Lema. α es un ordinal infinito si y solo si |α| es un cardinal infinito.

Demostracion. Si α es ordinal finito, es un numero natural y, por lo tanto, α = |α| es un cardinal

finito. Recıprocamente, si α es un ordinal infinito, entonces ω es conjunto y α ≥ ω por el Lema 1.9.

Luego, de 1.8 se sigue que |α| ≥ ω, es decir, |α| es un cardinal infinito.

1.11 Lema. Todo cardinal infinito es un ordinal lımite.

Demostracion. Si κ es un cardinal infinito, es claro que κ 6= 0. Ahora, si κ = β + 1 para algun β,

entonces β es ordinal infinito o, de lo contrario, κ serıa numero natural. Por 1.9, κ = β + 1 ≈ β, lo

cual contradice que κ es un cardinal.

1.12 Corolario. Las siguientes afirmaciones son equivalentes

(a) Axioma del Infinito. (b) Existe un ordinal infinito. (c) Existe un cardinal infinito.

Demostracion. (a)⇒ (b) se sigue de que ω es un ordinal infinito. (b)⇒ (c) se sigue de 1.10.

(c)⇒ (a) es inmediato del Lema 1.11.

2.1 Ejercicio. Dado un conjunto A, decimos que U ⊆ P(A) es una familia inductiva de subconjuntos de Asi 0 ∈ U y ∀z∈A∀X∈U (X ∪{z} ∈ U). Decimos que A es WR-finito si pertenece a cualquier familia inductivade subconjuntos de A. Pruebe que

(a) 0 es WR-finito. Si X es WR-finito entonces X ∪ {z} es WR-finito.

(b) (Induccion sobre conjuntos finitos) Sea ϕ(x) una formula. Si ϕ(0) y para todo X WR-finito y z /∈ X setiene ϕ(X)⇒ ϕ(X ∪ {z}), entonces ∀X(X WR− finito⇒ ϕ(X)).

(c) Si F es funcion y A es WR-finito, entonces F[A] es WR-finito.

(d) Si A ≈ B entonces A es WR-finito si y solo si B es WR-finito.

(e) Todo numero natural es WR-finito.

(f) X es WR-finito si y solo si X es finito. Por lo tanto, todas las afirmaciones anteriores se cumplen paraconjuntos finitos.

53

(g) Si 〈A,≤〉 es un C.P.O. finito no vacıo, entonces tiene un elemento maximal y uno minimal.

(h) X es finito si y solo si todo conjunto U ⊆ P(A) no vacıo tiene un elemento maximal (respecto a lacontencion).

2.2 Ejercicio. Un conjunto A es Dedekind-infinito si es equipotente con un subconjunto propio de el. De locontrario se llama Dedekind-finito. Pruebe que

(a) Todo conjunto finito es Dedekind-finito3.

(b) Si A es Dedekind-infinito entonces existe una funcion inyectiva F : ω → A.

(c) El Axioma del Infinito equivale a la existencia de un conjunto Dedekind infinito.

(d) A es Dedekind-infinito si y solo si ω � A (ω es conjunto al cumplirse alguna de las dos afirmaciones,ası que no es necesario en este caso considerar el Axioma del Infinito.)

(e) A es Dedekind-infinito si y solo si A ∪ {x} ≈ A para todo x /∈ A.

2.3 Ejercicio (ZF−−Inf ). (a) Si A es infinito entonces existe una funcion inyectiva F : ω → P(P(A)).Esto hara que ω sea conjunto y, por lo tanto, ω � P(P(A)). Pruebe tambien el recıproco de este ejercicio.

(b) El Axioma del Infinito equivale a la existencia de un conjunto infinito.

2.4 Ejercicio. Sea κ un cardinal y A ⊆ κ. Entonces |A| = κ si y solo si type(A,∈) = κ.

1.13 Definicion ((ZF−−P) Conjunto contable). Un conjunto A es contable si A � ω, y es enu-

merable si A ≈ ω. Es claro que todo conjunto contable hereda un buen orden de ω, por lo cual

podemos definir |A| y |A| ≤ ω. De este modo, decimos que un cardinal κ es contable si κ ≤ ω (es

decir, es finito o es ω).

1.14 Ejemplo (ZF−−P). Si α y β son ordinales contables, entonces α + β y α · β son contables.

En efecto, α + β es equipotente (mas aun, isomorfo) a la union disjunta de dos buenos ordenes de

tipo α y β, respectivamente, y dicha union es contable. α · β es equipotente (isomorfo) a β × α, el

cual es contable. Para la exponenciacion sucede lo mismo, αβ es contable (ver seccion 2.5). De este

modo, tenemos que ωω es contable. Esto ilustra que, con los metodos definidos hasta el momento

sin el Axioma de Partes, no podemos generar conjuntos ni cardinales no contables. Es un hecho que

no se puede probar la existencia de conjuntos no contables sin el Axioma de Partes.

Vamos a probar que, con el Axioma de Partes, podemos generar cardinales no contables y, mas

aun, generar la estructura completa de los cardinales infinitos, la cual es isomorfa a ON.

1.15 Lema. Si A es un conjunto de cardinales, entonces sup(A) es un cardinal.

Demostracion. Sea γ = sup(A) y β ∈ γ. Luego, existe κ ∈ A tal que β < κ y, como κ es un

cardinal, β 6≈ κ. Ahora, si β ≈ γ entonces, como β ⊆ κ ⊆ γ entonces los tres ordinales serıan

equipotentes, lo cual no es cierto. Por lo tanto, γ 6≈ β.3El recıproco de esta afirmacion no se puede probar sin AC

54

1.16 Lema (Cantor 1892). No existe una funcion sobreyectiva de A en P(A).

1.17 Lema (ZF−−Inf ). ∀α∃κ∈CA(κ > α).

Demostracion. Si α es finito, tome κ = α+1. Supongamos ahora que α es infinito y consideremos

el conjunto B = {R ⊆ α× α / 〈α,R〉 C.B.O.} (este es conjunto por el Axioma de Partes). Luego,

definamos N = {γ / ∃R∈B(type(α,R) = γ)}, el cual es conjunto por el Axioma de Reemplaza-

miento. Por lo tanto, existe κ = supN .

Veamos primero que N = {γ / γ ≈ α}. Es claro que se da ⊆. Ahora, si γ ≈ α entonces existe una

funcion biyectiva f : γ → α. Si definimos R ⊆ α× α tal que xRy⇔ f−1(x) ∈ f−1(y), entonces

〈α,R〉 es un C.B.O. y type(α,R) = γ, lo cual implica que γ ∈ N .

Es claro que α es un ordinal infinito. Luego, si γ ∈ N entonces γ ≈ α y, por 1.9, γ + 1 ≈ γ ≈ α,

es decir, γ + 1 ∈ N . Por lo tanto ∀γ∈N (γ + 1 ∈ N). Es claro que α ∈ N , lo cual implica que

α+ 1 ∈ N y α < α+ 1 ≤ κ.

Resta probar que κ es un cardinal. En efecto, κ /∈ N , o de lo contrario, κ + 1 ∈ N y luego

κ+1 ≤ supN = κ, lo cual es falso. De esta forma se garantiza que κ 6≈ α. Ahora, si β < κ =⋃N ,

existe γ ∈ N tal que β < γ. Si β ≈ κ, como β ⊆ γ ⊆ κ entonces κ ≈ γ ≈ α, lo cual es una

contradiccion.

1.18 Definicion (ZF−−Inf ). Dado un ordinal α definimos α+ = mın {κ ∈ CA / κ > α} el car-

dinal sucesor de α. Es claro que α+ es el menor cardinal mayor que α. Si κ es un cardinal infinito,

decimos que κ es un cardinal sucesor si ∃α(κ = α+). Decimos que κ es un cardinal lımite si κ > ω

y no es cardinal sucesor.

1.19 Lema (ZF−−Inf ). (a) α+ > α. Si κ es un cardinal, entonces κ ≤ α⇔ κ < α+

(b) ¬∃κ∈CA(α < κ < α+).

(c) α+ es cardinal infinito si y solo si α es ordinal infinito (si n es un natural, entonces n+ = S(n)).

(d) Si κ es un cardinal, entonces α < κ⇔ α+ ≤ κ.

(e) α+ = |α|+

(f) λ es un cardinal sucesor si y solo si ∃κ∈CA(λ = κ+). Aquı κ es un predecesor de λ, es unico y

es infinito.

Demostracion. (a) Si κ < α+ entonces, como α+ es el menor cardinal mayor que α, entonces

κ ≤ α. El recıproco es claro de que κ ≤ α < α+.

55

(b) Directo de (a).

(c) Si α es ordinal finito, es decir, un numero natural, es claro que α+ = S(α), el cual es un cardinal

finito. Si α es un ordinal infinito, como α+ > α entonces α+ es un cardinal infinito.

(d) Si α < κ entonces de (b) se sigue que α+ ≤ κ. El recırpoco se sigue de que α < α+ ≤ κ.

(e) Como |α| ≤ α < α+ de (d) se sigue que |α|+ ≤ α+. De 1.3, como |α| < |α|+ = ||α|+|entonces α < |α|+ y, por (d), α+ ≤ |α|+.

(f) Si λ es cardinal sucesor, existe α tal que λ = α+ = |α|+, ası que tomese κ = |α|. De (c)

tenemos que α, y por lo tanto κ, son infinitos. Si κ′ es un cardinal tal que λ = κ′+ entonces

κ+ = κ′+. Luego, como κ < κ′+ entonces κ ≤ κ′ por (a). De la misma forma, como κ′ < κ+

entonces κ′ ≤ κ.

Los cardinales lımite, ası como los ordinales lımite, son el lımite de una sucesion de cardinales

que no contienen a dicho cardinal, como se prueba en el siguiente resultado.

1.20 Lema (ZF−). Sea λ un cardinal. Las siguientes propiedades son equivalentes.

(i) λ es un cardinal lımite.

(ii) λ > ω y ∀α<λ(α+ < λ).

(iii) λ > ω y ∀α<λ∃κ∈CA(α < κ < λ).

(iv) λ > ω y sup {κ < λ / κ cardinal} = λ.

(v) Existe un conjunto de cardinales X tal que algun cardinal infinito esta en X , λ /∈ X y

sup(X) = λ.

Demostracion. Para (i)⇒ (ii), si α < λ entonces por 1.19 α+ ≤ λ, pero α+ 6= λ al ser λ cardinal

lımite (ademas, λ > ω). Para (ii)⇒ (iii) tome κ = α+. Para (iii)⇒ (iv), si α < λ entonces por,

(iii), α < sup {κ < λ / κ cardinal}, por lo cual λ ⊆ sup {κ < λ / κ cardinal}. La otra contencion

es inmediata. (iv)⇒ (v) es inmediato de tomar X = {κ < λ / κ cardinal} y es claro que ω ∈ X .

Finalmente, para (v)⇒ (i), es claro que ∀κ∈X(κ < λ). Como algun cardinal infinito pertenece a

X entonces ω < λ. Ahora, si λ fuera un cardinal sucesor, existe un cardinal (infinito) µ tal que

λ = µ+. Luego, ∀κ∈X(κ ≤ µ), lo cual contradice que λ = sup(X).

56

Ası como los ordinales, los cardinales infinitos se clasifican en ω (contable), sucesor o lımite.

Esta estructura no es ajena a la estructura ordinal, de hecho son isomorfas segun la siguiente defini-

cion.

1.21 Definicion ((ZF−). Alephs). Por recursion transfinita podemos definir la siguiente sucesion

indizada por ordinales

ω0 = ω

ωα+1 = ω+α

ωγ = supα<γ

ωα si γ es ordinal lımite.

En la mayorıa de los textos esta sucesion se denota por ℵα = ωα (los famosos alephs, ası, ℵ0 = ω).

Veamos que todos los cardinales infinitos estan definidos dentro de esta sucesion.

1.22 Teorema (ZF−). (a) La funcion ℵ : ON→ CA, definida por ℵ(α) = ωα, es normal.

(b) Si κ es un cardinal infinito, entonces existe un unico α tal que κ = ωα.

(c) ωα es un cardinal sucesor si y solo si α es sucesor.

(d) ωγ es un cardinal lımite si y solo si γ es lımite.

Demostracion. (a) Por 1.15 es facil probar por induccion que ωα es un cardinal infinito. Es claro

que la funcion ℵ es continua y que ℵ(α) < ℵ(α+ 1), lo cual hace de ℵ una funcion normal.

(b) Si κ fuera cota superior de los ωα entonces ℵ : ON → κ y, como ℵ es inyectiva, entonces

ℵ−1[κ] = ON es conjunto, lo cual es falso. Por lo tanto, {α / κ ≤ ωα} 6= 0, por lo cual tiene

mınimo, sea este α. Si α = 0 entonces κ ≤ ω y, al ser infinito, κ = ω; si α = β + 1 para algun

β, entonces ωβ < κ, por lo cual ωβ+1 = ω+β ≤ κ ≤ ωβ+1; y si α es ordinal lımite, entonces,

como ∀β<α(ωβ < κ) entonces ωα = supβ<α ωβ ≤ κ ≤ ωα. En cualquier caso, κ = ωα.

(c) Si ωα es un cardinal sucesor, entonces existe κ cardinal infinito tal que ωα = κ+. Por (b) existe

β tal que κ = ωβ , por lo cual ωα = ωβ+1 y luego α = β + 1 por la inyectividad de ℵ. El

recıproco es sencillo de probar.

(d) Si ωγ es cardinal lımite entonces γ no es ordinal sucesor por (c), y como ωγ > ω0 entonces

γ > 0, lo cual hace que γ sea lımite. Para el recıproco, por (c) ωγ no es cardinal sucesor y,

como γ > 0 entonces ωγ > ω, por lo cual ωγ es un cardinal lımite.

57

2.5 Ejercicio (ZF−−Inf ). Pruebe que el κ hallado en la prueba de 1.17 es α+.

2.6 Ejercicio (ZF−). Pruebe que CA no es conjunto.

2.7 Ejercicio (ZF−). Pruebe que:

(a) α ≤ ωα.

(b) Si α no es ordinal lımite entonces α < ωα.

(c) Todo ordinal α tal que ωα = α es un ordinal lımite.

(d) Dado un ordinal β existe un ordinal α > β tal que ωα = α.

2.2. Aritmetica Cardinal

En esta seccion determinamos la suma y producto cardinal con las conocidas propiedades de

absorcion de cardinales (2.6). Lo interesante es que estos resultados no requieren AC al tratarse de

cardinales bien ordenados.

2.1 Definicion (Suma y Producto Cardinal). Definimos κ⊕λ = |(κ×{0})∪ (λ×{0})| y κ⊗λ =

|κ× λ|. En relacion con la suma y producto de ordinales, es claro que κ⊕ λ = |κ+ λ| = |λ+ κ| yκ⊗ λ = |κ · λ| = |λ · κ|.

2.2 Lema. Sean K y L conjuntos que tienen buen orden. Entonces

(a) Si K ∩ L = 0 entonces |K ∪ L| = |K| ⊕ |L|.

(b) |K × L| = |K| ⊗ |L|

Por lo tanto, segun las propiedades de equipotencia, se siguen las propiedades aritmeticas de la

suma y producto de cardinales.

Demostracion. Se sigue directamente de que K ∪ L ≈ (|K| × {0}) ∪ (|L| × {1}) y de que

K × L ≈ |K| × |L|.

En el resultado 3.5 probamos que AC equivale a que, para todo conjunto infinitoA,A×A ≈ A.

En contraste con los cardinales definidos, podemos probar sin AC que κ⊗κ = κ para todo cardinal

infinito κ. Este hecho sera definito para conocer la suma y producto de dos cardinales. Para efectuar

esta prueba, debemos definir primero un buen orden para ON×ON que lo haga isomorfo a ON y

tal que κ× κ sea un segmento inicial de ese orden isomorfo a κ, esto para todo cardinal infinito κ.

58

2.3 Teorema. Definamos la siguiente relacion de orden en ON×ON

(α, β) <∗ (γ, δ)⇔ max{α, β} < max{γ, δ} ∨ (max{α, β} = max{γ, δ} ∧ (α, β) < (γ, δ))

donde (α, β) < (γ, δ) denota el orden lexicografico de ON×ON, es decir

(α, β) < (γ, δ)⇔ α < γ ∨ (α = γ ∧ β < δ).

Denotemos por S∗(α,β) = S(α,β)(ON×ON, <∗)

(a) (α, β) <∗ (γ, δ)⇒ max{α, β} ≤ max{γ, δ}.

(b) El orden lexicografico de ON×ON es un buen orden (pero no todos sus segmentos iniciales

son conjuntos).

(c) El orden <∗ es un buen orden para ON×ON.

(d) S∗(0,α) = α× α.

(e) S∗(α,β) es conjunto.

Demostracion. (a) Inmediato de la definicion de <∗.

(b) Se prueba de la misma forma que el Ejercicio 1.1.6.

(c) Es inmediato que <∗ es irreflexivo. Para la transitividad, supongamos que (α, β) <∗ (γ, δ) y

(γ, δ) <∗ (µ, ν). Si max{α, β} < max{µ, ν} es claro que (α, β) <∗ (µ, ν); si max{α, β} =

max{µ, ν} entonces tambien es igual a max{γ, δ}, por lo cual (α, β) < (γ, δ) y (γ, δ) <

(µ, ν), lo cual implica que (α, β) < (µ, ν) y, luego, (α, β) <∗ (µ, ν). Ahora veamos la

propiedad del buen orden (la cual implica que<∗ tambien es orden lineal). SeaC ⊆ ON×ON

no vacıo. Definamos M = {max{α, β} / (α, β) ∈ C} el cual es un conjunto no vacıo de ordi-

nales. Sea η = mın(M) y P = {(α, β) / max{α, β} = η}, el cual es no vacıo. Bajo el orden

lexicografico tomemos (µ, ν) = mın(P ), de donde es facil probar que este es el mınimo de C

bajo el orden <∗.

(d)

(ξ, η) <∗ (0, α)⇔ max{ξ, η} < α ∨ (max{ξ, η} = α ∧ (ξ, η) < (0, α))

⇔ (ξ < α ∧ η < α) ∨ ((η ≤ ξ = α ∨ ξ ≤ η = α) ∧ ξ = 0 ∧ η < α)

⇔ (ξ < α ∧ η < α) ∨ (η = α ∧ η < α)⇔ ξ < α ∧ η < α⇔ (ξ, η) ∈ α× α.

59

(e) Sea γ = max{α, β}+ 1. Es claro que (α, β) <∗ (0, γ), por lo cual S∗(α,β) ⊆ S∗(0,γ) = γ × γ.

2.4 Teorema. Dado un cardinal infinito κ, κ× κ = S∗(0,κ)∼= κ. Esto implica que κ⊗ κ = κ.

Demostracion. Veamos por induccion sobre κ que

κ cardinal infinito⇒ S∗(0,κ)∼= κ.

Supongamos que todo α < κ satisface la afirmacion anterior, y supongamos que κ es un cardinal

infinito. Si α < κ entonces |α × α| = |α| ⊗ |α| es finito o es igual a |α| por la hipotesis inductiva,

ası que en ambos casos |α×α| < κ. Luego, dado α, β < κ tenemos que γ = max{α, β}+1 < κ y

luego, segun la prueba del Teorema anterior,∣∣∣S∗(α,β)

∣∣∣ ≤ |γ × γ| < κ. Al considerar S∗(0,κ) = κ× κbien ordenado por <∗ tenemos que, S∗(0,κ)

∼= κ, o que S∗(α,β)∼= κ para algun (α, β) ∈ S∗(0,κ),

o S∗(0,κ)∼= γ para algun γ < κ. El segundo caso no es posible, o de lo contrario κ =

∣∣∣S∗(α,β)

∣∣∣ < κ,

y el tercer caso tampoco es cierto, pues de lo contrario κ ≤ |κ ⊗ κ| =∣∣∣S∗(0,κ)

∣∣∣ = |γ| < κ. Por lo

tanto, S∗(0,κ)∼= κ.

2.5 Corolario (ZF−). Dado un cardinal infinito κ existe un unico isomorfismo fκ : κ×κ→ κ con

κ× κ ordenado por <∗. Los fκ con κ cardinal infinito son compatibles, por lo cual Γ =⋃κ≥ω fκ

es el unico isomorfismo entre 〈ON × ON, <∗〉 y ON. Ademas, Γ(0, κ) = κ para todo cardinal

infinito κ.

Demostracion. De 2.4 y 1.2.4 se sigue la existencia unica del isomorfismo fκ con κ cardinal in-

finito. Sin perder generalidad, supongamos κ < λ donde κ y λ son cardinales infinitos. Luego,

fλ�S∗(0,κ) : κ× κ→ Sfλ(0,κ) es un isomorfismo y, como Sfλ(0,κ) = fλ(0, κ) y κ ∼= κ× κ entonces

fλ(0, κ) = κ. Ası, fλ�S∗(0,κ) y fκ son isomorfismos entre κ × κ y κ, por lo cual son iguales y, por

lo tanto, fλ y fκ son compatibles. Luego, Γ =⋃κ≥ω fκ es una biyeccion entre ON ×ON y ON

(esto es porque CA no es conjunto) y se conserva la propiedad de isomorfismo. La unicidad de Γ se

prueba de forma similar a 1.2.4. Si tomamos cualquier λ > κ, entonces Γ(0, κ) = fλ(0, κ) = κ.

2.6 Corolario (Absorcion de cardinales). Sean κ y λ cardinales, alguno de los dos infinito. Entonces

κ⊕ λ = max{κ, λ} y, si ambos cardinales son no nulos, entonces κ⊗ λ = max{κ, λ}.

Demostracion. Sin perder generalidad, supongamos que κ ≤ λ con λ infinito. Luego

λ ≤ κ⊕ λ ≤ λ⊕ λ ≤ 2⊗ λ ≤ λ⊗ λ = λ

60

y, si κ > 0, entonces

λ ≤ κ⊗ λ ≤ λ⊗ λ = λ.

Segun el contexto, escribiremos κ + λ y κ · λ = κλ para denotar la suma y el producto de

cardinales, a menos que se indique que corresponde a la suma y producto ordinal.

2.7 Corolario. ωα + ωβ = ωmax{α,β} y ωα · ωβ = ωmax{α,β}

2.8 Definicion. AB = {f / f : A→ B}. En el ejercicio 1.1.44 planteamos que AB es conjunto

cuando A es finito. Bajo el Axioma de Partes es claro que AB siempre es conjunto.

2.9 Definicion (ZF−−Inf ). Si κλ tiene buen orden, definimos κλ = |κλ|. Cuando se trate de

cardinales, la exponenciacion sera como cardinales y no como ordinales, a menos que se indique lo

contrario. Bajo AC la exponenciacion de cardinales siempre esta definida.

La exponenciacion de ordinales y de cardinales no coincide. Por ejemplo, 2ω = supn<ω 2n = ω

como exponenciacion ordinal, mientras que ω ≺ ω2 (de hecho, 2ω no se puede definir en ZF como

cardinal bien ordenado). Si λ es cardinal finito entonces κλ siempre esta definido y, si κ es finito,

κλ coincide con la exponenciacion como numeros naturales.

Bajo AC es claro que |P(κ)| = 2κ y, de 1.16, se sigue que κ < 2κ. Ası como en la suma y pro-

ducto de cardinales, cabe preguntarse si, dados α y β, se puede determinar un γ tal que ωωβα = ωγ .

Esto no se puede dar en ZFC pues, bajo la consistencia de ZFC, la afirmacion ω1 = 2ω no es

contradictoria, pero tampoco lo es 2ω = ωω1 ni 2ω7 = ω15, de modo que la exponenciacion de

cardinales es parte de los misterios que se dan en la Teorıa de Conjuntos.

Sin embargo, el misterio de la exponenciacion es resuelto totalmente cuando trabajamos con

la Hipotesis Generalizada del Continuo (GCH Generalized Continuum Hypothesis). Esta extiende

a la Hipotesis del Continuo (CH Continuum Hypothesis), afirmacion conjeturada por Cantor la

cual dice que no existe un subconjunto infinito de R que no es equipotente con ω ni con R. Esta

afirmacion se reduce a

CH ¬∃B(ω ≺ B ≺ ω2).

GCH Para todo conjunto infinito A, ¬∃B(A ≺ B ≺ A2).

61

Bajo el Axioma de Eleccion, las afirmaciones anteriores se reducen a

CHAC 2ω = ω1.

GCHAC 2κ = κ+ para todo cardinal infinito κ.

Para GCH no es necesario distinguir entre ambas afirmaciones, pues es un resultado de ZF−

que GCH⇒AC (Sierpinski, 1947), de donde GCH⇔GCHAC es Teorema de ZF−. En la

seccion 2.5 veremos como GCH determina la exponenciacion de cardinales (5.3), mas algunos

resultados interesantes sobre esta.

2.8 Ejercicio. Pruebe que ON×ON, bajo el orden lexicografico, no es isomorfo a ON.

2.9 Ejercicio. (a) Si n ∈ ω y A tiene buen orden, entonces nA tiene buen orden.

(b) La exponenciacion cardinal esta definida en ω y coincide con la exponenciacion ordinal.

(c) Si κ es un cardinal infinito y n ∈ ω, entonces κn esta definido y κn = κ.

2.10 Ejercicio. Sin utilizar el Teorema 2.4, pruebe directamente que κ + κ = κ para cualquier cardinalinfinito κ.

2.11 Ejercicio. Suponga que κ < λ. Pruebe que

(a) κ⊕ λ = κ+ λ (suma ordinal).

(b) κ⊗ λ = κ · λ (producto ordinal)

2.12 Ejercicio. Sea κ un cardinal infinito.

(a) Si µ < κ es un cardinal, entonces |κ \ µ| = κ.

(b) (ZF−−Inf ). Pruebe que κ+ \ κ = {α / |α| = κ} y que su cardinal es κ+.

2.13 Ejercicio. Sea κ un cardinal infinito. Si 0 < λ ≤ κ pruebe que κ se puede particionar en λ subconjuntosde tamano κ. ¿Sucede lo mismo si λ > κ? Sugerencia: Utilize una biyeccion g : λ× κ→ κ.

2.14 Ejercicio. Pruebe, bajo AC, que CHAC ⇔ CH y GCHAC ⇔GCH.

2.3. Numero de Hartogs

El numero de Hartogs es una herramienta para determinar, sin usar AC, la existencia de un

cardinal “cercano” a un conjunto arbitario (al cual no se le pueda definir buen orden). Este permite

demostrar resultados que “aproximan” a afirmaciones fuertes que solo se pueden probar bajo AC,

ademas de que permite probar equivalencias interesantes del AC con resultados relacionados a la

equipotencia de conjuntos. Todos los resultados de esta seccion se levan a cabo en ZF−−Inf , a

menos que se indique lo contrario.

62

3.1 Lema. Dado un conjunto A existe un ordinal α tal que α � A.

Demostracion. Consideremos R = {R ⊆ A×A / 〈domR,R〉 buen orden} el cual es conjunto.

Luego, por el Axioma de Reemplazamiento, H = {type(domR,R) / R ∈ R} es un conjunto de

ordinales. Veamos que H = {α / α � A}. Si α ∈ H, existe R ∈ R tal que α = type(domR,R),

por lo cual α ≈ domR � A. Para la otra contencion, si α � A, existe f : α → A inyectiva,

de donde podemos definir una relacion R ⊆ f [α] × f [α] de modo que f [α] = domR este bien

ordenada por R, por lo cual α = type(domR,R) ∈ H. Por lo tanto, como H = {α / α � A} es un

conjunto, existe un α /∈ H.

3.2 Definicion (Numero de Hartogs). Dado un conjuntoA, definamos por h(A) = mın {α / α � A}el numero de Hartogs de A.

3.3 Lema. (a) h(A) = {α / α � A}.

(b) h(A) es un cardinal.

(c) Si A tiene buen orden, entonces h(A) = |A|+.

(d) A es finito si y solo si h(A) es finito.

(e) El Axioma del Infinito equivale a la existencia de un conjunto infinito.

Demostracion. (a) Si α < h(A) es inmediato que α � A de la Definicion 3.2. Por otra parte, si

α � A y h(A) ≤ α, entonces h(A) � A, lo cual es falso.

(b) Si α < h(A) entonces α � A por (a). Luego, como h(A) � α se sigue que α 6≈ h(A).

(c) Como |A| � A entonces |A| < h(A), por lo cual |A|+ ≤ h(A). Por otra parte, como |A|+ � A

entonces h(A) ≤ |A|+.

(d) Si A es finito entonces h(A) = |A|+ es finito. Si A es infinito entonces n � A para todo n

natural, por lo cual ω es conjunto y ω ≤ h(A).

(e) Si A es infinito entonces h(A) es un cardinal infinito por (d). Luego, de 1.12, el Axioma del

Infinito se satisface.

63

El resultado anterior nos dice que el numero de Hartogs es la aproximacion al cardinal suce-

sor de un conjunto cuando estamos trabajando sin AC4. Dicha “aproximacion” es interesante para

probar equivalencias de afirmaciones con AC y para enunciar consecuencias de AC en el contex-

to de ZF. De hecho, con la nocion de Numero de Hartogs, es posible demostrar rapidamente que

GCH⇒AC.

A continuacion damos ejemplos de afirmaciones sobre equipotencia que son equivalentes a AC.

Antes, el siguiente preliminar.

3.4 Lema (ZF−−P−Inf ). Sean A y B disjuntos, donde B tiene buen orden. Si A × B � A ∪ Bentonces A � B o B � A.

Demostracion. Supongamos que existe h : A×B → A ∪B uno a uno. Consideremos dos casos.

Si ∃a∈A∀y∈B(h(a, y) ∈ A) entonces la funcion ha : B → A dada por ha(y) = h(a, y) es uno a

uno, por lo cual B � A. Supongamos por el contrario que ∀x∈A∃y∈B(h(x, y) ∈ B). Dado que B

tiene buen orden, definamos g : A→ B tal que, para todo x ∈ A, g(x) sea el menor y ∈ B tal que

h(x, y) ∈ B. Luego, la funcion f : A → B, dada por f(x) = h(x, g(x)), es inyectiva, por lo cual

A � B.

3.5 Teorema. Las siguientes afirmaciones son equivalentes.

(i) AC.

(ii) ∀A(A infinito⇒ A×A ≈ A).

(iii) ∀A,B(A � B ∨B � A).

(iv) ∀A(A ≺ h(A)).

Demostracion. Veamos que (i) implica (iii) y (iv). Bajo AC todo conjunto tiene buen orden, por

lo cual |A| esta definido para todo conjunto A. Luego |A| ≤ |B| ∨ |B| ≤ |A| y, por 2.4, |A×A| =|A| · |A| = |A| para todo A infinito.

Veamos ahora que, tanto (iii) como (iv), implican (ii). Si aplicamos (iii) a A y a h(A) tenemos

que A ≺ h(A) o h(A) � A, pero el ultimo caso no es cierto, por lo cual tenemos (ii). Si ahora

suponemos (iv) y A y B son conjuntos disjuntos donde A es infinito, B tiene buen orden y |B| =

h(A), entonces

A×B � (A×A) ∪ (A×B) ∪ (B ×A) ∪ (B ×B) = (A ∪B)× (A ∪B) ≈ A ∪B4De hecho, en los textos que definen cardinales para cualquier conjunto, el numero de Hartogs define al sucesor de

dicho cardinal.

64

de donde, por 3.4 obtenemos A � B o B � A pero, como B ≈ h(A) entonces el segundo caso no

es posible, por lo cual A � h(A).

Veamos finalmente que (ii) implica (i). Dado un conjunto A se tiene A ≺ h(A) pero, como h(A)

tiene buen orden, entonces A tambien.

Bajo AC se puede probar que, si existe f : Y → X sobreyectiva, entonces X � Y (Sin AC

solo es posible demostrar esto cuando Y tiene un buen orden). Consecuencias del mismo axioma es

que, dado que todo conjunto se puede ordenar bien, entonces κ+ ≤ 2κ para todo cardinal infinito κ.

Si precindimos de AC, no podemos demostrar nisiquiera que ω1 � P(ω), pero podemos demostrar

que existe una funcion sobreyectiva f : P(ω)→ ω1. Por lo tanto, el enunciado anterior “aproxima”

un resultado de ZFC en vista de que son equivalentes bajo el axioma de Eleccion. Procedemos

entonces a probar el resultado mencionado.

3.6 Teorema. Dado A existe una funcion sobreyectiva F : P(A×A)→ h(A).

Demostracion. Dado R ⊆ A×A definamos

F (R) ={

type(domR,R) si 〈domR,R〉 es b.o.0 en otro caso.

Es claro que F (R) � A. Por lo tanto F : P(A×A)→ h(A). Ahora, si α < h(A) entonces α � A,

por lo cual existe una relacion R ⊆ A × A tal que 〈domR,R〉 es un buen orden isomorfo a α, por

lo cual F (R) = α. Ası, F es sobreyectiva.

3.7 Corolario. Dado un cardinal infinito κ, existe una funcion sobreyectiva f : P(κ)→ κ+

Demostracion. Del Teorema 3.6 existe una funcionF : P(κ×κ)→ h(κ). Como P(κ×κ) ≈ P(κ)

y h(κ) = κ+ entonces existe la funcion f del enunciado.

2.15 Ejercicio. Pruebe que:

(a) h(A) > 0.

(b) Si h(A) = ω entonces A es infinito, pero no es Dedekind infinito.

(c) Bajo AC, ∀A(h(A) 6= ω).

(d) Pruebe que h(A) � P(P(P(A))). Sugerencia: Dado α < h(A) considere

Pα = {N ⊆ P(A) / 〈N ,⊆〉 es b.o. ∧ type(N ,⊆) = α} .

2.16 Ejercicio. Sea A un conjunto. Pruebe que:

1. Existe s(A) = sup {α / α � A}.2. Si A es finito entonces s(A) = |A|.3. Si A es infinito pruebe que s(A) = h(A).

En algunos libros definen al numero de Hartogs de A por s(A).

65

2.4. Cofinalidad

Fijamos para esta seccion un orden lineal 〈A,<〉 tal que |A| esta bien definido. En este orden

lineal identifiquemos dos tipos de sucesiones:

(i) Si A tiene maximo, sucesiones que contengan a este elemento.

(ii) Si A no tiene maximo, sucesiones que no esten acotadas.

Ambos tipos de sucesiones tienen en comun que, dado cualquier x ∈ A, es posible encontrar un

elemento de la sucesion mayor o igual que x. El concepto de cofinalidad para el orden 〈A,<〉esta relacionado con encontrar la sucesion de menor longitud que cumpla dicha condicion. Para el

tipo (i) es claro que la menor longitud de una sucesion tal es 1, mientras que en el tipo (ii), siA 6= ∅,

la sucesion es infinita.

La hipotesis de que |A| esta definido, es decir, que A tiene un buen orden (el cual no necesaria-

mente es <), es para poder definir una funcion de eleccion F : P(A) \ {0} → A sin necesidad de

utilizar AC (simplemente defina F(B) = mın(B) respecto al buen orden de A). Tambien tenemos

que todo subconjunto de A tiene cardinal (pues heredan el buen orden de A). Los resultados en esta

seccion estan dados en ZF−−P−Inf a menos que se indique lo contrario.

4.1 Definicion. B ⊆ A es cofinal en A si ∀x∈A∃y∈B(x ≤ y). Dado un orden lineal 〈L,<L〉, una

funcion f : L → A es cofinal si ranf es cofinal en A. Dado un conjunto C ⊆ A decimos que C

esta estrictamente acotado en A si existe s ∈ A tal que ∀x∈C(x < s). En esta caso decimos que s

es cota superior estricta de C.

2.17 Ejercicio. (a) A es cofinal en A.

(b) Si A tiene maximo, entonces B es cofinal en A si y solo si max(A) ∈ B.

(c) Si A es no vacıo y no tiene maximo, entonces todo subconjunto cofinal en A es infinito.

(d) B ⊆ A no es cofinal en A si y solo si es estrictamente acotado en A.

(e) Supongamos que C ⊆ B ⊆ A. Si C es cofinal en B y B es cofinal en A, entonces C es cofinal en A.

(f) Si C ⊆ B ⊆ A y C es cofinal en A, entonces B es cofinal en A

4.2 Teorema. Sea g : γ → A una sucesion cofinal y C ⊆ A cofinal en A. Entonces existe B ⊆ C

cofinal en C (y por el Ejercicio 2.17, cofinal en A) tal que 〈B,<〉 esta bien ordenado (con el orden

heredado de A) y type(B,<) ≤ γ.

66

Demostracion. Definamos f : γ → C por recursion transfinita. Si α < γ y f(ξ) esta definido para

todo ξ < α, consideremos dos casos para definir f(α). Si f [α] no es cofinal en C, entonces tiene

una cota superior estricta c ∈ C y, como C es cofinal, existe y ∈ C tal que y ≥ max{c, g(α)}.Definimos f(α) como algun y de ese tipo (f(y) = F({y ∈ C / y ≥ g(α) ∧ ∀ξ<α(f(ξ) < y)})).

Ası, f(α) > f(ξ) para todo ξ < α. Por otra parte, si f [α] es cofinal en C, definamos f(α) como

algun elemento de C mayor o igual que g(α) (f(α) = F({y ∈ C / y ≥ g(α)})).

Consideremos δ = {α < γ / f [α] no cofinal en C} y veamos que es un ordinal. Si ξ < α y α ∈ δ ,

como f [ξ] ⊆ f [α] entonces f [ξ] no es cofinal en C, es decir, ξ ∈ δ.

Luego, δ ≤ γ. Sea h = f � δ y veamos que es estrictamente creciente y cofinal. Es claro de la

definicion de f que h es estrictamente creciente. Ahora, si δ < γ entonces ranh = f [δ] es cofinal

en C o, de lo contrario, δ < δ. Por otra parte, si δ = γ entonces f [α] no es cofinal en C para todo

α < γ, por lo cual h = f . Ahora, si x ∈ C, como g es cofinal, entonces existe α < γ tal que

x ≤ g(α) y, de la definicion de f , x ≤ g(α) ≤ f(α). Por lo tanto, h = f es cofinal.

Sea B = ranh, el cual es cofinal en C y, ademas, como h : δ → B es biyectiva y estrictamente

creciente, entonces es un isomorfismo, lo cual implica que 〈B,<〉 es buen orden y type(B,<) =

δ ≤ γ.

4.3 Corolario. Existe B ⊆ A cofinal en A tal que 〈B,<〉 es un buen orden y type(B,<) ≤ |A|.

Demostracion. Directo al aplicar 4.2 para C = A, γ = |A| y g una biyeccion entre |A| y A.

4.4 Definicion (Cofinalidad). Definimos por

cf(A,<) = mın {type(B,<) / B ⊆ A cofinal y 〈B,<〉 buen orden}

(el cual existe por 1.17) la cofinalidad de 〈A,<〉. Cuando el orden se sobreentiende, simplemente

escribimos cf(A). Si α es un ordinal infinito, lo llamamos regular si cf(α) = α o, de lo contrario, α

es singular.

Lo anterior define la sucesion bien ordenada mas corta que es cofinal en A, esto es, la sucesion

estrictamente creciente mas corta con la cual se puede cubrir a todo el orden A. Mas adelante

veremos que esta nocion de cofinalidad es muy relevante para estimar cardinales.

4.5 Lema. (a) cf(A) ≤ |A|.

(b) cf(α) ≤ α.

(c) Todo ordinal regular es un cardinal.

67

(d) Todo ordinal infinito que no es cardinal es singular.

(e) Existe una funcion g : cf(A)→ A estrictamente creciente y cofinal.

(f) cf(A) = 0 si y solo si A = 0.

(g) Si A tiene maximo entonces cf(A) = 1.

(h) Todo ordinal sucesor tiene cofinalidad 1.

(i) Si A 6= 0 no tiene maximo, entonces cf(A) es un ordinal lımite.

(j) Para todo ordinal lımite, su cofinalidad es un ordinal lımite.

Demostracion. (a) Inmediato de 4.3 y la Definicion 4.4.

(b) cf(α) ≤ |α| ≤ α.

(c) Si α es un ordinal infinito regular, entonces |α| ≤ α = cf(α) ≤ |α|, por lo cual |α| = α.

(d) Contrarrecıproco del anterior.

(e) De 4.4 existe B ⊆ A cofinal en A tal que 〈B,<〉 es buen orden y type(B,<) = cf(A). Luego,

existe un isomorfismo h : cf(A) → B, por lo cual h : cf(A) → A es estrictamente creciente y

cofinal.

(f) 0 es el unico subconjunto de 0 y es cofinal. Por otra parte, si A 6= 0 entonces 0 no es cofinal en

A, por lo cual cf(A) > 0.

(g) Si A tiene maximo, {max(A)} es cofinal en A, por lo cual cf(A) ≤ 1. La igualdad se sigue de

que A es no vacıo.

(h) Inmediato de que todo ordinal sucesor tiene maximo (su predecesor).

(i) Como A es no vacıo entonces cf(A) > 0. Ahora, si cf(A) = β + 1, existe g : β + 1 → A

estrictamente creciente y cofinal. Luego, si x ∈ A existe α < β + 1 tal que x ≤ g(α), pero

g(α) ≤ g(β). Por lo tanto, g(β) es el maximo de A, lo cual contradice la hipotesis.

(j) Inmediato del anterior.

4.6 Corolario. Existe una funcion g : cf(α)→ α normal y cofinal.

68

Demostracion. Si α no es ordinal lımite la construccion de g normal es directa. Veamos el caso en

que α es un ordinal lımite. De 4.5 se sigue que cf(α) es un ordinal lımite y que existe f : cf(α)→ α

estrictamente creciente y cofinal. Luego, sea g : cf(α)→ α tal que

g(β) ={f(β) si β no es lımite,supξ<β(f(ξ)) si β es lımite.

Es claro que g(β) ≤ f(β) para todo β < cf(α). Como g(β) ≤ f(β) < f(β + 1) = g(β + 1)

y g es continua, entonces g es normal. Ahora, si η < α, como f es cofinal, existe β < α tal que

η ≤ f(β) < f(β + 1) = g(β + 1) (β + 1 < cf(α) al ser cf(α) ordinal lımite). Por lo tanto, g es

normal y cofinal.

4.7 Corolario. (a) Si γ < cf(A) entonces, para toda funcion f : γ → A, ranf esta acotado

estrictamente.

(b) El recıproco del anterior es cierto cuando A 6= 0.

(c) Sea α un ordinal lımite. α es regular si y solo si, dado γ < α y f : γ → α, ranf esta acotado.

Demostracion. (a) Si existe una funcion f : γ → A cofinal, entonces del Teorema 4.2 aplicado a

C = A, existe B ⊆ A cofinal en A tal que 〈B,<〉 es buen orden y cf(A) ≤ type(B,<) ≤ γ.

(b) Sabemos que existe g : cf(A) → A cofinal. Luego, si γ ≥ cf(A) y A 6= 0, al tomar cualquier

a ∈ A extendemos g a

f : γ −→ A

α 7−→ f(α) ={g(α) si α < cf(A)a si α ≥ γ

la cual es cofinal.

(c) Si α es regular y γ < α = cf(α), de (a) se sigue que todo f : γ → α esta acotado. Por otra

parte, si α es singular lımite, con γ = cf(α) < α existe f : γ → α cofinal y, por lo tanto, al ser

α ordinal lımite, ranf no es acotado.

4.8 Teorema. Si C ⊆ A es cofinal, entonces cf(C) = cf(A).

Demostracion. Como existe g : cf(A) → A cofinal, del Teorema 4.2 existe B ⊆ C cofinal en C

(y en A) tal que 〈B,<〉 es un buen orden y type(B,<) ≤ cf(A). Como B es cofinal en C entonces

cf(C) ≤ type(B,<), por lo cual cf(C) ≤ cf(A). La otra desigualdad es directa de la definicion de

cofinalidad (4.4).

69

4.9 Teorema. Sea 〈L,<L〉 un orden lineal que no tiene maximo y tal que |L| esta definido. Si

f : A→ L es cofinal y creciente, entonces cf(A,<) = cf(L,<L).

Demostracion. Sea D = ranf el cual es cofinal en L. Dado que |L| esta definido, L tiene un buen

orden (no necesariamente coincide con <L), por lo cual existe g : D → A uno a uno el cual es

inversa a derecha de f : A → D. Veamos que g es estrictamente creciente y cofinal. En efecto,

si x, y ∈ D y x <L y entonces, si g(y) ≤ g(x), se sigue que y = f(g(y)) ≤L f(g(x)) = x lo

cual es absurdo. Por otra parte, si a ∈ A, entonces f(a) ∈ L y, como L no tiene maximo, existe

z ∈ L tal que f(a) <L z. Luego, como f es cofinal, existe b ∈ A tal que z ≤L f(b), por lo cual

f(a) <L f(b). Si g(f(b)) ≤ a entonces f(b) = f(g(f(b))) ≤L f(a), lo cual es falso. Por lo tanto

a < g(f(b)) y ası g es cofinal.

Sea C = rang, el cual es cofinal en A. Como g : D → C es estrictamente creciente y sobreyectiva,

entonces es un isomorfismo entre D y C. Por lo tanto, cf(L) = cf(D) = cf(C) = cf(A) por

4.8.

4.10 Corolario. (a) cf(cf(A)) = cf(A).

(b) cf(A) es un cardinal, y es regular cuando A es no vacıo y no tiene maximo.

(c) (ZF−−P). ω es un cardinal regular.

(d) (ZF−). Si α es un ordinal lımite, entonces cf(ωα) = cf(α).

(e) (ZF−). ωω es un cardinal singular.

Demostracion. (a) Si A tiene maximo, la igualdad es directa de 4.5, pues cf(cf(A)) = cf(1) =

1 = cf(A). Ahora, si A no tiene maximo, como existe g : cf(A)→ A estrictamente creciente y

cofinal, de 4.9 se sigue que cf(cf(A)) = cf(A).

(b) Si A es vacıo o tiene maximo, entonces cf(A) es 0 o 1 respectivamente. Si A es no vacıo y no

tiene maximo, cf(A) es ordinal lımite por 4.5 y es (cardinal) regular por (a).

(c) Es facil ver que los subconjuntos cofinales de ω son precisamente sus subconjuntos infinitos.

Por lo tanto, cf(ω) = ω.

(d) Si α es un ordinal lımite, la funcion g : α → ωα definida por g(ξ) = ωξ es normal y cofinal.

Por lo tanto, se sigue de 4.9 que cf(α) = cf(ωα).

70

(e) Puesto que cf(ωω) = cf(ω) = ω.

Para los fines del texto, solo nos preocupamos por la cofinalidad de ordinales lımite. Es claro

que estas cofinalidades son cardinales regulares. En ZF no se puede probar que existen ordenes

lineales de cofinalidad no contable. Bajo el Axioma de Eleccion se siguen resultados interesantes.

Antes, algunos preliminares.

4.11 Lema (ZFC−−P−Inf ). (a) X � Y si y solo si X = 0 o existe f : Y → X sobreyectiva.

(b) (ZFC−−Inf ). Si ∀i∈I(|Xi| ≤ κ) entonces∣∣⋃

i∈I Xi

∣∣ ≤ |I| · κ.

(c) (ZFC−−Inf ).∣∣⋃

i∈I Xi

∣∣ ≤ |I| · supi∈I{|Xi|}.

(d) (ZFC−−Inf ). Si κ es un cardinal infinito, |I| ≤ κ y ∀i∈I(|xi| ≤ κ), entonces∣∣⋃

i∈I Xi

∣∣ ≤ κ.

Demostracion. (a) Sı g : X → Y es inyectiva, entonces X = 0 o existe una inversa a inversa a

izquierda f : Y → X de g, la cual es sobreyectiva. Para el recıproco, el claro que 0 � Y y, si

f : Y → X es sobre, dado que Y tiene un buen orden existe g : X → Y inversa a derecha de

f , la cual es inyectiva.

(b) Sin perder generalidad, supongamos que Xi 6= 0 para todo i ∈ I . Como |Xi| ≤ κ, por AC

podemos escoger una funcion sobreyectiva fi : κ→ Xi para cada i ∈ I . Luego, la funcion

F : I × κ −→⋃i∈I Xi

(i, α) 7−→ F ((i, α)) = fi(α)

es sobreyectiva, de donde se sigue el resultado.

(c) Inmediato del resultado anterior con κ = supi∈I{|Xi|}.

(d) Inmediato de (b) y de que |I| · κ ≤ κ · κ = κ.

Sin el Axioma de Eleccion no se puede probar que la union de conjuntos contables es con-

table. Por ejemplo, el hecho de que ω1 es la union contable de conjuntos contables no introduce

contradicciones en ZF. Bajo el Axioma de Eleccion, la situacion es muy diferente.

4.12 Teorema (ZFC−). Todo cardinal sucesor es regular. Por lo tanto, todo cardinal singular es

un ordinal lımite.

71

Demostracion. Sea f : κ→ κ+ una funcion arbitraria. Luego, |f(α)| ≤ κ para todo α < κ, por lo

cual |supα<κ{f(α)}| =∣∣⋃

α<κ f(α)∣∣ ≤ κ por 4.11. Por lo tanto, ranf esta acotado en κ+. Luego,

de 4.7 se sigue que κ < cf(κ+), por lo cual κ+ ≤ cf(κ+) ≤ κ+.

Respecto al resultado anterior cabe preguntarse si existe un cardinal lımite regular. Debido a que

no se puede probar la existencia de dicho cardinal en ZFC, definimos lo siguiente.

4.13 Definicion ((ZF−). Cardinal debilmente inaccesible). Un cardinal es debilmente inaccesible

si es cardinal lımite y regular.

Suponer que no existen cardinales debilmente inaccesibles es consistente con ZFC. Sin embar-

go, tenemos el siguiente resultado.

4.14 Teorema (ZF−). Si ωα es debilmente inaccesible, entonces ωα = α.

Demostracion. Puesto que α ≤ ωα = cf(ωα) = cf(α) ≤ α.

4.15 Ejemplo (ZF−). El recıproco del resultado anterior no es cierto. Definamos por recursion

σ0 = ω y σn+1 = ωσn para n < ω. Luego, sea σ = supn<ω σn. Es facil probar que esta sucesion

es estrictamente creciente, por lo cual σ es un cardinal lımite y, por la normalidad de la sucesion de

ωα, ωσ = supn<ω ωσn = supn<ω σn+1 = σ. Pero, como g : ω → σ tal que g(n) = σn es cofinal y

estrictamente creciente, entonces cf(σ) = cf(ω) = ω, lo cual indica que σ es un cardinal singular.

Para concluir esta seccion observemos algunos resultados que relacionan la cofinalidad de un

cardinal con la cantidad de conjuntos cuya union pueda alcanzar dicho cardinal.

4.16 Lema. Sea κ un cardinal infinito. λ ≥ cf(κ) si y solo si existe una sucesion {Xη}η<λ de

subconjuntos de κ tal que ∀η<λ(|Xη| < κ) y⋃η<λXη = κ.

Demostracion. Si λ ≥ cf(α) entonces, de 4.7, existe una funcion f : λ → κ cofinal, por lo

cual Xη = f(η) para η < λ satisface la afirmacion. Para el recıproco, consideremos la siguiente

afirmacion:

ϕ(λ) : existe una sucesion {Xη}η<λ de subconjuntos de κ tal que ∀η<λ(|Xη| < κ) y⋃η<λ

Xη = κ.

Sea λ0 el menor cardinal que satisface ϕ(λ). Como ϕ(cf(κ)) es cierto (por el argumento anterior)

entonces λ0 ≤ cf(κ). Para concluir la demostracion, basta probar que λ0 = cf(κ). Es claro que

λ0 es un cardinal infinito, pues ningun cardinal finito satisface ϕ(λ). Sea {Xη}η<λ0 una sucesion

72

que satisfaga ϕ(λ0) Dado η < λ0 definamos g(η) = type(⋃

ξ<ηXξ

). Es claro que g(η) ≤ κ y,

de hecho, g(η) < κ pues, si fueran iguales, entonces∣∣∣⋃ξ<ηXξ

∣∣∣ = κ y, luego, existen funciones

h :⋃ξ<ηXξ → κ y s : |η| → η biyectivas, de donde {h[Xs(ξ)]}ξ<|η| es una sucesion que satisface

ϕ(|η|), por lo cual λ0 ≤ |η| ≤ η, una contradiccion. De este modo, g : λ0 → κ y es creciente.

Sea β = supη<λ0{g(η)} y definamos la funcion t : κ → λ0 × β como sigue. Dado α < κ =⋃

η<λXη, sea η el menor ordinal < λ0 tal que α ∈ Xη, y sea γ = type(Xη ∩ α). Puesto que

Xη∩α = Sα(Xη,∈), entonces γ < type(Xη) ≤ g(η+1) ≤ β, por lo cual definimos t(α) = (η, γ).

Veamos que t es inyectiva. En efecto, si t(α1) = t(α2) = (η, γ) para α1, α2 ∈ κ, entonces α1, α2 ∈Xη y type(Xη ∩ α1) = type(Xη ∩ α2). Como ambos son segmentos iniciales de Xη, entonces

α1 = α2. Por lo tanto, κ ≤ λ0 · |β|. Consideremos dos casos. Si |β| ≤ λ0 entonces λ0 · |β| =

λ0 ≤ cf(κ) ≤ κ, lo cual implica que λ0 = κ = cf(κ). Por otra parte, si λ0 ≤ |β| entonces

λ0 · |β| = |β| ≤ β ≤ κ, lo cual implica que β = supη<λ0{g(η)} = κ. Luego, g es cofinal y, por

4.7, λ0 ≥ cf(κ).

4.17 Corolario. Sea κ un cardinal infinito. λ < cf(κ) si y solo si, para toda sucesion {Xη}η<λ de

subconjuntos de κ tal que ∀η<λ(|Xη| < κ), se sigue que∣∣∣⋃η<λXη

∣∣∣ < κ.

Demostracion. Es inmediato de 4.16 mas el siguiente detalle. Si λ < cf(κ) entonces, si {Xη}η<λes una sucesion de subconjuntos de κ tal que ∀η<λ(|Xη| < κ) y

∣∣∣⋃η<λXη

∣∣∣ = κ entonces, dada

una biyeccion h :⋃η<λXη → κ, se tiene que

⋃η<λ f [Xη] = κ, lo cual contradice 4.16.

Los resultados anteriores se proponen de una forma mas sencilla con AC, indicando que un

cardinal infinito κ no puede alcanzarse por la union de una sucesion de conjuntos de longitud menor

que cf(κ) cuyos miembros tienen tamano menor que κ.

4.18 Corolario (ZFC−−Inf ). Sea κ un cardinal infinito.

(a) λ < cf(κ) si y solo si, para toda sucesion de conjuntos {Xη}η<λ tal que ∀η<λ(|Xη| < κ), se

tiene que∣∣∣⋃η<λXη

∣∣∣ < κ.

(b) κ es un cardinal regular si y solo si, para todo λ < κ y para toda sucesion de conjuntos

{Xη}η<λ tal que ∀η<λ(|Xη| < κ), se tiene que∣∣∣⋃η<λXη

∣∣∣ < κ.

(c) κ es un cardinal singular si y solo si existe λ < κ y una sucesion de conjuntos {Xη}η<λ tal que

∀η<λ(|Xη| < κ) y∣∣∣⋃η<λXη

∣∣∣ = κ.

Demostracion. (a) Inmediato de 4.17.

73

(b) Si κ es regular y λ < κ = cf(κ), el resultado se sigue de (a). Por otra parte, si κ es singular,

con λ = cf(κ) < κ, por (a) existe una sucesion de conjuntos{Xη}η<λ tal que ∀η<λ(|Xη| < κ)

y∣∣∣⋃η<λXη

∣∣∣ = κ.

(c) Inmediato del anterior.

2.18 Ejercicio. Pruebe que

(a) Si 〈A,<〉 ∼= 〈L,<L〉 entonces cf(A) = cf(L).

(b) cf(A) = mın {γ / ∃f (f : γ → A cofinal)}.

(c) cf(A) = mın {|B| / B ⊆ A cofinal en A}.

(d) Si D ⊆ A y cf(D) 6= cf(A) entonces D es acotado estrictamente.

(e) Si α es regular y B ⊆ α es cofinal, entonces type(B,∈) = α.

2.19 Ejercicio. Sea C ⊆ ON transitivo. Si F : C→ ON es normal y γ ∈ C es un ordinal lımite, entoncescf(F(γ)) = cf(γ). Por lo tanto, si F(γ) es regular, entonces F(γ) = γ.

2.20 Ejercicio. Sea κ un cardinal infinito. Pruebe que κ es regular si y solo si todo subconjunto de κ detamano < κ esta acotado.

2.21 Ejercicio (ZF−). Pruebe que el cardinal σ definido en el Ejemplo 4.15 es el menor ordinal α tal queωα = α.

2.22 Ejercicio (ZF−). Si λ es un cardinal singular, entonces existe una sucesion {κα}α<cf(λ) estrictamentecreciente de cardinales regulares (sucesores) menores que λ tal que supα<cf(λ) κα = λ.

2.23 Ejercicio (ZF−). Si λ es un cardinal singular, entonces existe una sucesion normal {κα}α<cf(λ) cardi-nales infinitos menores que λ tal que κα es regular si α no es lımite, y supα<cf(λ) κα = λ.

2.24 Ejercicio (ZF−). Sea λ un cardinal singular y |X| = λ. Entonces existe una sucesion {Xα}α<cf(λ)creciente respecto a ⊆ tal que

(i) |Xα| es regular si α no es lımite,

(ii) si α es lımite, entonces Xα =⋃ξ<αXα y

(iii)⋃α<cf(λ)Xα = X .

2.25 Ejercicio (ZFC−−Inf ). Sea {Xi}i∈I una sucesion de conjuntos disjuntos y no vacıos. Si |I| es infinitoo existe i ∈ I tal que Xi es infinito, entonces

∣∣⋃i∈I Xi

∣∣ = |I| · supi∈I{|Xi|}.

74

2.5. Exponenciacion Aritmetica y Cardinal Infinita

Dados α y β, no es posible determinar, en ZFC, la solucion para γ de la ecuacion ℵγ = ℵℵβα ,

a menos que se utilize GCH. Sin embargo, a partir de la nocion de cofinalidad y de la aritmetica

infinita de cardinales, es posible hallar cotas y estimaciones de la exponenciacion de cardinales.

Para los fines de esta seccion, razonamos en ZF−, a menos que se indique lo contrario, usual-

mente con aplicaciones de AC. En los resultados que probemos sin AC abusamos del lenguaje

al referirnos a la exponenciacion κλ (la cual no puede definirse, en general, sin AC) como re-

presentante de λκ. Haremos esto solo para abreviar resultados de equipotencia, por ejemplo, si κ

es un cardinal infinito, al escribir κ < κcf(κ) sin AC, en realidad estamos haciendo referencia a

κ ≺ cf(κ)κ, o si copiamos κλ = 2λ hacemos referencia a λκ ≈ P(λ).

5.1 Lema (Konig). Si κ es un cardinal infinito, entonces κ < κcf(κ).

Demostracion. Sea H : κ→ cf(κ)κ arbitaria, y veamos que no es sobreyectiva. Sea g : cf(κ)→ κ

cofinal y f : cf(κ) → κ tal que, para todo α < cf(κ), f(α) = mın (κ \ {Hβ(α) / β < g(α)}).

Veamos que f /∈ ranH . En efecto, si δ < κ entonces existe α < cf(κ) tal que δ < g(α). Luego,

f(α) /∈ {Hβ(α) / β < g(α)}, por lo cual f(α) 6= Hδ(α). Ası, f 6= Hδ para todo δ < κ.

5.2 Lema. Sean κ y λ cardinales, con λ infinito.

(a) 2 ≤ κ ≤ 2λ⇒ κλ = 2λ.

(b) κ > 2λ⇒ κ ≤ κλ ≤ 2κ.

(c) Si κ es infinito y cf(κ) ≤ λ < κ, entonces κ < κλ ≤ 2κ.

(d) Si κ es infinito y λ < cf(κ) entonces λκ =⋃α<κ

λα. Con AC, κλ =(supµ<κ µλ

)· κ (donde

µ varıa para cardinales).

(e) Si κ es infinito y κλ = κ, entonces λ < cf(κ).

(f) Bajo AC, λ < cf(2λ).

Demostracion. (a) Directo de que 2λ ≤ κλ ≤ (2λ)λ.

(b) Pues κ ≤ κλ ≤ (2κ)λ = 2κ·λ = 2κ.

(c) Del Lema 5.1, κ < κcf(κ) ≤ κλ ≤ κκ = 2κ.

75

(d) Es claro que λα ⊆ λκ para todo α < κ. Por otra parte, si f : λ→ κ, como λ < cf(κ) entonces,

de 4.7, ranf esta acotada estrictamente, digamos que por α < κ. Por lo tanto, f ∈ λα para

algun α < λ.

Si suponemos AC, por 4.11 κλ ≤(supα<κ |α|λ

)· κ =

(supµ<κ µλ

)· κ. La otra desigualdad

se sigue de que supµ<κ µλ ≤ κλ y κ ≤ κλ.

(e) Si cf(κ) ≤ λ entonces κ < κcf(λ) ≤ κλ.

(f) Como (2λ)λ = 2λ, el resultado es directo de (e).

Al razonar con GCH la exponenciacion de cardinales queda mejor determinada, como se indica

a continuacion.

5.3 Corolario. Supongamos GCH (el cual implica AC) y sean κ ≥ 2 y λ ≥ ω cardinales.

Entonces

(a) κ ≤ λ⇒ κλ = λ+.

(b) cf(κ) ≤ λ < κ⇒ κλ = κ+.

(c) λ < cf(κ)⇒ κλ = κ.

Demostracion. (a) y (b) se siguen directamente de 5.2(a) y 5.2(c) respectivamente. Supongamos

que λ < cf(κ). Si µ < κ es un cardinal y ν = max{λ, µ} < κ, entonces µλ ≤ νν = 2ν = ν+ ≤ κ.

Por lo tanto, de 5.2 (d) se sigue que κ ≤ κλ =(supµ<κ µλ

)· κ ≤ κ · κ = κ.

5.4 Definicion. Dado un cardinal λ y un conjunto A tal que |A| esta definido, denotamos por

[A]λ = {X ⊆ A / |X| = λ} y [A]<λ = {X ⊆ A / |X| < λ}. Estos conjuntos tambien se suelen

denotar por Pλ(A) y P<λ(A) respectivamente.

5.5 Lema. (a) La Definicion 5.4 es uniforme respecto a � y a ≈ para un cardinal λ fijo.

(b) λ > κ⇒ [A]λ = 0.

(c) Si n ≤ m < ω entonces |[m]n| =(mn

).

(d) Bajo AC, si κ es infinito y λ ≤ κ, entonces |[κ]λ| = κλ.

76

Demostracion. Veamos (d) para el caso λ > 0. λκ ⊆ [λ × κ]λ ≈ [κ]λ � λκ. La contencion se

sigue de que, para toda funcion f , f ≈ domf ; la equipotencia se sigue de (a) y de que 0 < λ ≤ κ

con κ infinito; y � se sigue de que, para cada X ∈ [κ]λ se puede escojer, por AC, una funcion

biyectiva F (X) : λ→ X . Luego, F : [κ]λ → λκ es inyectiva.

5.6 Definicion. Dado un ordinal β, <βA =⋃α<β

αA denota el conjunto de sucesiones en A de

longitud menor que β. Dados los cardinales κ y λ, si <λκ tiene un buen orden, definimos κ<λ =∣∣<λκ∣∣.5.7 Lema (ZFC−). Suponga que |A| 6= 1. Si A es infinito o β es infinito, entonces

∣∣<βA∣∣ =

supµ<β{|A|µ} (donde µ varıa para cardinales).

Demostracion. Los casos cuando β ≤ 1 o A = 0 se siguen trivialmente, ası que supongamos

que |A| ≥ 2 y β ≥ 2. Como A es infinito o β es infinito, se satisfacen las hipotesis del Ejercicio

2.25, por lo cual∣∣<βA∣∣ = |β| · supα<β{|A||α|} = |β| · supµ<β{|A|µ}. Si α < β entonces α <

|A||α| ≤ supµ<β{|A|µ}, por lo cual β ⊆ supµ<β{|A|µ}, es decir, β ≤ supµ<β{|A|µ}. Por lo tanto,∣∣<βA∣∣ = |β| · supµ<β{|A|µ} = supµ<β{|A|µ}.

5.8 Corolario (ZFC−). Sean κ y λ cardinales.

(a) Si κ o λ son infinitos y κ 6= 1, entonces κ<λ = supµ<λ{κλ} (con µ variando sobre cardinales).

(b) Si 2 ≤ κ < λ y λ es infinito, entonces κ<λ = 2<λ.

(c) Si A es infinito y λ ≤ |A|, entonces |[A]<λ| = |A|<λ.

Demostracion. (a) Inmediato de 5.7.

(b) Es claro que 2<λ = supµ<λ{2µ} ≤ supµ<λ{κµ} = κ<λ. Ahora, si µ < λ entonces κ · µ < λ

y κµ ≤ 2κ·µ ≤ 2<λ, por lo cual κ<λ = supµ<λ{κµ} ≤ 2<λ.

(c) De 5.5, 5.7 y el Ejercicio 2.25,

|[A]<λ| =

∣∣∣∣∣∣⋃µ<λ

[A]µ

∣∣∣∣∣∣ = |λ ∩CA|·supµ<λ{|[A]µ|} = |λ ∩CA|·sup

µ<λ{|A|µ} = |λ ∩CA|·|A|<λ

= |A|<λ

La ultima igualdad se sigue de que |A|<λ ≥ λ segun la prueba de 5.7.

77

De los dos resultados anteriores se sigue rapidamente que µ<ω = ω cuando 0 < µ ≤ ω y que

κ<ω = κ cuando κ es un cardinal infinito. El primer caso se puede probar sin necesidad de AC,

segun el siguiente resultado.

5.9 Lema (ZF−−P). ω<ω ≈ ω. Ası, si A es contable tambien lo es A<ω.

Demostracion. Consideremos alguna biyeccion g : ω × ω → ω. Definamos por recursion sobre

n ∈ ω \ {0} la siguiente sucesion de funciones. f1 : 1ω → ω tal que f1(x) = x(0) y, habiendo

definido fn : nω → ω para n ≥ 1, sea fn+1 : n+1ω → ω tal que fn+1(x) = g(fn(x�n), x(n)).

Es facil probar, por induccion, que las funciones fn son biyectivas para n ≥ 1. Luego, definamos

h : (ω \ {0})×ω → ω<ω tal que h(n,m) = f−1n (m). Esta funcion es sobreyectiva, lo cual implica

que ω<ω � (ω \ {0})×ω ≈ ω (para esto no se necesita AC, pues como domh esta bien ordenado,

entonces se puede definir una inversa a derecha de h, la cual sera inyectiva). Esto indica que ω<ω es

contable y, por lo tanto, enumerable, al contener un subconjunto equipotente con ω.

5.10 Definicion (ZFC−). Un ordinal infinito α > ω es un lımite fuerte si ∀λ<κ(2λ < α). α es un

ordinal (fuertemente) inaccesible si es un lımite fuerte y regular.

Es claro que todo lımite fuerte es un cardinal lımite. Por lo tanto, todo ordinal fuertemente in-

accesible es un cardinal y es debilmente inaccesible (los conceptos son equivalentes bajo GCH).

Por lo tanto, no se puede demostrar en ZFC (incluso con GCH) que existe un cardinal fuerte-

mente inaccesible. De hecho, si ZF es consistente, no se puede demostrar que la hipotesis “Existe

un cardinal debilmente (o fuertemente) inaccesible” es consistente con ZFC, puesto que con un

cardinal inaccesible es posible demostrar que la teorıa ZFC con dicha hipotesis prueba su propia

consistencia.

5.11 Definicion ((ZFC−). Funciones especiales con exponenciacion). Definamos la funcion Beth

por recursion:

i0 =ω

iα+1 =2iα

iγ = supα<γ

iα si γ es lımite.

La funcion del continuo define C(κ) = 2κ para cualquier cardinal infinito κ. La funcion de Gimel

esta definida por κג = κcf(κ) para todo cardinal infinito κ.

Las definiciones anteriores son fundamentales para determinar la exponenciacion en ZFC y

clasificar los lımites fuertes. Ver los Ejercicios 2.33, 2.35 y 2.36 para detalles.

78

5.12 Definicion ((ZFC−). Aritmetica infinita de cardinales). Para una sucesion de conjuntos {Hi}i∈Idenotamos por

⊗i∈I Hi =

{x : I →

⋃i∈I Hi / ∀i∈I(x(i) ∈ Hi)

}. A modo de notacion, al escribir

x = 〈xi〉i∈I ∈⊗

i∈I Hi nos referimos a que x(i) = xi ∈ Hi para todo i ∈ I .

Sea {κi}i ∈ I una sucesion de cardinales. Definimos∑

i∈I κi =∣∣⋃

i∈I({i} × κi)∣∣ y∏i∈I κi =∣∣⊗

i∈I κi∣∣.

La definicion anterior representa la uniformidad al tomar la cardinalidad de una union disjunta

y un producto arbitrario, como lo ilustra el siguiente resultado.

5.13 Lema (ZFC−). Sea {Xi}i∈I una sucesion de conjuntos.

(a) Si los conjuntos son disjuntos entre sı, entonces∣∣⋃

i∈I Xi

∣∣ =∑

i∈I |Xi|.

(b)∣∣⊗

i∈I Xi

∣∣ =∏i∈I |Xi|.

Demostracion. Por AC escojamos, para cada i ∈ I , una funcion biyectiva fi : Xi → |Xi|. Para

probar (a) notese que, si los Xi son disjuntos entre sı, entonces F :⋃i∈I Xi →

⋃i∈I({i} × |Xi|),

definida por F (x) = (i, fi(x)) (donde i ∈ I es el unico tal que x ∈ Xi), es una funcion biyectiva,

de donde se sigue que∣∣⋃

i∈I Xi

∣∣ =∣∣⋃

i∈I({i} × |Xi|)∣∣ =

∑i∈I |Xi|. (b) se sigue de que la funcion

G :⊗

i∈I Xi →⊗

i∈I |Xi|, definida por G(〈xi〉i∈I) = 〈fi(xi)〉i∈I , es una biyeccion.

5.14 Lema (ZFC−). (a)∑

i∈I κ = κ · |I| y∏i∈I κ = κ|I|.

(b) λ ·∑

i∈I κi =∑

i∈I λ · κi.

(c) (∏i∈I κi)

λ =∏i∈I κ

λi .

(d)∏i∈I κ

λi = κ∑i∈I λi .

(e) Si ∀i∈I(κi ≤ λi) entonces∑

i∈I κi ≤∑

i∈I λi y∏i∈I κi ≤

∏i∈I λi.

(f) Si {It}t∈T es particion de I , entonces∑

i∈I κi =∑

t∈T∑

i∈It κi y∏i∈I κi =

∏t∈T∏i∈It κi.

Demostracion. De las igualdades y equipotencias que se concluyan en los siguientes incisos, el

respectivo resultado se sigue de tomar cardinales y aplicar el Lema 5.13.

(a) Se sigue directamente de que⋃i∈I({i} × κ) = I × κ y

⊗i∈I κ = Iκ.

(b) λ ·∑

i∈I κi =∣∣λ×⋃i∈I({i} × κi)

∣∣ =∣∣⋃

i∈I(λ× ({i} × κi))∣∣ =

∑i∈I λ · κi.

79

(c) Definamos la biyeccion G : λ⊗

i∈I κi →⊗

i∈Iλκi como sigue: dado f : λ →

⊗i∈I κi

denotemos por f(α) = 〈αi〉i∈I para cada α < λ. Luego, dado i ∈ I , sea gi : λ → κi definida

por gi(α) = αi y definamos G(f) = 〈gi〉i∈I .

(d) Definamos la biyeccion H :⊗

i∈Iλiκ →

⋃i∈I({i}×λi)κ como sigue: dado f = 〈fi〉i∈I ∈⊗

i∈Iλiκ (donde fi : λi → κ para cada i ∈ I), definamos H(f) = Hf :

⋃i∈I({i} × λi)→ κ

tal que Hf (i, ξ) = fi(ξ) para todo i ∈ I y ξ < λi.

(e) Se sigue directamente de que⋃i∈I({i} × κi) ⊆

⋃i∈I({i} × λi) y

⊗i∈I κi ⊆

⊗i∈I λi.

(f) Claramente,⋃i∈I({i} × κi) =

⋃t∈T⋃i∈It({i} × κi). Para el producto, sea F :

⊗i∈I κi →⊗

t∈T⊗

i∈It κi tal que, dado x = 〈xi〉 ∈⊗

i∈I κi, F (x) = 〈〈xi〉i∈It〉t∈T . Dicha F es biyecti-

va.

Los siguientes son resultados importantes para determinar facilmente un cardinal al operar con

aritmetica infinita.

5.15 Lema (ZFC−). Sea {κi}i∈I una sucesion de cardinales. Si I es infinito o algun κi es infinito,

entonces∑

i∈I κi = |I| · supi∈I{κi}.

Demostracion. Directo del Ejercicio 2.25.

5.16 Lema (ZFC−). Sea {κi}i∈I una sucesion de cardinales.

(a) Si J = {i ∈ I / κi 6= 0} entonces∑

i∈I κi =∑

i∈J κi.

(b) Si K = {i ∈ I / κi 6= 1} entonces∏i∈I κi =

∏i∈K κi.

Demostracion. (a) se sigue de que⋃i∈I({i}×κi) =

⋃i∈J({i}×κi). Para (b), sea F :

⊗i∈I κi →⊗

i∈K κi tal que F (〈xi〉i∈I) = 〈xi〉i∈K . Claramente esto es una biyeccion.

5.17 Lema (ZFC−). Sea {κi}i∈I una sucesion de cardinales. Si ∀i∈I(κi ≥ 2), entonces∑

i∈I κi ≤∏i∈I κi.

Demostracion. Del Lema 5.16 se sigue que, dado j ∈ I , κj =∏i∈I µi donde µj = κj y µi = 1

cuando i 6= j. Luego, κj =∏i∈I µi ≤

∏i∈I κi por la hipotesis y el Lema 5.14. Por lo tanto,

supi∈I{κi} ≤∏i∈I κi. Por otra parte, |I| < 2|I| =

∏i∈I 2 ≤

∏i∈I κi. Por lo tanto,

∑i∈I κi =∣∣⋃

i∈I({i} × κi)∣∣ ≤ |I| · supi∈I{κi} ≤

∏i∈I κi.

80

5.18 Lema (ZFC−). Sea λ un cardinal infinito y {κα}α<λ una sucesion creciente de cardinales no

nulos. Entonces∏α<λ κα = (supα<λ{κi})λ.

Demostracion. Fijemos κ = supα<λ{κα}. Es claro que∏α<λ κα ≤

∏α<λ κ = κλ. Para la otra

desigualdad, consideremos {Aξ}ξ<α una particion de λ tal que |Aξ| = λ para todo ξ < λ (esto se

consigue del Ejercicio 2.13). Como |Aξ| = λ entoncesAξ no es acotado en λ, por lo cual sup(Aξ) =

λ. Luego, como la sucesion {κα}α<λ es creciente, entonces supα∈Aξ{κα} = κ. Ademas, como

κα ≤∏α∈Aξ κα (pues todo κα es no nulo), entonces κ = supα∈Aξ{κα} ≤

∏α∈Aξ κα. Por lo

tanto, κλ =∏ξ<λ κ ≤

∏ξ<λ

∏α∈Aξ κα =

∏α<λ κα.

5.19 Lema ((ZFC−). Konig). Sean {κi}i∈I y {λi}i∈I sucesiones de cardinales. Si ∀i∈I(κi < λi)

entonces∑

i∈I κi <∏i∈I λi.

Demostracion. Sin perder generalidad, supongamos ∀i∈I(κi > 0). Luego,∑

i∈I κi ≤∑

i∈I λi ≤∏i∈I λi por el Lema 5.17 (pues λi > κi ≥ 1 para todo i ∈ I). Falta probar que dicha desigualdad

es estricta, lo cual se garantiza al ver que, para toda sucesion {Xi}i∈I de subconjuntos de⊗

i∈I λi

tal que |Xi| < λi, se tiene⋃i∈I Xi 6=

⊗i∈I λi (pues esto garantiza que no existe una funcion

sobreyectiva de∑

i∈I κi en⊗

i∈I λi). Dado i ∈ I sea Si = {xi / x ∈ Xi}, el cual cumple que

|Si| ≤ |Xi| < λi de donde, como Si ⊆ λi, existe βi = mın(λi \ Si). Ahora, sea x = 〈βi〉i∈I ∈⊗i∈I λi. Dado i ∈ I , xi = βi /∈ Si, por lo cual x /∈ Xi. Por lo tanto, x /∈

⋃i∈I Xi.

El anterior resultado es, en realidad, una generalizacion de Lema de Koning dado en 5.1. Antes

de mostrar esto, veamos un resultado que relaciona la cofinalidad de un cardinal infinito con una

sumatoria infinita.

5.20 Lema (ZFC−). Sea κ un cardinal infinito. Entonces

(a) λ < cf(κ) si y solo si, para toda sucesion {κα}α<λ de cardinales menores que κ, se tiene∑α<λ κi < κ.

(b) λ ≥ cf(κ) si y solo si existe una sucesion {κα}α<λ de cardinales menores que κ tal que∑α<λ κi = κ.

(c) κ es regular si y solo si, para cualquier cardinal λ < κ, si {κα}α<λ es una sucesion de cardi-

nales menores que κ, entonces∑

α<λ κi < κ.

Demostracion. (b) y (c) son consecuencias directas de (a). Para (a), la direccion ⇒ se sigue

directamente de 4.18. Para la direccion opuesta, sea {Xα}α<λ una sucesion de conjuntos tal que

81

∀α<λ(|Xα| < κ). Luego,∣∣⋃

α<λXα

∣∣ ≤ λ · supα<λ{|Xα|} =∑

α<λ |Xα| < κ, por lo cual, de

4.18, se sigue que λ < cf(κ).

Demos fin a este capıtulo con los siguientes ejemplos en donde aplicamos todos los resultados

mencionados sobre aritmetica infinita de cardinales.

5.21 Ejemplo (ZFC−). (1) Si κ es infinito, entonces κ < 2κ. En efecto, κ =∑

α<κ 1 <∏α<κ 2 =

2κ por 5.19.

(2) Si 2 ≤ κ ≤ λ y λ es infinito, entonces cf(κλ) > λ. Sea {µξ}ξ<λ una sucesion de cardinales

menores que κλ. Luego, de 5.19, se sique que∑

ξ<λ µξ <∏ξ<λ κ

λ = (κλ)λ = κλ. Por lo

tanto, de 5.20 se sigue que cf(κλ) > λ.

(3) Si κ es un cardinal infinito, entonces κ < κcf(κ). De 5.20 existe una sucesion {κα}α<cf(κ)

de cardinales menores que κ tal que∑

α<cf(κ) κα = κ. Luego, por 5.19 se sigue que κ <∏α<cf(κ) κ = κcf(κ).

(4)∑

n<ω n = ω · supn<ω{n} = ω por 5.15.

(5)∏

0<n<ω n = (sup0<n<ω{n})ω = 2ω por 5.18.

(6) Sea λ infinito. Luego 2<λ =∣∣⋃

α<λα2∣∣ =

∑α<λ 2|α| =

∑α<ω 2|α| +

∑ω≤α<λ 2|α| =∑

µ<ω 2µ +∑

ω≤µ<λ∑

α∈{α / |α|=µ} 2µ =∑

µ<ω 2µ +∑

ω≤µ<λ 2µ · | {α / |α| = µ} | =∑µ<ω 2µ +

∑ω≤µ<λ 2µ · µ+ =

∑µ<ω 2µ +

∑ω≤µ<λ 2µ =

∑µ<λ 2µ.

(7) Si κ es infinito, entonces (2<κ)cf(κ) = 2κ. De 5.20 existe una sucesion {κα}α<cf(κ) de cardi-

nales menores que κ tal que∑

α<cf(κ) κα = κ. Luego,

2κ = 2∑α<cf(κ)

κα =∏

α<cf(κ)

2κα ≤∏

α<cf(κ)

2<κ = (2<κ)cf(κ) ≤ (2κ)cf(κ) = 2κ.

2.26 Ejercicio (ZFC−). Pruebe la formula de Hausdorff: Si κ es un cardinal infinito y 0 < λ ≤ κ, entonces(κ+)λ = κλ · κ+

2.27 Ejercicio. Bajo GCH, dado α y β determine γ tal que ℵγ = ℵℵβα (puede considerar casos).

2.28 Ejercicio. Pruebe que, para β fijo, la Definicion 5.7 es uniforme respecto a � y ≈.

2.29 Ejercicio. Halle el cardinal de <βκ, sin usar AC, cuando

(i) β = 0.

(ii) β = 1.

82

(iii) κ = 0 y β ≥ 1.

(iv) κ = 1.

(v) 1 < κ < ω y 1 < β < ω.

(vi) (AC). Si κ ≥ 2, κ o β infinitos, y si β no es un cardinal.

(vii) (AC). Si κ ≥ 2 y β = λ+, donde λ es un cardinal infinito.

2.30 Ejercicio (ZFC−). Sean Iny(A) = {f : A→ A / f inyecitva}, Surj(A) = {f : A→ A / f sobre} yBiy(A) = {f : A→ A / f biyectiva}. Si A es infinito, pruebe que el cardinal de estos conjuntos es 2|A|.

2.31 Ejercicio (ZF−−P). (a) Pruebe que {α / α contable} es transitivo.

(b) Si {An}n<ω es una sucesion de conjuntos contables y {fn}n<ω es una sucesion de funciones sobreyec-tivas fn : ω → An, entonces

⋃n<ω An es contable. Sugerencia: Construya una funcion sobreyectiva

F : ω × ω →⋃n<ω An como en 4.11.

(c) Construya, por recursion, funciones sobreyectivas fα : ω → α para α > 0 contable. Sugerencia: Para elcaso en que α es lımite, se tiene que cf(α) = ω, por lo cual existe una sucesion {γn}n<ω estrictamentecreciente y cofinal en α. Utilize el inciso anterior para construir fα.

(d) Si α y β son contables, entonces αβ es contable. Sugerencia: Por induccion sobre β. En el paso lımite,considere la cofinalidad de β y razone como en el inciso anterior.

2.32 Ejercicio (ZFC−). Demuestre que:

1. Todo ordinal lımite fuerte es un cardinal lımite.

2. Todo cardinal fuertemente inaccesible es debilmente inaccesible.

3. Un cardinal κ es un lımite fuerte si y solo si κ > ω y ∀µ,ν<κ(µν < κ).

4. Si λ es un lımite fuerte, entonces 2<λ = λ.

5. GCH implica que, para todo λ infinito, entonces 2<λ = λ.

6. GCH implica que, para todo λ regular, λ<λ = λ.

7. GCH implica que debilmente inaccesible equivale a fuertemente inaccesible.

2.33 Ejercicio (ZFC−). Pruebe que:

(a) La funcion i es normal.

(b) κ es un lımite fuerte si y solo si κ = iγ para algun ordinal lımite γ.

(c) cf(iγ) = cf(γ) para todo γ ordinal lımite.

(d) Si iγ es fuertemente inaccesible, entonces iγ = γ.

(e) Dado α existe β > α tal que iβ = β y cf(β) = ω.

(f) Si α no es lımite, entonces α < iα.

(g) {κ / κ lımite fuerte} es clase propia.

(h) GCH implica que iα = ωα para todo α.

2.34 Ejercicio (ZFC−). Hallar los siguientes cardinales.

(a)∑n<ω ωn.

83

(b)∏n<ω ωn.

(c)∏α<ω+ω ωα.

(d)∏α<ω1+ω

ωα.

2.35 Ejercicio (ZFC−). Una sucesion de ordinales {αξ}ξ<γ con γ lımite es eventualmente constante siexiste β < γ a partir del cual la sucesion toma un valor constante.

(a) Si κ ≥ 2 y λ es infinito, entonces κλ = (κ<λ)cf(λ).

(b) Si κ ≥ 2, λ es cardinal singular y la sucesion de cardinales {κν}ν<λ es eventualmente constante y tomael calor constante µ, entonces κλ = µ = κ<λ.

(c) Si κ ≥ 2, λ es un cardinal lımite y la sucesion {κν}ν<λ no es eventualmente constante, entoncescf(κ<λ) = cf(λ) y κλ = κ<λג .

(d) Si λ es regular, entonces 2λ = .λג

(e) Si λ es lımite fuerte, entonces 2λ = .λג

(f) Si κ es un cardinal lımite y λ ≥ cf(κ), entonces κλ = (supµ<κ{µλ})cf(κ)

2.36 Ejercicio (ZFC−). La exponenciacion de cardinales esta determinada por la funcion del continuo y lafuncion de Gimel. Sean κ y λ cardinales infinitos.

(a) Si existe µ < κ tal que µλ ≥ κ, entonces κλ = µλ.

(b) Si λ < κ y ∀µ<κ(µλ < κ) entonces

(i) λ < cf(κ)⇒ κλ = κ.

(ii) cf(κ) ≤ λ⇒ κλ = .κג

(c) κλ es igual a 2λ, κ o a µג para algun µ ≤ κ tal que cf(µ) ≤ λ < µ. Sugerencia: El primer caso surge alsuponer λ ≥ κ, el segundo de suponer las hipotesis de (b) y (i). Las hipotesis de (b) y (ii) implican queκλ = κג ası que resta asumir la hipotesis de (a). En este caso, considere a µ como el mınimo tal queµλ ≥ κ.

2.37 Ejercicio (ZFC−).

Construir las inversas de las funciones biyectivas definidas en la prueba de 5.14, y verificar que efectivamentelo son (inversas a izquierda y a derecha).

2.38 Ejercicio (ZFC−). Si κ ≥ 2 y λ es infinito, pruebe que κ<λ =∑µ<λ κ

µ (donde µ varıa sobrecardinales).

2.39 Ejercicio (ZFC−). Pruebe que

(a) Si κ es cardinal lımite y 0 < λ < cf(κ), entonces κλ =∑α<κ |α|λ.

(b) ωω1ω = ωωω · 2ω1 .

(c) Si α < ω1 entonces ωω1α = ωωα · 2ω1 .

(d) Si α < ω2 entonces ωω2α = ωω2

α · 2ω2 .

(e) Si κ es debilmente inaccesible, entonces κ<κ = 2<κ.

(f) Si κ es fuertemente inaccesible, entonces κ<κ = κ.

84

Capıtulo 3

Buena Fundacion

En el estudio de los dos capıtulos previos hemos utilizado todos los axiomas de ZFC− para

generar todos los fundamentos de la matematica. El hecho de no haber considerado el Axioma de

Regularidad (ZF2, ver la definicion 1.1.1) ni ver su aplicacion lleva a cuestionar si este tiene alguna

relevancia para generar resultados fuertes en matematicas. Es verdad que este axioma no implica

resultados que se apliquen directamente a otras areas de las matematicas, mas permite dar una es-

tructura fuerte al universo de los conjuntos y, por lo tanto, facilita muchos razonamientos en Teorıa

de Conjuntos.

En este capıtulo nos centramos en el estudio de la estructura global del universo. El la seccion

3.1 definimos la clase WF de “conjuntos bien fundados”, la cual se puede clasificar por niveles o

rangos como una jerarquıa acumulativa y veremos por que toda la matematica toma lugar dentro

de la clase WF. En la seccion 3.2 definimos las relaciones bien fundadas como una generalizacion

de la relaciones bien ordenadas en el sentido de que poseen principios de Induccion y Recursion, y

veremos tambien que la relacion ∈ es un orden bien fundado en WF. Finalmente, en la seccion 3.3

introducimos el Axioma de Regularidad, probamos que es equivalente al a afirmacion V = WF

y obtendremos resultados importantes en Teorıa de Conjuntos que provienen de este axioma. En la

primera seccion razonamos en ZF−−Inf y en la segunda y la ultima en ZF−−P−Inf , a menos

que se indique lo contrario.

La razon por la cual el Axioma de Regularidad no genera resultados en otras areas de la

matematica es por su equivalencia con V = WF ya que, como toda la matematica tiene lugar

dentro de WF, el hecho de que V = WF no aporta en este caso ningun resultado de avance

teorico dentro de otras areas.

3.1. Jerarquıa Acumulativa

Los resultados de esta seccion se prueban en ZF−−Inf , a menos que se indique lo contrario.

1.1 Definicion (Jerarquıa Acumulativa). Por Recursion Transfinita, definamos la siguiente sucesion

de conjuntos:

V0 = 0

Vα+1 = P(Vα)

Vγ =⋃α<γ

Vα si γ es lımite.

Sea WF =⋃α∈ON Vα la clase de los conjuntos bien fundados (mas adelante explicamos el sentido

de este nombre). En algunos libros denotan R(α) = Vα.

1.2 Lema. (a) Los Vα son transitivos. WF es una clase transitiva.

(b) α < β ⇒ Vα ⊆ Vβ .

(c) α ∈ Vα+1 \ Vα.

(d) ON ⊆WF. Por lo tanto, WF es clase propia.

(e) Si x ∈WF entonces el mınimo ordinal β tal que x ∈ Vβ es un ordinal sucesor.

Demostracion. (a) Por induccion sobre α. Es claro que V0 = 0 es transitivo. Si Vα es transitivo,

entonces Vα+1 = P(Vα) es transitivo, pues las partes de un conjunto transitivo es transitivo.

Si γ es lımite y Vα es transitivo para todo α < γ, entonces Vγ =⋃α<γ Vα es una union

de conjuntos transitivos, la cual es transitiva. Finalmente, WF es transitivo al ser union de

conjuntos transitivos.

(b) Por induccion sobre β.

(c) Por induccion sobre α. Es claro que V1 = P(0) = {0}, por lo cual 0 ∈ V1 = V1 \ V0. Si

α ∈ Vα+1 \ Vα entonces α ⊆ Vα+1 por la transitividad de Vα+1 y, luego, α + 1 ⊆ Vα+1, por

lo cual α + 1 ∈ Vα+2. Ahora, si α + 1 ∈ Vα+1 = P(Vα), entonces α + 1 ⊆ Vα y, como

α ∈ α + 1, se tiene α ∈ Vα, una contradiccion. Para el paso lımite, si γ es un ordinal lımite y

α ∈ Vα+1 \ Vα para todo α < γ, como Vα+1 ⊆ Vγ se sigue que α ∈ Vγ . Por lo tanto, γ ⊆ Vγ ,

de donde γ ∈ Vγ+1. Si γ ∈ Vγ =⋃α<γ Vα entonces existe α < γ tal que γ ∈ Vα y luego,

como Vα es transitivo, entonces α ∈ Vα, una contradiccion.

86

(d) Inmediato del inciso anterior.

(e) Sean x y β como en el enunciado. Es claro que β 6= 0, pues V0 es vacıo. Ahora, si β fuese lımite,

entonces x ∈ Vβ =⋃α<β Vα, por lo cual existe α < β tal que x ∈ Vα, lo cual contradice la

minimalidad de β. Por lo tanto, β es sucesor.

El resultado anterior justifica el nombre de jerarquıa acumulativa en la cual se divide la clase

WF. Puesto que los ordinales se encargan de dividir los niveles, es posible hallar el mınimo nivel

en el cual este un elemento de WF, lo cual le da una jerarquıa o rango dentro de esa clase. Puesto

que ese mınimo nivel siempre es ordinal sucesor, optamos por definir el rango de un elemento de

WF como su predecesor, lo cual se ilustra en la siguiente

1.3 Definicion (Rango en la jerarquıa acumulativa). Dado x ∈ WF definimos el rango de x en

WF como rank(x) = mın {α / x ∈ Vα+1}.

1.4 Lema. Sean x, y ∈WF.

(a) rank(x) + 1 = mın {β / x ∈ Vβ}.

(b) x ⊆ Vrank(x).

(c) α = rank(x) si y solo si x ∈ Vα+1 \ Vα.

(d) rank(α) = α para todo ordinal α.

(e) Vα = {x ∈WF / rank(x) < α}.

(f) x ∈ y⇒ rank(x) < rank(y).

(g) rank(x) = supz∈x{rank(z) + 1}.

Demostracion. (a) Sea δ = rank(x) y β = mın {β / x ∈ Vβ}. Claramente, x ∈ Vδ+1, por lo

cual β ≤ δ + 1. De 1.2 se sigue que β es sucesor, es decir, β = α + 1 para algun α. Luego

α + 1 ≤ δ + 1, por lo cual α ≤ δ y, como x ∈ Vα+1, entonces δ = α por definicion de rank.

Luego, β = δ + 1 = rank(x) + 1.

(b) Directo de que x ∈ Vrank(x)+1 = P(Vrank(x)).

(c) Si α = rank(x) entonces x ∈ Vα+1 y x /∈ Vα por (a). Recıprocamente, si x ∈ Vα+1 \ Vαentonces, si ξ ≤ α entonces Vξ ⊆ Vα, por lo cual x /∈ Vξ. Ası, α + 1 = mın {β / x ∈ Vβ} =

rank(x) + 1, de donde α = rank(x).

(d) Directo del inciso anterior y de 1.2.

87

(e) Si x ∈ Vα entonces rank(x) + 1 ≤ α por (a). Luego, rank(x) < α. Por otra parte, si x ∈WF

y rank(x) < α, entonces x ∈ Vrank(x)+1 ⊆ Vα.

(f) Supongamos que x ∈ y. De (b) se sigue y ⊆ Vrank(y), por lo cual x ∈ Vrank(y) y, de (a),

rank(x) < rank(x) + 1 ≤ rank(y).

(g) Sea δ = supz∈x{rank(z)+1}. Veamos primero que Si z ∈ x entonces rank(z) < rank(x), por

lo cual rank(z)+1 ≤ rank(x). Por lo tanto, δ ≤ rank(x). Supongamos α < rank(x) y veamos

que existe z ∈ x tal que α ≤ rank(z). Supongamos por el contrario que ∀z∈x(rank(z) < α).

Si z ∈ x entonces z ∈ Vrank(z)+1 ⊆ Vα. Por lo tanto, x ⊆ Vα y, luego, x ∈ Vα+1 ⊆ Vrank(x), es

decir, x ∈ Vrank(x), lo cual contradice (c).

Dado que existe z ∈ x tal que rank(z) ≥ α, entonces α < rank(z) + 1 ≤ δ, esto para todo

α < rank(x). Por lo tanto, rank(x) ⊆ δ, es decir, rank(x) ≤ δ.

El resultado anterior indica que el rango de un conjunto esta definido mediante el rango de sus

elementos, lo cual es una idea totalmente recursiva. En la seccion 3.2 veremos que esta forma re-

cursiva del rango esta asociada a la buena fundacion de la relacion ∈ en la clase WF.

Ahora vamos a ver que tan proximo esta WF de V. La primera idea la da el siguiente resultado.

1.5 Lema. x ∈WF si y solo si x ⊆WF.

Demostracion. Una direccion es directa de la transitividad de WF. Supongamos que x ⊆ WF.

Sea α = supy∈x{rank(z) + 1}, el cual esta bien definido. Esto implica que x ⊆ Vα, por lo cual

x ∈ Vα+1.

El siguiente resultado ilustra que las operaciones tıpicas de Teorıa de Conjuntos son cerradas

dentro de WF. A la vez tenemos que WF contiene a los ordinales y, bajo el Axioma del Infinito,

R, C y sus operaciones aritmeticas estan definidas en WF. Esta es la primera evidencia fuerte de

que toda la matematica toma lugar dentro de WF.

1.6 Lema. Sean x, y ∈WF. Entonces:

(a) {x} ∈WF y rank({x}) = rank(x) + 1.

(b) P(x) ∈WF y rank (P(x)) = rank(x) + 1.

(c)⋃x ∈WF y rank (

⋃x) ≤ rank(x).

88

(d) {x, y} ∈WF y rank({x, y}) = max{rank(x), rank(y)}+ 1.

(e) x ∪ y ∈WF y rank(x ∪ y) = max{rank(x), rank(y)}.

(f) (x, y) ∈WF y rank((x, y)) = max{rank(x), rank(y)}+ 2.

(g) x× y ∈WF y rank(x× y) ≤ max{rank(x), rank(y)}+ 2.

(h) yx ∈WF y rank(yx) ≤ max{rank(x), rank(y)}+ 3.

(i) (ZF−). ω,Z,Q,R,C, sus operaciones aritmeticas y sus relaciones de desigualdad pertenecen

a WF y sus rangos son menores que ω + ω.

Demostracion. (a) Como {x} ⊆ WF del Lema 1.5 se sigue que {x} ∈ WF. Luego, del Lema

1.4 se tiene rank({x}) = supz∈{x}{rank(z) + 1} = rank(x) + 1.

(b) Si z ⊆ x entonces, como x ⊆ Vrank(x) entonces z ⊆ Vrank(x), es decir, z ∈ Vrank(x)+1. Ası,

P(x) ⊆ Vrank(x)+1, por lo cual P(x) ∈ Vrank(x)+2. Luego, P(x) ∈ WF y rank (P(x)) ≤rank(x) + 1. Por otra parte, como x ∈ P(x) entonces rank(x) < rank (P(x)), de donde se

sigue que rank (P(x)) = rank(x) + 1.

(c) Si z ∈ x entonces z ∈WF por la transitividad de WF y, luego, z ⊆WF. Por lo tanto,⋃x ⊆

WF y, por Lema 1.5,⋃x ∈ WF. Ahora, si w ∈

⋃x entonces existe z ∈ x tal que w ∈ z.

Luego, rank(w) + 1 ≤ rank(z) < rank(x). Por lo tanto, rank (⋃x) = supw∈⋃

x{rank(w) +

1} ≤ rank(x).

(d) Como {x, y} ⊆WF entonces {x, y} ∈WF. Luego, rank({x, y}) = supz∈{x,y}{rank(z) +

1} = max{rank(x), rank(y)}+ 1.

(e) Sin perder generalidad, supongamos que rank(x) ≤ rank(y). Luego, x ∪ y ⊆ Vrank(y), por

lo cual x ∪ y ∈ Vrank(y)+1 y rank(x ∪ y) ≤ rank(y). Ahora, como y ⊆ x ∪ y se sigue que

rank(y) ≤ rank(x ∪ y), por lo cual rank(x ∪ y) = rank(y) = max{rank(x), rank(y)}.

(f) Recordemos que (x, y) = {{x}{x, y}} el cual pertenece a WF por (d). Luego, rank((x, y)) =

max{rank({x}), rank({x, y})}+1 = max{rank(x)+1,max{rank(x), rank(y)}+1}+1 =

max{rank(x), rank(y)}+ 2.

(g) Del inciso anterior y la transitividad de WF se sigue que x × y ⊆ WF. Luego, por 1.5,

x × y ∈ WF y rank(x × y) = sup(z,w)∈x×y{rank((w, z)) + 1}. Como rank((z, w)) +

89

1 = max{rank(z), rank(w)} + 3 < max{rank(x), rank(y)} + 3 para (z, w) ∈ x × y,

entonces rank((z, w)) + 1 ≤ max{rank(x), rank(y)} + 2. Por lo tanto, rank(x × y) ≤max{rank(x), rank(y)}+ 2.

(h) Como yx ⊆ P(y × x), del inciso anterior se sigue que yx ⊆WF y, por lo tanto, yx ∈WF.

Luego, rank(yx) ≤ rank(P(x× y)) ≤ max{rank(x), rank(y)}+ 3.

(i) Es claro que ω ∈ Vω+1 ⊆WF y que rank(ω) = ω. Recordemos como los conjuntos Z,Q,Ry C estan definidos. Con la siguiente relacion de equivalencia en ω × ω

(n,m) ∼− (k, l)⇔ n+ l = k +m, n,m, k, l ∈ ω

se define Z = (ω×ω)/ ∼−⊆ P(ω×ω). De (g) y (f) se sigue que ω×ω ∈WF y rank(ω×ω) =

supn,m∈ω{rank((n,m)) + 1} = ω, por lo cual P(ω×ω) ∈WF y rank(P(ω×ω)) = ω+ 1.

Por lo tanto, como Z ⊆ P(ω × ω) ⊆WF, entonces Z ∈WF y rank(Z) ≤ ω + 1 (de hecho,

se cumple la igualdad, ver Ejercicio 3.3).

Con la siguiente relacion de equivalencia en Z× (Z \ {0Z})

(a, b) ∼÷ (c, d)⇔ a · d = b · c a, b, c, d ∈ Z, b, d 6= 0Z

se defineQ = (Z× (Z \ {0Z}))/ ∼÷. ComoQ ⊆ P(Z× (Z \ {0Z})) y este ultimo pertenece a

WF y rank(Z× (Z\{0Z})) ≤ ω+ 3 (de hecho son iguales), entoncesQ ∈WF y rank(Q) ≤ω + 4 (de hecho, se cumple la igualdad, ver Ejercicio 3.3).

Los numeros reales estan definidos por cortaduras de Dedekind, es decir, x ∈ R si y solo si

x ( Q es una seccion no vacıa (bajo el orden de Q) que tiene cota superior. Ası, R ⊆ P(Q),

por lo cual R ∈WF y rank(R) ≤ ω + 5 (de hecho, se cumple la igualdad, ver Ejercicio 3.3).

Se defineC = R×R (asociando la primera componente a la parte real, y la segunda componente

a la parte imaginaria), por lo cual C ∈WF y rank(C) ≤ ω+ 7 (de hecho, es una igualdad, ver

Ejercicio 3.3).

Finalmente, si X ∈ {ω,Z,Q,R,C}, entonces sus operaciones aritmeticas y su desigualdad son

subconjuntos de (X× X)× X o de X× X, por lo cual estan en WF y su rango es ≤ ω + 11.

Con el siguiente resultado se tiene que, efetivamente, toda la matematica tiene lugar dentro de

WF, pues cualquier tipo de estructura definida en matematicas es isomorfa a algun conjunto en

WF.

90

1.7 Teorema (ZFC−). Todo espacio vectorial, topologico, grupo, anillo, campo, etc. es isomorfo

(u homeomorfo) a un espacio del mismo tipo que pertenece a WF.

Demostracion. Para cualquier tipo de espacio el metodo de prueba es similar, tomemos por ejemplo

un grupo. Sea 〈G, ∗〉 un grupo y sea κ = |G|. Luego, existe f : κ → G biyectiva. Definamos la

operacion ? : κ× κ→ κ como α ? β = f−1(f(α) ∗ f(β)). Es claro que κ, bajo la operacion ?, es

un grupo isomorfo a 〈G, ∗〉, y κ ∈WF.

Al obtener evidencia suficiente de que la clase WF basta para hacer matematicas, ¿Se puede

probar (en ZFC−) que V = WF? La respuesta a esta pregunta es negativa, pues se tiene que esa

afirmacion es independiente de ZFC−. Sin embargo, los dos resultados previos son suficientes para

aceptar la afirmacion V = WF como axioma de la Teorıa de Conjuntos y, en realidad, este corre-

sponde al Axioma de Regularidad (ZF2), como veremos en la seccion 3.3.

Para concluir esta seccion, veamos como es la cardinalidad de cada nivel de la jerarquıa acumu-

lativa.

1.8 Teorema. (a) Vn es finito para todo n ∈ ω.

(b) (ZF−). |Vω| = ω

(c) (ZFC−). |Vω+α| = iα.

(d) (ZFC−). Si α ≥ ω · ω entonces |Vα| = iα.

Demostracion. (a) Defina por recursion la siguiente sucesion de numeros naturales:

a0 = 0an+1 = 2an n ∈ ω.

Construyamos, por Recursion, funciones biyectivas fn : an → Vn para cada n ∈ ω. f0 = 0;

al haber definido fn : an → Vn biyectiva, tenemos que g : P(an) → Vn+1, definida por

g(X) = fn[X], es biyectiva. Sea h : 2an → P(an) la funcion que resulta al definir h(m) como

el conjunto de exponentes de la expansion binaria de m, es decir, h(m) es el unico subconjunto

de an tal que m =∑

k∈h(m) 2k. Es claro que h es una biyeccion, por lo cual podemos definir

fn+1 = g ◦ h. Por lo tanto, |Vn| = an.

(b) Del inciso anterior consideremos las funciones biyectivas fn : an → Vn. Sea F : ω × ω → Vω

definida por

F (n,m) ={fn(m) si m < an,0 si m ≥ an.

91

Es claro que esta funcion es sobreyectiva, lo cual implica que Vω es contable. Por otra parte,

como ω ⊆ Vω (pues rank(ω) = ω), entonces Vω ≈ ω. Para otra prueba, ver el Ejemplo 3.12.

(c) Por induccion sobre α: para α = 0 se obtiene del inciso anterior que |Vω| = ω = i0.. Si

|Vω+α| = iα entonces |Vω+α+1| = |P(Vω+α)| = 2iα = iα+1. Ahora, para el paso lımite, sea

γ un ordinal lımite tal que |Vω + α| = iα para todo α < γ. Luego,

|Vω+γ | ≤ |γ| · supα<γ{|Vω+α|} = |γ| · sup

α<γ{iγ} = |γ| · iγ = iγ .

Por otra parte, como Vω+α ⊆ Vω+γ entonces iα ≤ |Vγ |, por lo cual iγ = supα<γ{iα} ≤ |Vγ |.

(d) Si α ≥ ω · ω entonces existe δ tal que α = ω · ω + δ. Luego, ω + α = ω + ω · ω + δ =

ω · (1 + ω) + δ = ω · ω + δ = α, por lo cual |Vα| = |Vω+α| = iα.

3.1 Ejercicio. Pruebe que

(a) Vα =⋃ξ<αP(Vξ).

(b) Si x, y ∈WF y x ⊆ y, entonces rank(x) ≤ rank(y).

(c) Si x ∈WF entonces rank(x) = mın {α / x ⊆ Vα}.

(d) Vα ∩ON = α para todo α ∈ ON.

3.2 Ejercicio. Si A es una clase tal que ∀x(x ⊆ A⇒ x ∈ A), entonces WF ⊆ A.

3.3 Ejercicio. Sean x, y ∈WF. Probar los resultados de (e) en adelante en ZF−.

(a) Si rank(x) no es sucesor, entonces rank (⋃x) = rank(x) y rank(x× x) = rank(x).

(b) Si rank(x) es sucesor, entonces rank (⋃x) es el predecesor de rank(x) y rank(x× x) = rank(x) + 2.

(c) Si rank(x) ≤ rank(y), entonces rank(x× y) = rank(y × x) = rank(y × y).

(d) Si n ∈ ω entonces rank(n) = n.

(e) Todo numero entero tiene rango ω.

(f) rank(Z) = ω + 1.

(g) Todo numero racional tiene rango ω + 3.

(h) rank(Q) = ω + 4.

(i) Todo numero real tiene rango ω + 4.

(j) rank(R) = ω + 5.

(k) Todo numero complejo tiene rango ω + 6.

(l) rank(C) = ω + 7.

3.4 Ejercicio (ZFC−). Sea κ es un cardinal no contable. Pruebe que

1. |Vκ| = iκ.

2. Si κ es fuertemente inaccesible, entonces |Vκ| = κ.

92

3.2. Relaciones Bien Fundadas

En esta seccion razonamos en ZF−−P−inf , a menos que se indique lo contrario. Del Capıtulo

1 sabemos que la estructura de los ordinales satisface un principio de buen orden, un principio de

Induccion y un principio de Recursion. Debido a que todo conjunto bien ordenado es isomorfo a

un ordinal, estos tres principios tambien se cumplen para buenos ordenes. De este modo cabe pre-

guntarse, ¿que tan general debe ser un orden para una clase, de modo que se cumplan los principios

de “buen orden”, Induccion y Recursion? escribimos “buen orden” entre comillas para denotar que

no buscamos en la relacion propiedades de orden como reflexividad, transitividad, orden lineal, etc.

sino que se cumpla una condicion de “minimalidad”, es decir, cambiar la propiedad del buen orden

por “todo subconjunto no vacıo posee un elemento minimal”, lo cual se puede definir sin importar

si la relacion tiene un orden o no. A este tipo de relaciones las llamaremos bien fundadas, como lo

especificamos a continuacion, junto con generalizaciones de las definiciones de segmento inicial y

seccion para ordenes parciales.

2.1 Definicion (Relacion bien fundada). Sea A una clase y R una relacion.

(1) Para a ∈ A, denotemos por Sa(A,R) = {x ∈ A / xRa} el segmento inicial de a. Cuando R

y A se sobreentienden, solo escribimos Sa.

(2) Una clase B es cerrada en 〈A,R〉 sii ∀x∈B∀y∈A(yRx⇒ y ∈ B). Esto es equivalente a

∀x∈B(Sx ⊆ B).

(3) R es conjuntista en A si Sa(A,R) es conjunto para todo a ∈ A.

(4) R es una relacion bien fundada en A si ∀x⊆A(x 6= 0⇒ ∃z∈x∀y∈x¬(yRz)). En este caso

decimos que z es R-minimal en x (tomando a R como una especie de relacion “menor que”).

(5) Si R es una relacion conjuntista en A definamos, por recursion

S0a(A,R) = Sa(A,R)Sn+1a (A,R) =

⋃x∈Sna (A,R) Sx(A,R) n ∈ ω.

Llamamos la clausura de a respecto a R a la clase cl(A,R)(a) =⋃n∈ω S

na (A,R). La clausura

de a respecto a R es conjunto bajo el Axioma del Infinito. Cuando A y R se sobreentienden,

bastara con escribir Sna y cl(a) para las definiciones anteriores.

(6) Es claro que la relacion ∈ es conjuntista en V. Si x es un conjunto, llamamos la clausura

transitiva de x a la clase tcl(x) = cl(V,∈)(x).

93

(7) Una clase A se llama bien fundada si ∈ es una relacion bien fundada en A.

Fijemos en adelante una clase A y una relacion R. Vamos a probar en esta seccion que si R

es bien fundada y conjuntista en A de modo que todas sus clausuras son conjuntos (es decir, cl(a)

es conjunto para todo a ∈ A), entonces tiene un principio de induccion y de recursion. De he-

cho, los principios de Buena Fundacion, Induccion y Recursion, son equivalentes para relaciones

conjuntistas donde sus clausuras son conjuntos. Estas consideraciones cobraran mucha mas impor-

tancia cuando hablemos de la relacion ∈ respecto a WF, la cual vamos a definir en esta seccion sin

necesidad del Axioma de Partes. Antes, veamos los siguientes preliminares.

2.2 Lema. Suponga que B ⊆ A

(a) Si R es bien fundada en A, entonces es bien fundada en B.

(b) Sb(B,R) = Sb(A,R) ∩B para todo b ∈ B. Por lo tanto, si R es conjuntista en A entonces

tambien lo es en B.

(c) Si R es conjuntista en A y B es cerrada en A entonces, para todo b ∈ B, Snb (B,R) =

Snb (A,R) para todo n ∈ ω. Por lo tanto, cl(B,R)(b) = cl(A,R)(b)

Demostracion. (a) Si x ⊆ B es no vacıo, entonces esta contenido en A y, como R esta bien

fundada en A, se sigue que x tiene un elemento R-minimal.

(b) Sb(B,R) = {x ∈ B / xRb} = {x ∈ A / x ∈ B ∧ xRb} = Sb(A,R) ∩B.

(c) Veamos por induccion sobre n que Snb (B,R) = Snb (A,R). Como B es cerrada y b ∈ B

entonces Sb(A,R) ⊆ B, por lo cual Sb(B,R) = Sb(A,R) ∩ B = Sb(A,R). Para el paso

inductivo, supongamos que Snb (B,R) = Snb (A,R), de donde

Sn+1b (B,R) =

⋃y∈Snb (B,R)

Sy(B,R) =⋃

y∈Snb (A,R)

Sy(A,R) = Sn+1b (A,R).

2.3 Lema. Sea R una relacion conjuntista en A.

(a) cl(a) es la menor subclase cerrada de A que contiene a Sa.

(b) cl(a) ∪ {a} es la menor subclase cerrada de A a la cual le pertenece a.

94

(c) cl(a) =⋃x∈Sa cl(x) ∪ {x}1.

(d) Bajo la relacion ∈ en V, Sx = x y Sn+1x =

⋃Snx para n ∈ ω (intuitivamente, Snx resulta de

aplicar n veces la operacion de union generalizada a x).

(e) Una clase T es cerrada en 〈V,∈〉 si y solo si T es transitiva.

(f) tcl(x) es la menor clase transitiva que contiene a x.

(g) tcl(x) ∪ {x} es la menor clase transitiva a la cual le pertenece x.

Demostracion. (a) De la definicion de cl(a) es claro que Sa ⊆ cl(a). Ahora, si x ∈ cl(a) entonces

existe n ∈ ω tal que x ∈ Sna y, luego, Sx ⊆ Sn+1a ⊆ cl(a). Por otra parte, supongamos que

B ⊆ A es cerrada y Sa ⊆ B. Por induccion se sigue directamente que Sna ⊆ B para todo

n ∈ ω, de donde cl(a) ⊆ B.

(b) Es claro que cl(a) ∪ {a} es cerrada, pues cl(a) es cerrada y Sa ⊆ cl(a). Sea B una subclase

cerrada de A tal que a ∈ B. Luego Sa ⊆ B y, del inciso anterior, cl(a) ⊆ B. Por lo tanto,

cl(a) ∪ {a} ⊆ B.

(c) Es facil probar que la union de subclases cerradas es una subclase cerrada, de donde se sigue

que⋃x∈Sa cl(x) ∪ {x} es cerrada y, como contiene a Sa, de (a) se concluye que cl(a) ⊆⋃

x∈Sa cl(x)∪ {x}. Por otra parte, si x ∈ Sa entonces x ∈ cl(a), de donde cl(x)∪ {x} ⊆ cl(a)

por (b). Por lo tanto,⋃x∈Sa cl(x) ∪ {x} ⊆ cl(a).

(d) Para cualquier conjunto x, Sx = Sx(V,∈) = {z ∈ V / z ∈ x} = x y, luego, Sn+1x =⋃

z∈Snx Sz =⋃z∈Snx z =

⋃Snx para n ∈ ω.

(e) T es cerrada en 〈V,∈〉 si y solo si ∀x∈T(Sx ⊆ T), lo cual equivale a que ∀x∈T(x ⊆ T), es

decir, T es transitiva.

(f) De (a), tcl(x) = cl(V,∈)(x), es la menor subclase cerrada en 〈V,∈〉 que contiene a Sx de donde,

por (d) y (e), es la menor clase transitiva que contiene a x.

(g) Directo de (b) y (e).

1Esta union es un abuso del lenguaje, pues representa una union de colecciones que posiblemente no son conjuntos.Su buena definicion se justifica de que representa la clase {z / ∃x∈Sa(z ∈ cl(x) ∪ {x})}.

95

Al considerar el Axioma del Infinito basta con indicar que una relacion es bien fundada y con-

juntista para determinar que satisface los principios de Recursion e Induccion. Al prescindir de

dicho Axioma hay que considerar la hipotesis adicional de que todas las clausuras de la clase son

conjuntos. La razon por la cual presentamos los resultados de esta forma es para probar que la

Recursion Transfinita de Ordinales (Teorema 1.5.9) no depende del Axioma del Infinito.

2.4 Teorema (Induccion Transfinita). Sea R una relacion bien fundada y conjuntista en A tal que

cl(a) es conjunto para todo a ∈ A (esta ultima hipotesis es redundante bajo el Axioma del Infinito).

Entonces

(a) Toda subclase no vacıa de A tiene un elemento R minimal.

(b) Si ϕ(x) es una formula tal que ∀a∈A ((∀z∈Saϕ(z))⇒ ϕ(a)) entonces ∀a∈Aϕ(a).

Demostracion. (a) Supongamos X ⊆ A no vacıa y sea a ∈ X. Si X ∩ cl(a) = 0 entonces a

es R-minimal en X, pues Sa ∩ X = 0. Por el contrario, si X ∩ cl(a) es no vacıa, como es

conjunto entonces tiene un elemento R-minimal, sea este z. Luego, si x ∈ A y xRz entonces

x ∈ Sz ⊆ cl(a), por lo cual x /∈ X segun la R-minimalidad de z en X ∩ cl(a). Por lo tanto, z

es R-minimal en X.

(b) Sea X = {a ∈ A / ϕ(a)} y supongamos que A\X es no vacıo. Por el inciso anterior esa clase

tiene un elemento R-minimal a, por lo cual ∀z∈Saϕ(z) y, por la hipotesis, se sigue ϕ(a), es

decir, a ∈ X, lo cual es una contradiccion.

Notese que para definir cl(a) utilizamos recursion finita, lo cual puede parecer un razonamiento

circular al no haber probado ningun principio de Recursion hasta el momento. Hay dos razones

por la cual no estamos haciendo un razonamiento circular: primero, que cl(a) se define a partir de

Recursion en ω, el cual se puede probar independientemente sin ningun problema y, segundo, cl(a)

se puede definir sin Recursion segun el Ejercicio 3.5.

El principio de Recursion Transfinita que exponemos contiene la idea general de que podemos

definir una funcion de modo que la imagen de un termino esta definida segun las imagenes de los

terminos “que lo preceden”.

2.5 Teorema (Recursion Transfinita). Sea R una relacion bien fundada y conjuntista en A tal que

cl(a) es conjunto para todo a ∈ A (esta ultima hipotesis es redundante bajo el Axioma del Infinito).

Si F : A×V→ V entonces existe una unica funcion G : A→ V tal que G(a) = F(a,G�Sa).

96

Demostracion. Veamos primero la unicidad, pues esta se utilizara para probar la existencia. Sean

G y G′ dos funciones que satisfacen la recursion y veamos, por induccion transfinita en R, que

G(a) = G′(a) para todo a ∈ A. Supongamos que G(x) = G′(x) para todo x ∈ Sa, es decir,

G�Sa = G′�Sa, por lo cual G(a) = F(a,G�Sa) = F(a,G′�Sa) = G′(a).

Para probar la existencia vamos a definir la funcion G por etapas, es decir, vamos a probar que,

dado a ∈ A, existe una unica funcion ga : cl(a)∪{a} → V tal que ga(x) = F (x, ga�Sx) para todo

x ∈ cl(a) ∪ {a} (recordemos que, como cl(a) ∪ {a} es cerrado, Sx ⊆ cl(a) ∪ {a}). La unicidad

es directa de la prueba de la unicidad de este Teorema. Antes de probar la existencia de cada ga,

veamos que tienen que ser compatibles entre sı. En efecto, sean a, b ∈ A, ga y gb segun la recur-

sion definida. Si x ∈ (cl(a) ∪ {a}) ∩ (cl(b) ∪ {b}) entonces, como esta interseccion es cerrada2,

se sigue por 2.3 que cl(x) ∪ {x} ⊆ (cl(a) ∪ {a}) ∩ (cl(b) ∪ {b}). Sean g1x = ga�cl(x) ∪ {x} y

g2x = gb�cl(x)∪{x}. Luego, dado z ∈ cl(x)∪{x}, g1x(z) = ga(z) = F(z, ga�Sz) = F(z, g1x�Sz)

y de la misma forma se prueba que g2x(z) = F(z, g2x �Sz). Por lo tanto, g1x = g2x = gx y

ga(x) = g1x(x) = g2x(x) = gb(x).

Ahora probemos la existencia de cada ga por induccion transfinita. Supongamos que existe gx(recordar que es unica) para cada x ∈ Sa. Como los gx son compatibles entonces h =

⋃x∈Sa gx

es una funcion, su dominio es⋃x∈Sa(cl(x) ∪ {x}) = cl(a) (ver 2.3) y, para todo x ∈ cl(a),

h(x) = gx(x) = F(x, gx�Sx) = F(x, h�Sx). Ahora definamos ga : cl(a) ∪ {a} → V tal que

ga(z) ={h(z) si z ∈ cl(a)F(a, h�Sa) si z = a.

Esta funcion esta bien definida, aun cuando a ∈ cl(a) (lo cual es implosible porque R es bien

fundada en A, ver Ejercicio 3.5) pues, de ser ası, existe x ∈ Sa tal que a ∈ cl(x)∪ {x} = dom(gx)

y gx(a) = F(a, h�Sa). De cualquier forma, se obtiene que ga(z) = F(z, h�Sz) = F(z, ga�Sz) para

todo z ∈ cl(a) ∪ {a}.Como tenemos la existencia unica de ga para cada a ∈ A y como cada una es conjunto y son

compatibles entre sı, entonces G =⋃a∈A ga esta bien definida, su dominio es A y G(a) = ga(a) =

F(a, ga�Sa) = F(a,G�Sa).

2.6 Corolario. (a) Sea 〈A,R〉 es un buen orden tal que todos sus segmentos iniciales son con-

juntos. Si F : A × V → V entonces existe una unica funcion G : A → V tal que

G(a) = F(a,G�Sa).

2Es facil probar que la interseccion de cerrados es cerrado, de hecho, basta con que solo un miembro de la interseccionsea cerrado.

97

(b) (Teorema 1.5.9) Dado C ⊆ ON transitivo y F : C × V → V, existe una unica funcion

G : C→ V tal que G(α) = F(α,G�α).

Demostracion. (a) Para un buen orden 〈A,R〉 cuyos segmentos iniciales son conjuntos, es claro

que cl(a) = Sa porque Sa es cerrado. Por lo tanto, R es bien fundado en A, conjuntista, y sus

clausuras son conjuntos, es decir, satisface las condiciones del Teorema 2.5.

(b) Directo de que 〈C,∈〉 es un buen orden cuyos segmentos iniciales son conjuntos. Del inciso

anterior se sigue directamente el resultado, ademas que Sα(C,∈) = Sα(V,∈) = α para todo

α ∈ C (por la transitividad de C y 2.3).

2.7 Ejemplo. Sean A y R como en la hipotesis del Teorema 2.5. Sea F : A×V→ ON tal que

F(a, r) ={

supx∈dom(r){r(x) + 1} si r es funcion y ran(r) ⊆ ON,0 en otro caso.

Por el Teorema 2.5 existe una unica funcion G : A → ON tal que G(a) = F(a,G �Sa) =

supx∈Sa{G(x) + 1}. Notese la similitud de esta funcion con la propiedad 1.4.(g) de la funcion

rank definida en WF. Esto motiva la siguiente definicion para ρAR = G.

2.8 Definicion. Sean A y R como en la hipotesis del Teorema 2.5. La funcion ρAR : A → ON

definida por Recursion como ρAR(x) = supz∈Sx{ρAR(z)+1} segun el ejemplo anterior, la llamamos

funcion rango en 〈A,R〉. Cuando A y R se sobreentienden en el contexto, solo escribimos ρ para

dicha funcion.

2.9 Lema. Sean A y R como en la hipotesis del Teorema 2.5.

(a) Si x, y ∈ A, xRy⇒ ρ(x) < ρ(y).

(b) Si B ⊆ A y b ∈ B entonces ρBR(b) ≤ ρAR(b).

(c) Si B ⊆ A es cerrado y b ∈ B, entonces ρBR(b) = ρAR(b).

Demostracion. (a) Si x, y ∈ A y xRy entonces x ∈ Sy, por lo cual ρ(x) < ρ(x) + 1 ≤ ρ(y).

(b) Por induccion sobre b ∈ B. Si para todo x ∈ Sb(B,R) se tiene ρBR(x) ≤ ρAR(x), entonces

ρBR(b) = supx∈Sb(B,R)

{ρBR(x)+1} ≤ supx∈Sb(A,R)∩B

{ρAR(x)+1} ≤ supx∈Sb(A,R)

{ρAR(x)+1} = ρAR(b).

98

(c) La misma prueba del inciso anterior, solo que el hecho de que B sea cerrado cambia las de-

sigualdades por igualdades.

Recordemos que llamamos a una clase A bien fundada cuando la relacion ∈ es bien fundada en

dicha clase. Vamos ahora a analizar el caso particular de las clases bien fundadas (por ∈).

2.10 Lema. A es una clase bien fundada si y solo si ∀x⊆A(x 6= 0⇒ ∃z∈x(x ∩ z = 0)).

Demostracion. Directo de que, si x ⊆ A es no vacıo, entonces ∃z∈x∀y∈x(y /∈ z) equivale a que

∃z∈x(x ∩ z = 0)).

Cabe preguntarse si existe una clase bien fundada maximal. De hecho, existe una clase bien

fundada maxima y esta es WF. Mas aun, podemos definir a WF sin necesidad del Axioma de

Partes y, ademas, probar todas las propiedades de la seccion 3.1 excepto aquellas que se refieren

directamente a nociones definidas con partes (ver Ejercicio 3.15).

2.11 Definicion. WF− = {x / tcl(x) es bien fundado}.

2.12 Lema. (a) WF− es transitivo.

(b) WF− es bien fundado.

(c) x ⊆WF− si y solo si x ∈WF−.

(d) Sea A una clase transitiva. Entonces A es bien fundada si y solo si A ⊆WF−.

(e) ON es bien fundada. Por lo tanto ON ⊆WF−.

(f) Las propiedades (a),(c)-(g) del Lema 1.6 se cumplen para WF− salvo las consideraciones del

rango (que aun no hemos definido).

Demostracion. (a) Supongamos que y ∈ x y que tcl(x) es bien fundado. Luego y ∈ tcl(x) y,

como tcl(x) es transitivo, tcl(y) ∪ {y} ⊆ tcl(x). Por lo tanto, de 2.2 se sigue que tcl(y) es bien

fundado.

(b) Sea u ⊆WF− no vacıo y tomemos x ∈ u. Si x∩ u = 0 entonces x es un elemento ∈-minimal

en u. Por otra parte, si x ∩ u 6= 0, como x ∩ u ⊆ tcl(x) ∩ u y tcl(x) es bien fundado, entonces

existe z ∈ tcl(x)∩u tal que z∩ tcl(x)∩u = 0. Pero z∩u = z∩ tcl(x)∩u = 0, pues z ⊆ tcl(x)

por la transitividad de tcl(x).

99

(c) Si x ⊆WF− entonces tcl(x) ⊆WF− por (a). Luego, como WF− es bien fundada, tambien

lo es tcl(x) y, por lo tanto, x ∈WF−. El recıproco es directo de la transitividad de WF−.

(d) Sea A una clase transitiva. Si A es bien fundada y x ∈ A, de la transitividad de A se sigue que

tcl(x) ⊆ A y, luego, tcl(x) es bien fundado, es decir, x ∈WF−. Por lo tanto, A ⊆WF−. El

recıproco es inmediato.

(e) Dado que ON esta bien ordenado por ∈, es una clase bien fundada. Luego, de (d), ON ⊆WF−.

(f) En la prueba del Lema 1.6 solo se usa la transitividad de WF y el Lema 1.5. Luego, por (a) y

(c), podemos imitar las mismas pruebas en WF−.

El siguiente resultado indica que las propiedades del Lema anterior no son niguna coincidencia

de la definicion de WF−.

2.13 Teorema (ZF−). WF = WF−

Demostracion. Como x ∈WF−⇔ x ⊆WF− del Ejercicio 3.2 se sigue que WF ⊆WF−. Por

el Axioma del Infinito se tiene que tcl(x) es conjunto para todo x ∈ WF−, lo cual permite hacer

Induccion en WF− para probar que WF− ⊆ WF. En efecto, si x ∈ WF− y Sx = x ⊆ WF

entonces x ∈WF por 1.5, lo cual concluye la induccion.

De esta forma, WF− corresponde a definir WF sin el Axioma de Partes. Por lo tanto, dejare-

mos de escribir WF− de aquı en adelante y al referirnos a WF en razonamientos que no utilizan

el Axioma de Partes, sabemos que esta definido como en 2.11 y que satisface el Lema 2.12, por lo

cual WF es la maxima clase bien fundada. Ademas, podemos definir la Jerarquıa Acumulativa y la

funcion rank dejando de lado el Axioma de Partes (pero recurriendo, sin embargo, al Axioma del

Infinito), como lo ilustra la siguiente

2.14 Definicion (ZF−−P). Dado x ∈WF definamos rank(x) = ρWF∈ (x) = supz∈x{rank(z)+1}

y Vα = {x ∈WF / rank(x) < α}. Con el Axioma de Partes estas definiciones coinciden con 1.1

y 1.3 en virtud del Lema 1.4, la unica diferencia es que, sin el axioma de Partes, no se puede probar

que Vα es un conjunto.

3.5 Ejercicio. Sea R una relacion conjuntista en A, a ∈ A.

100

(a) x ∈ Sna si y solo si existe una funcion f : n+ 2→ A tal que f(0) = a, f(n+ 1) = x y f(k+ 1)Rf(k)para todo k ≤ n.

(b) x ∈ cl(a) si y solo si existen n ∈ ω y f : n+ 2→ A como en el inciso anterior.

(c) cl(a) = Sa ∪⋃z∈Sa cl(a).

(d) Si R es bien fundada, entonces a /∈ cl(a). Sugerencia: Utilize el inciso (b).

Este ejercicio muestra una forma de definir Sna y cl(a) sin necesidad de suponer que R es conjuntista en A.

3.6 Ejercicio. Sea A una clase y R una relacion. Si H : A → ON es una funcion tal que, para todox, y ∈ A, xRy⇒H(x) < H(y), entonces R es bien fundada en A.

3.7 Ejercicio. Sea A una clase y R una relacion. Decimos que H : A → ON es una funcion de rango en〈A,R〉 si {H(z) / z ∈ Sa} es un conjunto para todo a ∈ A y H(a) = supz∈Sa{H(z) + 1}. Pruebe que

(a) Si existe una funcion de rango, es unica.

(b) Si existe una funcion de rango entonces R es bien fundada en A.

(c) Si existe una funcion de rango, entonces toda subclase no vacıa de A tiene elemento R-minimal y, porlo tanto, 〈A,R〉 tiene induccion transfinita.

(d) Si B ⊆ A es cerrado y H es una funcion de rango, entonces H[B] es transitivo (es decir, es un ordinalo es ON).

3.8 Ejercicio (ZF−−P). Sea A una clase y R una relacion.

(a) Si R esta bien fundada en A entonces no existe una funcion f : ω → A tal que ∀n∈ω(f(n+ 1)Rf(n)).

(b) (ZFC−−P). R es bien fundada en A si y solo si no existe una funcion f : ω → A tal que f(n+1)Rf(n)para todo n ∈ ω.

3.9 Ejercicio. Sean A y P clases, Rx una relacion para cada x ∈ P. Definamos la relacion RP en P ×Acomo (x, a)RP(y, b) si y solo si (x = y ∧ aRxb). Pruebe que, dado (x, a) ∈ P×A,

(a) S(x,a) = {x} × Sa(A,Rx). Aquı S(x,a) = S(x,a)(P×A,RP). Por lo tanto, si Rx es conjuntista en Apara cada x ∈ P, entonces RP es conjuntista en P×A.

(b) Supongamos que Rx es conjuntista en A para todo x ∈ P. Entonces Sn(x,a) = {x} × Sna (A,Rx) paratodo n ∈ ω.

(c) Si Rx es conjuntista en A para todo x ∈ P, entonces cl(P×A,RP)((x, a)) = {x} × cl(A,Rx)(a).

(d) Supongamos que Rx es conjuntista en A para todo x ∈ P. Si para todo x ∈ P las clausuras de 〈A,Rx〉son conjuntos, entonces las clausuras de 〈P×A,RP〉 son conjuntos.

(e) Si Rx es bien fundada en A para todo x ∈ P, entonces RP es bien fundada en P×A.

3.10 Ejercicio. Sea P una clase, A una clase y, para cada x ∈ P, sea Rx una relacion que satisface lashipotesis del Teorema de la Recursion (2.5) para A. Si F : P × A × V → V entonces existe una unicafuncion H : P×A→ V tal que H(x, a) = F(x, a,H�{x}×Sa(A,Rx)). Sugerencia: El ejercicio anteriorpermite aplicar Recursion en 〈P×A,RP〉.

3.11 Ejercicio. Sea A una clase y R una relacion que satisfacen las hipotesis del Teorema de la Recursion(2.5). Sea P una clase. Si F : P×A×V→ V entonces existe una unica funcion H : P×A→ V tal queH(x, a) = F(x, a,H�{x} × Sa).

101

3.12 Ejercicio. Sea A y R como en las hipotesis del Teorema 2.5. Definamos Aα = {x ∈ A / ρ(x) < α}.Pruebe que

(a) Los Aα son cerrados.

(b) A0 = 0.

(c) α < β ⇒Aα ⊆ Aβ .

(d) Si γ es lımite, entonces Aγ =⋃α<γ Aα.

(e) Si B ⊆ A es cerrado, entonces ρ[B] es transitivo.

(f) Si ∃x∈A(ρ(x) = β) entonces Aβ ( Aβ+1.

(g) De un ejemplo de A y R como en este ejercicio tal que Aα es clase propia para todo α 6= 0.

3.13 Ejercicio. Pruebe que tcl({x}) = tcl(x) ∪ {x}.

3.14 Ejercicio (ZF−−Inf ). Si x ∈WF pruebe que tcl(x) es un conjunto bien fundado.

3.15 Ejercicio (ZF−−P). Pruebe que rank definido en 2.14 cumple, si usar el Axioma de Partes,

(a) V0 = 0, Vα+1 = {x / x ⊆ Vα} y Vγ =⋃α<γ Vα.

(b) Los resultados de la seccion 3.1, excepto los incisos (b), (h) e (i) del Lema 1.6.

(c) Si A es una clase transitiva bien fundada, entonces ρA∈ (x) = rank(x) para todo x ∈ A.

3.16 Ejercicio (ZF−−P). Pruebe que

(a) Vn = Vn es un conjunto finito para todo n ∈ ω.

(b) Vω = Vω es conjunto y ∀x∈Vω (x finito).

(c) Sea x ∈WF. Entonces x ∈ Vω si y solo si tcl(x) es un conjunto finito.

(d) Definamos la siguiente relacion en ω: n ∈2 m si y solo si 2n hace parte de la expansion binaria de m.Entonces ∈2 es una relacion bien fundada y conjuntista en ω.

(e) Dado m ∈ ω, cl(ω,∈2)(m) ⊆ m.

(f) 〈Vω,∈〉 ∼= 〈ω,∈2〉. Sugerencia: Defina por recursion F : Vω → ω tal que F (x) =∑z∈x 2F (z).

(g) Vω es un conjunto enumerable.

3.3. Teorıa de Conjuntos con el Axioma de Regularidad

En esta seccion razonamos en ZF−−P−Inf , a menos que se indique lo contrario. A pesar

de la forma tan simple en que esta enunciado el Axioma de Regularidad, se tienen las siguientes

equivalencias, las cuales le dan sentido a este axioma.

3.1 Lema. Las siguientes afirmaciones son equivalentes.

(i) El Axioma de Regularidad.

102

(ii) V = WF.

(iii) Todas las clases son bien fundadas.

(iv) Todos los conjuntos son bien fundados.

Bajo el Axioma del Infinito, (iii) se puede cambiar por “toda clase no vacıa tiene un elemento

∈-minimal”.

Demostracion. (i)⇒ (ii) El Axioma de Regularidad dice, respecto a 2.10, que V es una clase bien

fundada. Por 2.12 se sigue que WF = V ya que WF es la maxima clase bien fundada.

(ii)⇒ (iii) Como V esta bien fundado, entonces toda clase (subclase de V) esta bien fundada.

(iii)⇒ (iv) Directo.

(iv)⇒ (i) Inmediato.

Aunque tiene mas contenido enunciar que V = WF es el Axioma de Regularidad, esta afir-

macion depende fuertemente de los otros axiomas de ZF−−P−Inf , por lo cual es mas practico

enunciar dicho Axioma como se propone en el capıtulo 1, pues de esa forma no se requieren otros

axiomas para definirse.

Muchos resultados y definiciones de esta seccion se prueban sin usar el Axioma de Regularidad

pero, en el caso de usarse, simplemente se puede cambiar WF por V. Por ejemplo, aunque el

siguiente resultado enuncia una clasificacion de ordinales para los elementos de WF, con el Axioma

de Regularidad esta clasificacion es valida para todos los conjuntos.

3.2 Corolario. Sea x ∈WF. Entonces

(a) x /∈ x.

(b) (ZF−−P). No existe una funcion F : ω →WF tal que F (n+ 1) ∈ F (n) para todo n ∈ ω.

(c) x es un ordinal si y solo si x es transitivo y linealmente ordenado por ∈.

Demostracion. (a) Como {x} ⊆WF entonces x es un elemento ∈-minimal en {x}. Por lo tanto,

∀y∈{x}(y /∈ x), es decir, x /∈ x.

103

(b) Si existiera dicha funcion, entonces ran(F ) ⊆WF y, por la buena fundacion de WF, entonces

ran(F ) tiene un elemento ∈-minimal, sea este F (m) con algunm ∈ ω. Pero F (m+1) ∈ F (m),

lo cual contradice la minimalidad de F (m).

(c) Como x ∈ WF entonces x es bien fundado, por lo cual todo subconjunto no vacıo de x

tiene elemento ∈-minimal. Por lo tanto, 〈x,∈〉 esta bien ordenado si y solo si esta linealmente

ordenado, lo cual prueba el resultado.

Definamos a continuacion el concepto del Colapso de Mostowski, herramienta la cual permite

transformar una clase bien fundada y conjuntista en una clase transitiva bien fundada. Para gener-

alizar la nocion de morfismo para ordenes parciales, decimos que F〈A,R〉 → 〈B,S〉 es un mor-

fismo si ∀x,y∈A(xRy⇒ F(x)SF(y)). F es un ismomorfismo si es biyectiva y xRy⇔ F(x)SF(y)

para todo x, y ∈ A.

3.3 Ejemplo. Sea 〈A,R〉 como en la hipotesis del Teorema de la Recursion (2.5). Consideremos

la funcion F : A × V → V tal que F(a, x) = ran(x). Por Recursion, existe una unica funcion

G : A→ V tal que G(a) = F(a,G�Sa) = {G(x) / x ∈ Sa}.

3.4 Definicion (Colapso de Mostowski). Sea 〈A,R〉 como en la hipotesis del Teorema de la Re-

cursion (2.5). Definimos la funcion colapso de Mostowski como la (unica) funcion G : A→ V tal

que G(a) = {G(x) / x ∈ Sa}. M = ranG se llama el colapso de Mostowski de 〈A,R〉.

3.5 Lema. Con la notacion del la definicion anterior.

(a) ∀x,y∈A(xRy⇒G(x) ∈ G(y)).

(b) M es una subclase transitiva de WF.

(c) (ZF−−P) Si x ∈ A entonces ρAR(x) = rank(G(x)).

Demostracion. (a) Directo de la definicion de G(y).

(b) Por induccion sobre 〈A,R〉 veamos que ∀x∈A(G(x) ∈ WF). Si ∀y∈Sx(G(y) ∈ WF) con

x ∈ A, entonces G(x) ⊆ WF y, por 2.12, G(x) ∈ WF. Por lo tanto, M = ranG ⊆ WF.

Para ver que es transitivo, basta ver que ∀x∈A(G(x) ⊆M), lo cual es inmediato de la definicion

de G.

104

(c) Por induccion sobre 〈A,R〉: si x ∈ A y ρAR(y) = rank(G(y)) para todo y ∈ Sx, entonces

rank(G(x)) = supz∈G(x)

{rank(z)+1} = supy∈Sx{rank(G(y))+1} = sup

y∈Sx{ρAR(y)+1} = ρAR(x).

De este modo, la funcion colapso es un morfismo sobreyectivo entre una clase bien fundada

y conjuntista con una clase transitiva bien fundada. Por lo general la funcion colapso no es un

isomorfismo, nisiquiera inyectivo. Bajo el siguiente concepto, si es un isomorfismo.

3.6 Definicion (Relacion Extensional). R es una relacion extensional en A si, para todo x, y ∈ A,

(∀z∈A(zRx⇔ zRy)) ⇒ x = y . Esto equivale a decir que Sx = Sy ⇒ x = y para todo x, y ∈ A.

En terminos de modelos, decir que R es extensional en A significa que el Axioma de Exten-

sionalidad es cierto en 〈A,R〉 (al interpretar el ∈ como R en A).

3.7 Lema. (a) Si R es extensional en A y B ⊆ A es cerrado, entonces R es extensional en B

(b) ∈ es extensional en V.

(c) ∈ es extensional en cualquier clase transitiva N.

Demostracion. (a) Inmediato de que Sx(B,R) = Sx para todo x ∈ B.

(b) Inmediato (de hecho, es el Axioma de Extensionalidad).

(c) Se sigue de (a) y (b) porque N es cerrado en 〈V,∈〉.

Intuitivamente, si A es una clase bien fundada y extensional, pero no es transitiva, significa que

hay elementos que no estan en A pero que estan en algun elemento de A, es decir, hay “huecos”

entre elementos de A. El Colapso de Mostowki lo que hace es bajar aquellos elementos que tienen

dichos huecos por debajo para taparlos, de modo que se obtiene una clase transitiva M que no tiene

huecos (es decir, es transitiva) y que es isomorfa a A.

3.8 Teorema (del Colapso de Mostoski). Sea R una relacion extensional en A que satisface las

hipotesis del Teorema de la Recursion. Entonces el colapso de Mostowski y la funcion colapso de

Mostowski son los unicos M transitivo y G tal que G : 〈A,R〉 → 〈M,∈〉 es un isomorfismo.

105

Demostracion. Sean G y M la funcion colapso y el colapso de Mostowski, respectivamente, de

〈A,R〉. Del Lema 3.5 solo basta probar que G es uno a uno para ver que G : A → M es un

isomorfismo. Veamos por induccion en 〈A,R〉 que ∀z∈A(G(x) = G(z)⇒ x = z) para todo

x ∈ A. En efecto, si G(x) = G(y), es decir, G[Sx] = G[Sy], de la hipotesis inductiva se sigue

que Sx = Sy y, como R es extensional, entonces x = y. Luego, G es isomorfismo ya que, dado

x, y ∈ A, xRy⇒G(x) ∈ G(y) y, para el recıproco, si G(x) ∈ G(y) entonces existe z ∈ Sy tal

que G(x) = G(z), de donde x = z ∈ Sy.

Veamos ahora la unicidad: si G′ : 〈A,R〉 → 〈M′,∈〉 es un isomorfismo con M′ transitivo, basta

probar que G′(x) = {G′(y) / y ∈ Sx} para todo x ∈ A. En efecto, como G′ es un isomorfismo,

entonces {G′(y) / y ∈ Sx} ⊆ G′(x). Ahora, si z ∈ G′(x), como M′ es transitivo, z ∈ M′, es

decir, existe y ∈ A tal que z = G′(y). Luego, G′(y) ∈ G′(x), por lo cual yRx, es decir, y ∈ Sx y

z = G′(y) ∈ {G′(y) / y ∈ Sx}.Por lo tanto, como G es la unica funcion tal que G(x) = {G(y) / y ∈ Sx} para todo x ∈ A,

concluımos que G′ = G y M′ = ranG′ = M.

Por otra parte, si T es subclase transitiva de una clase bien fundada A, T no tiene huecos entre

sus elementos y, por lo tanto, no es afectada por el Colapso de Mostowski, como se ilustra en el

siguiente resultado.

3.9 Corolario (ZF−−P). Sea A una clase bien fundada en donde ∈ es extensional. Consideremos

el isomorfismo G : A → M dado por el Teorema del Colapso de Mostowski (3.8). Si T ⊆ A es

transitiva, entonces ∀x∈T(G(x) = x).

Demostracion. Sea G′ = G � T. Dado que T es cerrado en A, entonces, para x ∈ T, G′(x) =

{G(z) / z ∈ Sx(A,∈)} = {G(z) / z ∈ Sx(T,∈)} = {G′(z) / z ∈ Sx(T,∈)}. Por lo tanto, G′

es la funcion colapso de 〈T,∈〉. Luego, como la funcion identidad idT : T→ T es un isomorfismo

y T es transitivo entonces, del Teorema 3.8, G′ = idT.

3.10 Ejemplo ((ZF−P). Definicion de P-nombres). Sea P un conjunto. Decimos que τ es un

P-nombre si y solo si τ es una relacion y, si (σ, p) ∈ τ , entonces σ es un P-nombre y p ∈ P.

Notese que esta definicion esta dada recursivamente por los elementos del dominio de τ que son P-

nombres. Veamos como el Teorema de la Recursion permite formalizar esta definicion. Definamos

x ∈P y como ∃p∈P((x, p) ∈ y). Notese que x ∈P y⇒ rank(x) < rank(y). Por lo tanto, esta

relacion es bien fundada en V pues, si A 6= ∅, con β = mınx∈A{rank(x)} cualquier z ∈ A tal que

rank(z) = β es un elemento minimal de A. ∈P tambien es conjuntista, pues Sx ⊆ dom(x). Ası,

106

〈V,∈P〉 cumple los principios de Induccion y Recursion. Definamos F : V×V×V→ {0, 1} tal

que

F(P, x, v) ={

1 si x es relacion, ran(x) ⊆ P y v : {P} × Sx(V,∈P)→ {1},0 en otro caso.

Por el Ejercicio 3.10, existe G : V×V→ {0, 1} tal que G(P, x) = F(P, x,G�{P}×Sx(V,∈P)).

Luego, decimos que un conjunto τ es un P-nombre sii G(P, τ) = 1. Esta definicion coincide con

la propuesta inicialmente en este ejemplo, pues

τ es P-nombre ⇔G(P, τ) = 1⇔ F(P, τ,G�{P} × Sτ (V,∈P)) = 1⇔ τ es relacion, ran(τ) ⊆ P y ∀σ∈Sτ (V,∈P)(G(P, σ) = 1)⇔ τ es relacion y ∀σ∀p((σ, p) ∈ τ ⇒ (σ es P-nombre ∧ p ∈ P)).

El concepto de P-nombre es la primera definicion correspondiente a la nocion de forcing: sea

M un modelo transitivo de ZFC, 〈P,≤〉 un quasiorden3 que pertenece a M y sea G ⊆ P un filtro4

(el cual no pertenece a M ). Se espera construir un modelo transitivo M [G] de ZFC que extienda a

M y en donde sean ciertas algunas afirmaciones que no lo son en M , como puede ser, por ejemplo,

¬CH5. M [G] se llama la extension generica de M respecto a P y G. La idea del modelo M [G] es

que, tanto sus elementos como sus afirmaciones, se puedan codificar enM y que esos “codigos” solo

se puedan decodificar con G. Como G /∈ M entonces en M no se pueden decodificar los objetos y

afirmaciones de M [G], a pesar de que contiene todos los codigos. Como M [G] es el menor modelo

transitivo de ZFC que contiene a M ∪ {G}, entonces en M [G] si se decodifican los codigos. En

terminos precisos, los codigos de los elementos de M [G] son los P-nombres que pertenecen a M y,

si τ es un P-nombre enM , denotamos por τG la decodificacion de τ . Estas nociones, independiente

de las consideraciones dadas previamente, se definen en el siguiente ejemplo.

3.11 Ejemplo ((ZF−P). Extensiones genericas). SeanM y P conjuntos,G ⊆ P yMP el conjunto

de P-nombres que pertenecen a M . Veamos como se define el conjunto M [G] ={τG / τ ∈MP

}donde τG = {σG / ∃p∈G((σ, p) ∈ τ)}. Definamos la relacion σ ∈G τ como ∃p∈G((σ, p) ∈ τ).

Como en el ejemplo anterior, la relacion ∈G es bien fundada y conjuntista en MP. De la definicion

3.4, si H : V → M es la funcion colapso y M el colapso de Mostowski de 〈V,∈G〉, entonces

τG = H(τ) corresponde a lo que se quiere definir, y M [G] = H[MP].

3≤ es reflexivo y transitivo en P. Por lo general, para hace forcing, se requiere que P tenga un elemento maximo 1.4G es no vacıo, ∀p,q∈G∃r∈G(r ≤ p, q) y ∀p∈G∀q∈P(p ≤ q⇒ q ∈ G).5Esta fue la forma como Cohen probo en 1963 la independencia de la Hipotesis del Continuo respecto a los axioma

de ZFC. Con esta misma tecnica probo la independencia de AC respecto a los demas axiomas de ZF, lo cual lo hizoacreedor de la medalla Fields.

107

Como otro ejemplo, veamos como el colapso de Mostowski brinda otra prueba de que Vω es

contable.

3.12 Ejemplo (ZF−−P). Segun el Ejercicio 3.16, la relacion ∈2 es una relacion bien funda-

da y conjuntista en ω y, ademas, es extensional en ω, pues si Sm(ω,∈2) = Sl(ω,∈2) entonces

m =∑

k∈Sm(ω,∈2) 2k =∑

k∈Sl(ω,∈2) 2k = l. Luego, del Teorema del Colapso de Mostowski (3.8)

aplicado a 〈ω,∈2〉 existe una unica clase transitiva T (la cual es conjunto) y un unico isomorfis-

mo G : 〈ω,∈2〉 → 〈T,∈〉 definido por G(m) = {G(k) / k ∈2 m}. Del Lema 3.5 es claro que

T ⊆ WF. Si G(k) ∈ Vω para cada k ∈2 m entonces G(m) ⊆ Vω y, como G(m) es finito, en-

tonces G(m) ⊆ Vn para algun n ∈ ω, de donde G(m) ∈ Vω. De esta forma hemos probado por

induccion en 〈ω,∈2〉 que T ⊆ Vω. Mas aun, son iguales. Veamos por induccion sobre n ∈ ω que

Vn ⊆ T . En efecto, V0 = 0 ⊆ T y, si Vn ⊆ T y x ∈ Vn+1, es decir, x ⊆ Vn ⊆ T , entonces x

es finito y, si definimos m =∑

z∈x 2G−1(z), obtenemos que k ∈2 m equivale a que G(k) ∈ x.

Luego, G(m) = {G(k) / k ∈2 m} = x y ası x ∈ T . En definitiva, como T = Vω entonces

G : 〈ω,∈2〉 → 〈Vω,∈〉 es un isomorfismo, por lo cual |Vω| = ω. La funcion F definida en el

Ejercicio 3.16 es la inversa de G.

Para concluir el presente capıtulo, veamos como el axioma de Regularidad permite definir el

cardinal de cualquier conjunto. Esto surge de que dicho axioma permite extraer, a cualquier clase

no vacıa A, un subconjunto no vacıo, lo cual se hace de manera uniforme. Si A es no vacıa, sea

αA = mın {α /A ∩ Vα 6= 0}. Definimos en general

τ(A) ={

A ∩ VαA si A 6= 0,0 si A = 0

donde τ(A) es un subconjunto no vacıo de A cuando esta clase es no vacıa.

Un problema que sucede al trabajar con una relacion de equivalencia R en una clase propia A

es que no se puede definir A/R (la coleccion de clases de equivalencias) porque es probable que las

clases de equivalencias sean en sı mismas clases propias y no conjuntos. Este problema se resuelve

en el siguiente Lema con la consideracion del parrafo anterior.

3.13 Lema (ZF−Inf ). Sea A una clase y E una relacion de equivalencia en A. Entonces existe

una funcion G : A→ V tal que ∀x,y∈A(xEy⇔G(x) = G(y)).

Demostracion. Dado x ∈ A, sea [x] = {y ∈ A / xEy} (el cual posiblemente no es conjunto, pero

al menos es una clase no vacıa) y definamos G(x) = τ([x]), el cual es un subconjunto no vacıo de

108

[x]. Sean x, y ∈ A. Si xEy entonces, como E es relacion de equivalencia, [x] = [y] y, por lo tanto,

G(x) = τ([x]) = τ([y]) = G(y). Por otra parte, si C = G(x) = G(y), entonces C ⊆ [x] ∩ [y] es

no vacıo, por lo cual existe z ∈ C y, como pertenece a [x] ∩ [y], se sigue que xEy.

Sabemos que la relacion equipotencia (≈) es una relacion de equivalencia en V. Puesto que

{y / x ≈ y} es una clase propia cuando x 6= 0, entonces no se puede definir |x| como esa clase de

equivalencia, pues se espera que |x| sea un conjunto. Sin embargo, como en el Lema 3.13, podemos

extraer de esa clase un subconjunto no vacıo y solucionar el problema. Sin embargo, como hemos

definido los cardinales para los conjuntos bien ordenados y esa definicion ha dado muy buenos

resultados en la Teorıa de Conjuntos, solo definiremos los cardinales de la forma mencionada para

conjuntos que no tengan buen orden, como se ilustra a continuacion.

3.14 Definicion (ZF−Inf ). Dado un conjunto x, definimos

|x| ={

mın {α / x ≈ α} si x tiene un buen orden,τ({y / x ≈ y}) si x no tiene un buen orden.

De esta forma, la clase de los cardinales queda extendida a CA = {|x| / x ∈ V}. Es claro

que CA ⊆ CA, por lo cual llamamos alephs o cardinales bien ordenados a los cardinales que

pertenecen a CA (es claro que, bajo AC, CA = CA, lo cual hace la Definicion 3.14 irrelevante).

De hecho, |x| ∈ CA si y solo si x tiene un buen orden, segun el siguiente resultado.

3.15 Lema (ZF−Inf ). x tiene buen orden si y solo si |x| es un ordinal.

Demostracion. Es claro que |x| es un ordinal cuando x tiene un buen orden. Veamos el recıproco.

Si x no tiene buen orden, entonces |x| = {y ∈ Vα / x ≈ y} donde α = mın{ξ / ∃y∈Vξ(x ≈ y)

}.

Es claro que x 6= 0 (pues 0 tiene buen orden), por lo cual x 6≈ 0 y 0 /∈ |x|. De la definicion de α

es claro que |x| 6= 0. Por lo tanto, |x| no es un ordinal, pues si lo fuera, entonces 0 < |x|, es decir,

0 ∈ |x|, lo cual no es cierto.

El siguiente resultado es el que realmente le da sentido a la definicion de |x|.

3.16 Teorema (ZF−Inf ). x ≈ y si y solo si |x| = |y|.

Demostracion. Dividamos la prueba en dos casos, cuando x tiene buen orden y cuando no. En el

primer caso, al suponer x ≈ y o |x| = |y| se tiene que y tiene un buen orden (Lema 3.15), por lo

cual la equivalencia se sigue del Teorema 2.1.4. En el segundo caso, al suponer x ≈ y o |x| = |y|se tiene que y no tiene buen orden, por lo cual la equivalencia se sigue del Lema 3.13.

109

En CA podemos definir tambien la relacion de desigualdad, suma, producto y exponenciacion.

Por lo tanto, el cardinal 2ω = |ω2| esta definido, solo que no se puede demostrar sin AC que este

cardinal es un aleph, es decir, que pertenece a CA. Por otra parte, aunque CA esta bien ordenado,

CA solo esta parcialmente ordenado, y su orden lineal equivale a AC como vimos en 2.3.5 (de la

misma forma que ∀a∈CA(a es infinito⇒ a · a = a)).

3.17 Ejercicio. Si 〈A,<〉 es un orden parcial bien fundado y extensional (por la relacion<), entonces 〈A,<〉es un orden lineal. Sugerencia: Suponga que A tiene un elemento que no es comparable con algun otro, seax un elemento <-minimal de los que cumplen dicha condicion y sea y un elemento <-minimal de los que noson comparables con x. Pruebe que Sx = Sy y llegue a un absurdo.

3.18 Ejercicio. Sea x ∈WF. Las siguientes afirmaciones son equivalentes.

(i) x es un ordinal.

(ii) x es transitivo y ∀y∈x(y transitivo).

(iii) x es transitivo y ∀y,z∈x(y ∈ z ∨ y = z ∨ z ∈ y).

(iv) x es transitivo y ∀y(x(y transitivo⇒ y ∈ x).

(v) ∀y∈x(S(y) ∈ x ∨ S(y) = x) y ∀y⊆x(⋃y ∈ x ∨

⋃y = x) (recordemos que S(y) = y ∪ {y}).

Sugerencia: Para (ii)⇒ (iii) pruebe que 〈x,∈〉 es un orden parcial bien fundado y extensional, luego utilizeel Ejercicio 3.17. Para (iv)⇒ (v) considere y = x ∩ ON, pruebe que es un ordinal y obtenga un absurdocon y ( x. Para (v)⇒ (i) sea α el menor ordinal que no pertenece a x y pruebe que x = α.

3.19 Ejercicio (ZF−Inf ). Sea ϕ(x, y) una formula, A un conjunto. Suponga que ∀x∈A∃yϕ(x, y). Pruebeque

(a) ∃B∀x∈A∃y∈Bϕ(x, y).

(b) Pruebe que AC implica que existe una funcion f tal que domf = A y ∀x∈Aϕ(x, f(x)).

3.20 Ejercicio (ZF). Sea R una relacion bien fundada en A. Pruebe que

(a) Toda subclase no vacıa de A tiene un elemento R-minimal. Sugerencia: Sea B ⊆ A una clase no vacıa.Defina la relacion S = {(x, y) ∈ B×B / x ∈ τ(Sy ∩B)} y pruebe que S es bien fundada y conjuntistaen B. Del Teorema 2.4 B tiene un elemento S-minimal x0. Pruebe que x0 es elemento R-minimal de B.

(b) Pruebe que A cumple el principio de Induccion del Teorema 2.4.

3.21 Ejercicio (ZF−−P). Dados x, y ∈WF definamos xRy⇔ x ∈ tcl(y). Pruebe que

(a) R es bien fundada y conjuntista en WF

(b) Sea G la funcion colapso para 〈WF,R〉. Pruebe que G(x) = rank(x) para todo x ∈WF.

3.22 Ejercicio (ZF−P). SeaP un conjunto. Para cualquier conjunto x definimos x = {(y, p) / y ∈ x ∧ p ∈ P}.Sea G ⊆ P no vacıo. Pruebe que

(a) Utilice adecuadamente el Teorema de la Recursion para definir x.

(b) x es un P-nombre.

110

(c) (x)G = x (ver ejemplo 3.11).

(d) M = V con M en el ejemplo 3.11.

(e) M ⊆M [G] con M [G] definido como en el ejemplo 3.11.

(f) M [G] es transitivo.

(g) Si M es contable, entonces tambien lo es M [G].

(h) Sea Γ = {(p, p) / p ∈ P}. Pruebe que Γ es un P-nombre y ΓG = G.

(i) Si M es un modelo transitivo de ZF entonces Γ ∈MP. Utilice este hecho para probar que G ∈M [G].

3.23 Ejercicio (ZF−Inf ). Dados a, b ∈ CA defina a ≤ b si y solo si existen x y y tal que |x| = a, |y| = by x � y. Defina tambien a < b si y solo si existen x y y tal que |x| = a, |y| = b y x ≺ y. Tambien decimosque un cardinal a es infinito si ∃x(x infinito ∧ |x| = a). Pruebe que

(a) x � y⇔ |x| ≤ |y|.

(b) x ≺ y⇔ |x| < |y|.

(c) < es un orden parcial en CA.

(d) ∀a,b∈CA(a < b⇔ a ≤ b ∧ a 6= b).

(e) ∀a,b∈CA(a ≤ b⇔ a < b ∨ a 6= b).

(f) ∀κ,λ∈CA(κ ∈ λ⇔ κ < λ) (el ultimo menor es el de CA. Esto indica que el orden como ordinales enCA coincide con el orden de CA.)

(g) Si a es un cardinal finito entonces a ∈ ω.

(h) Un cardinal a es infinito si y solo si ∀n∈ω(n < a).

(i) Si κ es un cardinal bien ordenado y a ≤ κ es un cardinal, entonces a es un cardinal bien ordenado.

(j) a < 2a.

(k) Todo conjunto de cardinales tiene una cota superior.

3.24 Ejercicio (ZF−Inf ). Dados a, b ∈ CA, definimos a+b = |x∪y| donde x y y son conjuntos disjuntostal que |x| = a y |y| = b. Definimos tambien a · b = |x × y| y ab = | yx| donde x y y son conjuntos talque |x| = a y |y| = b. Sabemos que estas operaciones estan bien definidas, lo mismo que sus propiedades,incluso con desigualdades. Pruebe que

(a) Estas operaciones suma y producto coinciden con la suma y el producto para CA.

(b) Si ωα ≤ a + b entonces ωα ≤ a o ωα ≤ b.

(c) Si ωα ≤ a · b entonces ωα ≤ a o ωα ≤ b.

(d) a + b = a⇔ b · ω ≤ a.

3.25 Ejercicio (ZF−Inf ). Dado a ∈ CA se define a+ = h(x) (el numero de Hartogs de x) donde |x| = a.Pruebe que

(a) a+ es un cardinal bien ordenado.

(b) ¬(a+ ≤ a, a+ ≤ 22a·ay a+ ≤ 222a

.

(c) Si a es un cardinal bien ordenado, entonces a+ (definido en el inciso anterior) coincide con el cardinalsucesor de a en CA.

111

3.26 Ejercicio (ZF−−Inf ). Un ordinal β es par si ∃δ(β = 2 · β), y es impar si ∃δ(β = 2 · δ + 1).

(a) Pruebe que x no tiene b.o.⇒ Inf .

(b) Si β no es un ordinal sucesor, entonces β = 2 ·β. Por lo tanto, todo ordinal no sucesor es par. Sugerencia:Pruebe por induccion generalizada que β no sucesor⇒ β = 2 · β. El caso β = 0 es trivial, ası quesuponga que β es lımite y que todos los α < β cumplen la afirmacion. Utilize una nueva induccion sobreα para probar que α < β ⇒ 2 · α < β.

(c) Si α es un ordinal infinito, entonces 2 × Vα ≈ Vα. Sugerencia: Pruebelo por induccion sobre α. Enel paso lımite no se necesita la hipotesis inductiva, solo defina funciones 1-1 F0 : Vα → V 0

α y F1 :Vα → V 1

α en donde V 0α = {x ∈ Vα / rank(x) par} y V 1

α = {x ∈ Vα / rank(x) impar}. Esto implicaque 2× Vα � Vα.

3.27 Ejercicio (ZF−Inf ). Si x no tiene buen orden, entonces ||x|| > |x|6. Sugerencia: Utilice (c) del ejerci-cio anterior.

6No se puede probar en ZF que existe una funcion F : V → V tal que x ≈ y⇔ F(x) = F(x) y x ≈ F(x) paratodo x, y ∈ V.

112